Download as docx, pdf, or txt
Download as docx, pdf, or txt
You are on page 1of 119

1| P h a r m a c o l o g y

Introduction to Pharmacology

What is a generic name?


- Chemical name of a drug
- Refers to the chemical make-up rather than the advertised brand
- A term referring to any drug marketed under its chemical name without advertising

What is drug classification?


- Similar chemical structures
- Same mechanism of action

Drugs that have similar chemical structures


Same mechanism of action
A related mode of action
Used to treat the same disease

SAMPLES OF CLASSIFICATIONS OF DRUGS


A. 5-Alpha-reductase inhibitor
B. Angiotensin II receptor antagonist
C. ACE inhibitor
D. Alpha-adrenergic agonist
E. Beta blocker
F. Dopamine agonist
G. Dopamine antagonist
H. Incretin mimetic
I. Nonsteroidal anti-inflammatory drug – cyclooxygenase drug
J. Proton-pump inhibitor
K. Renin inhibitor

Pharmacokinetics - Process of absorption, distribution, metabolism and elimination of drugs.

1. Absorption- movement of drug particles from GI tract to the body fluids.


2. Distribution- is the process by which drugs become available to the body fluids and tissues.
3. Protein Binding- portion of the drug that is bound inactive.
4. Metabolism- the process by which the body inactivates or biotransforms drugs.
5. Hepatic first pass- drugs pass first in the liver before going to the blood.
6. Excretion/Elimination- main route is through kidneys.
7. Half Life- the time it takes for the one half of a drug to be concentrated to be eliminated.

Pharmacodynamics - Study how drugs affect the body.

1. Onset of Action- time it takes to reach the minimum effective concentration after the drug is
administered.
2. Peak of Action- occurs when the drug reaches its highest blood or plasma concentration.
3. Duration of Action- Length of time the drug has a pharmacologic effect.
4. Agonist- produce a response
2| P h a r m a c o l o g y

5. Antagonist- Blocks a response


6. Side Effects- Physiologic effect not related to the desired effect
7. Adverse Effect- more severe than side effect. It is dangerous and detrimental.
8. Toxic effects or toxicity- occurs when there is unintentional overdose of a drug.
9. Tolerance- a decreased responsiveness over the course of the therapy
10. Placebo effect- is a psychological benefit from a compound that may not have the chemical
structure of a drug effect.

RA 9165-Comprehensive Dangerous Drug Act 2002


- Creation of PDEA
- Nurses must administer controlled drugs with a witness
- Sharing of controlled drugs is prohibited

Dosage Calculation Conversion


- 1 grain = 60 mg
1 teaspoon = 5 ml
1 tablespoon = 15 ml
1 oz = 30 ml
1oz = 2 tbsp
1 lbs = 0.45 kg
1 kg = 2.2 lbs
1 cup = 180 ml
1 pint = 500 ml
1 quart = 1000ml

Calculating Oral Dosages


Formula:
D/S X Q= Amount to give

Example:
1. Give 150 mg of ibuprofen syrup every 6 hours for 7 days. The available stock dose is 150 mg/ml.
How many ml of syrup should be given to the patient?
Answer: 28 ml per week

Computation:
1. 150mg/150mg x 1 ml= 1ml
2. 1ml x 4 (6 hours in a day is x4)= 4ml/day
3. 4ml x 7 (7 days)= 28 ml per week
Take note: Dosage above 5000mg causes toxicity

Calculation of Flow Rates


1. Milliliters per hours
Formula: number of ml/number of hours= ml/hr

Example: Infuse 1000ml lactated ringer’s solution over 10 hours.


Answer: 100 ml/hr
Computation:
1000ml/10 hours= 100 ml/hr
3| P h a r m a c o l o g y

IV SETS
Microdrip- 60 gtts/min
Macrodrip- 15 gtts/ min

Formula:
Ml/hr x gtts/min all over 60= gtts/min

Pediatric Calculation

1. Fried’s Rule (under 2 y.o)


Formula: age in mos/ 150 lbs x adult dose= infant dose

2. Young’s Rule ( 2-12 y.o)


Formula: age in yrs./ age in yrs. + 12

3. Clark’s Rule
Formula: Child’s weight in lbs./ 150 lbs x adult dose

ANTIBACTERIALS
1. Bacteriostatic drugs – inhibit the growth of bacteria (inhibit growth to prevent multiplication)
2. Bactericidal drugs – kill bacteria
3. Culture and Sensitivity test (C&S) – can detect the infective microorganism
present in a sample and what drug can kill it.
4. Additive – equal to the sum of the effects of the two antibiotics
5. Potentiative – occurs when one antibiotic potentiates the effects of the second antibiotic,
increasing the effectiveness
6. Antagonistic – is a combination of a drug that is bactericidal and a drug that is bacteriostatic.
When these two drugs are used together, the desired effect may be greatly reduced.
7. Narrow-spectrum – primarily effective against one type of organism
8. Broad-spectrum – can be effective against both gram-positive and gram-negative organisms.

Mechanisms of Antibacterial Action


1. Inhibition of bacterial cell-wall synthesis( prevent bacterial cell wall to build up)
2. Alteration of membrane permeability (when taking antibiotics, the cell wall will allow fluids to enter
and it will swell, it will rupture and the cell will die)
3. inhibition of protein synthesis (when taking some antibiotics it will prevent the cell of the bacteria to
produce protein and if no protein, the cell will die) Ribosomes = protein (Rough ER)
4. inhibition of the synthesis of bacterial ribonucleic acid (RNA) and deoxyribonucleic acid (DNA) (A
cell cannot live without DNA, it will not multiply. No DNA = dead cell)

ANTIBIOTIC RESISTANCE
1. Self-Medicating (DO NOT SELF MEDICATE!!)
2. Don’t stop taking medications even to symptoms subsided
3. Repeated exposures
4| P h a r m a c o l o g y

4. Do not argue with the doctor’s order (do not demand a drug you don’t need)
5. Follow orders, take prescribed drugs religiously
6. Do not take antibiotics if it is a viral infection (Antibiotics are ineffective against viruses)
7. Do not give leftover/expired antibiotics
8. Give it round the clock

General Adverse Reactions to Antibacterials


1. Allergic (hypersensitivity) reactions, most common side effect = skin test!!
2. Superinfection (kills normal flora or the body)
– *E. coli – large colon, helps produces Vitamin K
– *Bile – large intestine that produces colon bacilli that produces brown color in stool
– *Lactobacillus – vagina, helps keep acidic environment
– *Improve normal flora: Drink yakult to increase colon bacilli, yogurt, probiotics, kimchi
3. Organ toxicity

PENICILLINS

Penicillin
- Penicillin’s beta lactam structure (beta lactam ring)interferes with bacterial-cell wall synthesis by
inhibiting the bacterial enzyme that is necessary for cell division and cellular synthesis.
- Can be both bacteriostatic and bactericidal
- Mainly referred to as beta-lactam antibiotics

Beta lactamase - enzyme produced by microorganisms, the penicillin cannot penetrate/activate, so


that penicillin cannot kill the bacteria

Pharmacokinetics – Penicillin
After oral administration, penicillin is absorbed mainly in the duodenum and upper jejunum of the
small intestine.

Pharmacodynamics – Penicillin
- Bactericidal in action

Penicillin: Mechanism of Action


- Penicillins enter the bacteria via the cell wall.
- Inside the cell, they bind to penicillin-binding protein.
- Once bound, normal cell wall synthesis is disrupted.
- Result: bacteria cells die from cell lysis.
- Penicillins do not kill other cells in the body.

Natural penicillins
- penicillin G (Pentids, Pfizerpen, Celinex)
- penicillin V (V-Cillin, Betapen-VK)
Aminopenicillins
- Amoxicillin (Amoxin)
- Ampicillin

Penicillinase-resistant penicillins
5| P h a r m a c o l o g y

- Cloxacillin (Ciclox)
- Dicloxacillin (Dynapen)
- Nafcillin (Nafcil)

Extended-spectrum penicillins
- piperacillin, ticarcillin, carbenicillin, mezlocillin

Penicillin-beta-lactamase inhibitor combination drugs:


– ampicillin + sulbactam = Unasyn
– amoxicillin + clavulanic acid = Augmentin
– ticarcillin + clavulanic acid = Timentin
– piperacillin + tazobactam = Zosyn

Beta-Lactamase Inhibitors
- inhibits the bacterial beta-lactamases, making the antibiotic effective and extending its
antimicrobial effect
- Oral use: Clavulanic acid (Augmentin, TImentin)
- Parenteral use: Sulbactam (Unasyn) and Tazobactam (Zosyn)

Side effects and Adverse Reactions


- Nausea, vomiting and diarrhea are common GI disturbances
- Severe allergic reaction leads to anaphylactic shock. Clinical manifestations of a severe allergic
reaction include laryngeal edema, severe bronchoconstriction with stridor, and hypotension.

CEPHALOSPHORINS
- administered when patient is sensitive to penicillin
- have a beta lactam structure and act by inhibiting the bacterial enzyme necessary for cell wall
synthesis

Pharmacokinetics
- Administered orally
- Food decreases absorption
- Administered parenterally.
- Not Absorbed in the GI tract.

Pharmacodynamics
- Inhibit cell wall synthesis

Indications:
- Pharyngitis
- Tonsillitis
- Lower resp tract infections
- Septicemia
- Meningitis
- Perioperative prophylaxis

CEPHALOSPHORIN GENERATIONS
6| P h a r m a c o l o g y

1. First Generation of Cephalosporins – effective against gram postivie bacteria and most gram
negative bacteria)

2. Second Generation of Cephalosporins – same effectiveness as first generation. Possess a broader


spectrum against other gram negative bacteria

*First and second generation of Cephalosporins are usually localized

3. Third Generation of Cephalosporins – same effectiveness of second and first generations. Also
effective against gram negative bacteria, less effective against gram positive bacteria

4. Fourth Generation of Cephalosporins – is similar to the third generation. It is resistant to most beta-
lactamase bacteria. Has broader gram positive coverage than the third generation.

* Third and fourth generation of Cephalosporins can cross


the blood-brain barrier

Cephalosporins: First Generation


- Cefadroxil
- Cephalexin (Keflex, Cefalin, Ceporex)
- Cephradine
- Cefazolin Na (Ancef and Kefzol)
- Good gram-positive coverage
- Poor gram-negative coverage used for surgical prophylaxis, URIs, otitis media

Cephalosporins: Second Generation


- Cefaclor (Ceclor)
- Cefoxitine (Mefoxin)
- Cefuroxime (Kefurox and Ceftin)
- Good gram-positive coverage
- Better gram-negative coverage than first generation
- Used prophylactically for abdominal or colorectal surgeries

Cephalosporins: Third Generation


- Ceftizoxime (Tergecin, Ceftizox)
- Ceftazidime (Fortaz, Fortum)
- Cefotaxime (Claforan)
- Cefixime (Suprax)
- Ceftriaxone (Rocephin) Most potent group against gram-negative Less active against gram-
positive Used for difficult-to-treat organisms such as Pseudomonas

Cephalosporin: Fourth Generation


- cefepime (Maxipime) Newest cephalosporin agents. Broader spectrum of antibacterial activity than
third generation, especially against gram-positive bacteria.

MACROLIDES
7| P h a r m a c o l o g y

- Azithromycin (Zithromax) – long half-life, up to 3 days


- Clarithromycin (Biaxin) – Klaricid
- Erythromycin (E-Mycin)
- administered orally or intravenously (IV)

Pharmacodynamics:
1. Macrolides suppress bacterial protein synthesis (no protein synthesis – attack ribosomes of the cell
to prevent protein synthesis)
2. Azithromycin (Zithromax) has up to a 40- to 68- hour half-life and is prescribed to be taken only
once a day for 3-5 days.

SIDE EFFECTS AND ADVERSE REACTIONS


- GI disturbances such as nausea, vomiting, diarrhea and abdominal cramping

NURSING PROCESS FOR MACROLIDES


1. Monitor patients for liver damage resulting from prolonged use and high dosage of macrolides such
as azithromycin. Signs of liver dysfunction include elevated liver enzyme levels (Alt and Ast) and
jaundice.
2. Administer oral azithromycin 1 hour before or 2 hours after meals. Give with a full glass of water,
not fruit juice. Give the drug with food if GI upset occurs.
3. Administer antacids either 2 hours before or 2 hours after azithromycin (decrease absorption of
drug)

NEVER GIVE ANTACID WITH OTHER DRUGS!!

Lacrosamides – very strong and expensive


Clindamycin (Cleocin) and Lincomycin (Lincocin)
- active against most gram positive organisms

Glycopeptides
1. Vancomycin (Vancocin) – when penicillin is resistant (because of self medication = superinfection),
will take a patient a longer time to heal.

Pharmacokinetics:
Vancomycin is given IV for severe infection.

Pharmacodynamics:
- Vancomycin inhibits bacterial cell wall synthesis (for it not to enter the cell wall)
Side Effects and Adverse Reactions
- Vancomycin may cause nephrotoxicity and ototoxicity.

*Ototoxicity results in damage of cranial nerve VIII, can result in a permanent hearing loss (auditory
branch) or temporary or permanent loss of balance (vestibular branch)

TETRACYCLINE
- inhibiting bacterial protein synthesis
- have bacteriostatic effect
8| P h a r m a c o l o g y

- Tetracycline in combination with metronidazole and bismuth subsalicylate is useful in treating


Helicobacter pylori, a bacterium in the stomach that causes peptic ulcer.
- Tetracycline should not be taken with magnesium and aluminum antacid preparations, milk products
containing calcium or iron containing drugs, because these substances bind with tetracycline and
prevent absorption of the drug.
- Should be taken with an empty stomach 1 hour or 2 hours after mealtime.

Side effects and Adverse Reaction


- Gi disturbances such as nausea, vomiting, and diarrhea
- Photosensitivity (sunburn reaction)

NURSING PROCESS FOR TETRACYCLINE


1. Advise patients to use sunblock and protective clothing (sunglasses, umbrella) during sun
exposure. Photosensitivity is associated with tetracycline.
2. Educate patients to avoid milk products, iron and antacids. Tetracycline should be taken 1 hour
before or 2 hours after meals with a full glass of water. If GI upset occurs, the drug can be taken with
non dairy foods.

AMINOGLYCOSIDES
- inhibiting bacterial protein synthesis
- are for serious infections

Pharmacodynamics:
- usually administered IV

Streptomycin
- first aminoglycoside available
- used to treat tuberculosis
- Because of ototoxicity and the bacterial resistance that can develop, it is infrequently used today.
Despite its toxicity, streptomycin is the drug of choice to treat tularemia and bubonic pneumonic forms
of plague.

• Parenteral anti-tb

Neomycin sulfate
- for liver diseases
- decreases bacteria in bowel
- preoperative bowel antiseptic abdominal surgery (for killing of colon basili to prevent E. coli from
spreading)

Side effects and Adverse Reactions


- ototoxicity and nephrotoxicity
- the nurse must assess changes in patient’s hearing, balance, and urinary output.

FLUOROQUINOLONES (QUINOLONES)
- used to interfere with the enzyme DNA gyrase
- Norfloxacin is indicated for urinary tract infections.
- Ciprofloxacin is approved for use for urinary tract infections
9| P h a r m a c o l o g y

- Levofloxacin (Levaquin) is used primarily to treat respiratory problems

NURSING PROCESS FOR FLUOROQUINOLONES


1. Administer levofloxacin 2 hours before or after antacids and iron products for absorption. Give with
a full glass of water. If GI distress occurs, drugs may be taken with food.
2. Inform patients that photosensitivity is a side effect of most fluoroquinolones. Patients should use
sunglasses, sun block, and protective clothing when in the sun.

ANTITUBERCULARS
- Antitubercular Drugs –take religiously, if not repeat!

Isoniazid (INH)
- When a person is diagnosed with TB, family members are usually given prophylactic doses of
isoniazid for 6 months to 1 year.
- Single drug therapy with isoniazid proved ineffective in treating TB, because resistance to the drug
developed in a short time.
- When a combination of antitubercular drugs was used, bacterial resistance did not occur, and the
duration of treatment was reduced from 2 years to 6 to 9 months.

Side effects and Adverse Reactions


- peripheral neuropathy (destroys nerves, tingling sensation/numbness)
- can be prevented by taking pyridoxine (VitaminB6)
- Hepatotoxicity is an adverse reaction to isoniazid, streptomycin, pyrazinamide and rifampin.

Side effects and Adverse Reactions


1. Isoniazid = hyperglycemia, hyperkalemia,hypophosphatemia, and hypocalcemia
2. Rifampin = body fluids orange, soft contact lenses permanently discolored
3. Pyrazinamide = gout, sensitivity to light, liver toxicity
4. Ethambutol = dizziness, confusion, hallucinations and joint pain
5. Streptomycin = ototoxicity, optic nerve toxicity, encephalopathy, angioedema, CNS and respiratory
depression, nephrotoxicity, and hepatotoxicity

NURSING PROCESS ANTITUBERCULAR DRUGS


1. Isoniazid (INH) 1 hour before or 2 hours after meals. Food decreases absorption rate.
2. give pyridoxine (B6) as prescribed with isoniazid to prevent peripheral neuropathy
3. encourage eye examinations (every 3 months) for
patients taking isoniazid and ethambutol, because these antitubercular drugs may cause visual
disturbances.
4. teach patients not to take antacids while taking antitubercular drugs, because they decrease drug
absorption. patient should also avoid alcohol, because it may increase risk of hepatotoxicity
5. inform patients taking rifampicin that urine, feces, saliva, sputum, sweat, and tears may turn a
harmless red orange color. soft contact lenses may be permanently stained

Metronidazole (Flagyl)
- intestinal antibiotic
- used to treat intestinal amebiasis
- classified as an antibacterial/antiprotozoal
10| P h a r m a c o l o g y

- commonly used with other agents to treat Helicobacter pylori, which is associated with frequent
recurrent peptic ulcers

Side effects and adverse reaction


- dark or reddish brown urine
- metallic taste
- nausea
- headaches
- Allergies
- seizures

PHARMACOLOGY REVIEWER FOR MIDTERMS

Pain Medications

Inflammation
Part of the first line defense
Response to tissue and injury infection
A vascular reaction takes place in which fluids, elements of blood, leukocytes(WBCs), and chemical
mediators accumulate
Supposed to be normal and counteract infection
Protective mechanism in which the body attempts to neutralize and destroy harmful agents at the site
of injury and to establish conditions for tissue repair

Infection
Caused by microorganisms and results in inflammation

Chemical Mediators

Histamine
o First mediator in inflammatory process
o Cause dilation of arterioles
o Increase capillary permeability

Kinins(Bradykinin)
o Increase capillary permeability
o Increase pain

Prostaglandin
o Increase capillary permeability
o Increase vasodilation
o Increase fever and pain

Pathophysiology

Cardinal signs
o Redness
o Swelling
11| P h a r m a c o l o g y

o Heat
o Pain
o Loss of function

Important Terms
We don’t reverse histamine and bradykinin

Prostaglandins (chemical mediator)


Effects: vasodilation, relaxation of smooth muscle, increased capillary permeability, and sensitization
of nerve cells to pain cause of inflammation and pain
Cyclooxygenase (COX)- Enzyme responsible for converting arachidonic acid to prostaglandins

COX 1- traditional; converts arachidonic acids to prostaglandins


o The stomach has prostaglandin coating= protection and regulate blood platelets
o COX 1 converted to COX 2= prostaglandins

COX 2- triggers inflammation and pain


Stopping the pathway= decreases inflammation

Earliest type of NSAIDs

Aspirin (salicylates)- Inhibit biosynthesis of prostaglandins (prostaglandin inhibitor)


o Prostaglandin inhibitor- Affect the inflammatory process (anti-inflammatory agents)
o Anti-inflammatory agents- relieve pain (analgesic), reduce elevated body temperature (antipyretic)
also an anticoagulant (platelet aggregation) to prevent stroke and MI; it is over the counter; not safe
for children; not any more used for inflammation and pain due to many side effects and adverse

Most NSAIDs are used to decrease inflammation and pain for patients who have some type of arthritic
condition
o For chronic pain or arthritis- NSAIDS is used

Anti-Inflammatory Drugs

Anti-inflammatory drug groups


o Non-steroidal and anti-inflammatory drugs
o Corticosteroids
o Disease modifying antirheumatic drugs
o Anti-gout drugs

Either steroidal or non-steroidal


Non-Steroidal Anti-Inflammatory Drugs
More appropriate for reducing swelling, pain, and stiffness of joints
Common but has a lot of side and adverse reactions
Common for arthritic pain, menstrual cramps, migraine, and toothache
COX 2- more potent in inflammation
1st- blocks cox 1; 2nd- blocks cox 2
Prostaglandin- potent inflammatory agent
12| P h a r m a c o l o g y

First generation of NSAIDs


Blocks COX 1 and COX 2, but blocks COX 1 first which prevents the conversion to COX 2
Ketorolac- given as IV; post-op patients
2nd gen
Cox 2 inhibitors- safer compared to the first generation

Aspirin
Not used as an antipyretic but anticoagulant
Gastric distress- prostaglandins lines the stomach w/o cox 1= Upper GI bleeding

Pharmacokinetics- if taken as anticoagulant should be given with meals; after lunch or 12 afternoon;
PC (post meals)

Hypersensitivity to aspirin
o Effects: tinnitus (ringing of the ears), vertigo (dizziness), anaphylaxis, and bronchospasm
(contraction of the bronchial muscles=difficulty in breathing)

Nursing process
Observe patients for signs of bleeding
o Melena- black tarry stools- sign of upper GI bleeding
o Bleeding gums, petechiae, ecchymosis, and purpura when taking high doses of aspirin
300 mg for pain; 81 mg for anticoagulant

Pharmacokinetics
Aspirin is well absorbed in the GI tract which can cause GI upset
o Should be taken with food, milk, or water
o Should not be taken on the last trimester of pregnancy= causes premature closure of the ductus
arteriosus

Nursing Interventions
Eliminate risk for injuries
Advise patient to not take aspirin with alcohol (increases gastric juices)
Educate patient to inform dentist if aspirin is taken
Instruct to discontinue aspirin 5 to 7 days before surgery to reduce bleeding
W/ dengue= cannot prescribe aspirin will cause platelet drop due to anticoagulant effect
Keep aspirin out of reach of children

Reye’s syndrome- problem for aspillet or aspirin


o Viral problem then given aspirin; <7 yrs old= causes liver damage= brain encephalopathy.
New formula for aspillet- neo-aspilet
Phenylacetic acid derivatives
Ketorolac (Toradol)

First injectable NSAID


Recommended for short term management of pain
o IV formula; usually given to post-op patients
Possible of GI bleeding and irritation
Because it is not safe it is used as creams
13| P h a r m a c o l o g y

Skin test first


Propionic acid derivatives
Ibuprofen
Reduce fever
Treat pain or inflammation caused by many conditions such as headache, toothache, back pain,
arthritis, menstrual cramps, or minor injuries
Nephrotoxicity and GI distress= GI ulcers= most common
Side and adverse effects of NSAIDs
Ototoxic- tinnitus
GI distress
Dizziness, confusion, and edema
Blood dyscrasia and dysrhythmias
Nephrotoxic- worst adverse reaction= acute kidney injury
Upper GI bleeding
CVD- in long term use
Allergy
Constipation
Selective COX-2 Inhibitors (Second generation NSAIDs)
Coxibs- end names
Celecoxibs (Celebrex) and Nabumetone (Relafen)
COX 2 inhibitors- decrease risk of peptic ulcers
o Clinical trials revealed effects= cardiovascular diseases and stroke
Designed for arthritis
NSAIDs are not really safe; prescriptions and advices are needed.
Anti-gout Medications
Anti—inflammatory gout medication
For elderly- gouts can cause bone and joint deformity
Colchicine- first drug to be given for gouts
Stops migration of WBCs to the injured site
o Reduce swelling= reduce pain
Gastric inflammation- a common problem
o Should be taken with food
o With high doses- nausea, vomiting, diarrhea, or abdominal pain occurs in approximately 75% of
patients taking the drug
Has drug interaction with antibiotics
Uric acid inhibitor
Prophylaxis/To prevent gouty arthritis- allopurinol; for maintenance only
Allopurinol (Zyloprim)- never given for acute pain
Inhibits final steps or uric acid biosynthesis
o Lowers serum uric acid levels, preventing the precipitation of an attack
Diet modification+allopurinol
Taken during chemotheraphy
12 | P h a r m a c o l o g y
Pharmacodynamics: xanthine oxidase inhibitor
Inhibits the production of uric acid by inhibiting the enzyme xanthine oxidase
o Hypoxanthine=xanthine=uric acid
o Needed in the synthesis of uric acid
If not converted to uric acid it would stay as a nonpotent serum
14| P h a r m a c o l o g y

Interferes with conversion of ATP/ADP


Side effects and adverse reactions- GI distress and dysrhythmias
Uricosurics
Increase excretion uric acid through urine
Probenecid-Affects the nephrons- In the distal tubule; prevent recycling of uric acid
Colchicine (prevent migration of WBC)+probenecid(excretion)= prevent acute attack
Use of aspirin should be avoided, because it causes uric acid retention
Corticosteroids- from the adrenal cortex
Prednisone, prednisolone, and dexamethasone
Control inflammation by suppressing or preventing the many components of the inflammatory process
Not the drug of choice for arthritis
o Due to numerous side effects
Frequently used to control arthritic flare ups
Last option for inflammation
Exact mechanism is unknown thought to decrease lysosomal activities
Decreases immune system
Can cause cushing’s syndrome as effect
o Buffalo hump
o Moon face
o Central obesity
Can also cause GI ulcers
Taper off when discontinuing- if patient is using a steroid tablet; it should not be immediately stopped
because it will cause hypertension.
o Gradually decrease dosages
Non opioid and opioid analgesic
NSAIDs- can be classified in non-opioid analgesic
Non opioid
Known as non-narcotic analgesic
Acetaminophen aka paracetamol- inhibit prostaglandin synthesis
Antipyretic and analgesic
Non-opioid but is not an NSAID
Pharmacokinetics
Short half-life can be administered every 4 hours as needed
o Max dose- 4g/day
o Frequent takes should limit the dose to 2 g/day to avoid possibility of hepatic or renal dysfunction
o Large doses or overdoses can be toxic to hepatic cells
o Adult- 10 days max
o Children- 5 days max
Or 1,000 mg Q6/day- abroad to prevent seizures
Antidote for paracetamol overdose- acetylcysteine- mucolytic (respiratory drug; lyses mucus
secretions) and also a liver protectant
Pharmacodynamics
Eliminates mild to moderate pain
Side effects and adverse reactions
o Overdoses- extremely toxic to the liver cells= hepatotoxicity= s/sx (NAVDA)
Opiod: Narcotic Analgesic
Known as narcotic analgesic
13 | P h a r m a c o l o g y
15| P h a r m a c o l o g y

Addiction- psychological and physical dependence upon a substance beyond normal volume control,
usually after prolonged use of a substance
Effects: analgesia, respiratory depression, euphoria, sedation, orthostatic hypotension (decrease in
blood pressure when rising from a sitting-lying position)
Pain=peripheral nerves=chemical mediators (histamine, bradykinin, and prostaglandins)
=spinothalamic tract= thalamus (pain is already determined) =parietal lobe(processing; pain)= frontal
lobe(reaction)
Non opioid- blocks only the chemical mediators
o Mild to moderate pain
Narcotic (opioids)- moderate to severe pain
o Acts on the brain; decreases function of brain so it cannot interpret pain; depresses CNS function
Patients can die due to apnea; respiratory arrest
Tramadol
Opioid medication used to treat moderate to severe pain
Often combined with paracetamol to relieve pain
1/10 the potency of morphine
Equally potent compared to pethidine and codeine
Can be in IV and tablet form
Vomiting and hypotension- side effect
Side effects and adverse reactions: it is a downer- not recommended for suicidal patients
o Constipation, itchiness, nausea, and seizures
Morphine
For chronic pain
Check PR, if below 101 bpm do not give morphine
Only used for patients with MI; last option
Patients with cancer and post-op patients
Antidote for morphine- Naloxone or Narcan
o When there is respiratory depression
Opioids does not only suppress pain impulses but also respiration and coughing by acting on the
respiratory and cough centers in the medulla of the brainstem
Nursing process for Morphine
Morphine overdose: pinpoint pupils and slower reaction to light; decreased RR; decrease gag reflex
Codeine
Analgesic but not as strong as morphine
o But also relieves pain and cough
1/15 to 1/20 potency but can also relieve mild to moderate pain
Most opioids (except meperidine; Demerol) has an antitussive effect= cough suppression= cause sore
throat and chest pain
A derivative of morphine
Used as an antitussive- cough suppression
Meperedine (Demerol)
Common for OB px; pacreatitis
Large doses= neurotoxicity; nervousness, tremors, agitation, irritability, and worst case: seizures
Not for long-term usage
Dose is limited to 600 mg/24-hour period no longer than 48 to 72 hours
Patient controlled analgesia(PCA)-Syringe pump a patient can control and it will deliver the drug;
everytime there is pain the patient can press it the Demerol will enter.
Fentanyl
16| P h a r m a c o l o g y

Can be mixed with anesthesia


Available in injections and dermal patches
Side effect: hypotension, decrease in HR
75 times stronger than morphine
Most widely used synthetic opioid
Hydromorphone (dilaudid)
A semisynthetic opioid similar to morphine
14 | P h a r m a c o l o g y
Analgesic effect is approximately 6 times stronger/more potent then morphine with fewer hypnotic
effects and GI distress
Patients with stage 4 cancer
Side effects and adverse reactions: respiratory depression, orthostatic hypotension, tachycardia,
drowsiness, and mental clouding
Withdrawal syndrome- irritability, diaphoresis, restlessness, muscle twitching, increased PR and BP,
agitation, and anxiousness
Side effects and Adverse Reactions of opioid analgesics In general
Anxiety, depression, flu-like symptoms, headache, GI upset, hallucinations, hypoventilation and apnea
Naloxone (Narcan)
Opiod antagonist
Administered IM or IV
Monitor for bleeding and hypotension
Anxiety
Anxiety is more on GABA- drugs decreases this
SSRI- used for depression; selective serotonin receptor inhibitor
Sleep and Insomnia
Following stage 4 of NREM sleep, will have 5-20 minutes of REM sleep with dreaming and increased
physiologic activity
REM sleep is felt to be mentally and emotionally restorative
REM deprivation can lead to psychological problems and psychosis
Insomnia
Can result from pain, anxiety, illness, changes in environment, and from certain medications
Benzodiazepines
Used for anxiety and insomnia
Prototype: diazepam
Can result in psychologic dependency, thus, abuse
Withdrawal symptoms can result if abruptly stopped; should be gradually tapered and discontinued
Can cause excessive sedation, impairment of physical and mental activities, and respiratory
depression
Not for long-term use
Do not suppress REM sleep
Vary in plasma half-lives, metabolites, and uses
Bind with benzodiazepine receptors in nerve cells of the brain, the receptor also has binding sites for
GABA
o When benzo/GABA binding occurs, then chloride ions in the cells causing decreased response to
the excitatory neurotransmitters such as norepinephrine
Competes with the GABA sites
Action potential- positive causes it
o Na goes in= electrical impulse
17| P h a r m a c o l o g y

o Blocking GABA= chloride (-) goes inside= less excitation= more relaxation
K
Pho
Pro
Mg
HCO3
Cl
Ca
Na
15 | P h a r m a c o l o g y
Classifications: muscle relaxants, hypnotics, and for anxiety
Pharmacokinetics
Well-absorbed orally
o Given oral or IV
Widely distributed in body tissues
Highly bound to plasma proteins
Lipid soluble; can easily enter the CNS
Metabolized by the liver by cytochrome p450 enzymes and by CYP3A4 enzymes in intestines
Most benzodiazepines are metabolized into active metabolites that require further metabolism before
clearance
Depending on half-life, can result in accumulation and subsequent adverse drug effects
High doses= decrease brain function (CNS depression), decreases vital signs
Do not give together with downer drugs
Shorter-acting benzos
o Versed(midazolam)- 30-60 minutes
o Halcion(triazolam)- 4-6 hours
o Dalmane(flurazepam)- 6-8 hours
o Xanax(alprazolam)-4-6 hours
given to px with cardiac conditions; MI patients
Given at 9 pm
One of the safe types of benzodiazepines
o Serax(oxazepam)- 2-4 hours
Longer-acting benzos
indicated for seizure px
o Klonopin(clonazepam)- action may last for weeks
o Librium(chlordiazepoxide)- several days
o Tranxene(chloraxepate)- lasts for days
Drugs and its common uses
Xanax(alprazolam)- anxiety and panic disorder
Librium(chlordiazepoxide)- anxiety and alcohol withdrawal
Klonopin(clonazepam)- seizure disorders and panic disorders
Valium(diazepam)- anxiety, seizure disorders, alcohol withdrawal, muscle spasms, and for
preoperation medication
o For active seizure give valium IV (for babies)
o If no IV, IM or anal
Dalmane(flurazepam)- insomnia
Ativan(lorazepam)- anxiety and preop medication
Versed(midazolam)- preop sedation and anesthetic induction
18| P h a r m a c o l o g y

Restoril(temazepam)- insomnia
Contraindication to use
Respiratory disorders
Severe liver or kidney disease
History of alcohol or drug abuse
o If there is alcohol abuse= higher doses are needed
Hypersensitivity reactions
o Skin tests are not needed= because allergic reactions are not common
Valium (Diazepam)
Valium is prescribed 30 minutes before operation
o Only lasts for 1 hour; short acting
o Decrease stress and anxiety
o Antidote- flumazenil
Used as hypnotics
Anti-anxiety meds may also be given to px with cardiac diseases
Miscellaneous antianxiety and sedative-hypnotic agents
Noctec (chloral hydrate)- chloroform
o Oldest sleeping medication
o Does not affect REM sleep
o Tolerance increases after two weeks
16 | P h a r m a c o l o g y
Natural hypnotics
o Melatonin-hormone- produced by the pineal gland
o Endogenous melatonin is derived from tryptophan= converted to serotonin= melatonin
o Melatonin affects sleep-wake cycles; released during sleep and levels are low during waking hours
o Used for jet lag due to disruption of circadian rhythms
o Caution with patients with liver or renal problems
Benzodiazepine withdrawal
Mild s/sx- sweating, irritation
o Occurs to half of clients after taking doses for 6-12 weeks or longer
o Severe s/sx if taking large doses for 4 months or longer with abrupt discontinuation
o Effects are related to decrease in GABA neurotransmission resulting to CNS stimulation
Signs and symptoms include: anxiety, psychomotor agitation, insomnia, irritability, headache, tremors,
and palpitations
o Others- confusion, depersonalization, psychosis, and seizures
Severe s/sx most pronounced in short-acting drugs such as Xanax, Ativan, and Halcion. Reduce dose
by 10-25% every 1-2 weeks over 4-16 weeks
Benzodiazepine toxicity
Effects include: excessive sedation, respiratory depression, and coma
Has shorter duration than many benzos, so, repeated dosing may be necessary
o For overdose, give 0.2 mg/30 seconds, wait 30 seconds, then 0.3 mg/30 seconds, then 0.5 mg/60
seconds up to max of 3 mg
Toxicity is common in geriatric patients and children
Antidote: Romazicon (flumazenil)
Naloxone is for opioids, flumazenil is for benzodiazepines
Drug therapy for anxiety
Not recommended for everyday stress
For chronic pain, have not proven to be effective
19| P h a r m a c o l o g y

Antipsychotics
Psychosis- severe mental disorder characterized by disordered thought processes, inappropriate
emotional responses, bizarre behavior, agitation, aggressiveness, hostility, social withdrawal,
deterioration in occupational and social functioning, hallucinations, and paranoid delusions
Nursing diagnosis: Altered thought processes
To be classified as psychotic: should be observed with paranoid delusions and hallucinations
Psychosis
Hallucinations- false interpretations of sensory perceptions
o Objects or people are absent
o Unable to distinguish between false perception and reality
o Schizophrenia or bipolar disorder- auditory
o Delirium- visual or tactile
o Dementia- visual
Common in alcoholics- delirium and visual or tactile
Illusions- objects are present but different interpretation
Delusions- false beliefs that persist in absence of reason or evidence
o Delusion of grandeur- perceives self with high standing
o May believe others control their thoughts, feelings, or seek to harm them
Psychosis may be acute or chronic
o Acute- confusion or delirium; can be precipitated by illness, drug effects, or superimposed on
chronic dementias
Schizophrenia
Consist of variety of related disorders
Does have a genetic predisposition
Positive symptoms- high brain activity
o CNS stimulation, agitation, behavioral disturbances, delusions, hallucinations, insomnia, and
paranoia
Negative symptoms- low brain activity
17 | P h a r m a c o l o g y
o Anhedonia, lack of motivation, blunted effect, poor hygiene, poor social skills, and social withdrawal
Etiology of schizophrenia
Evidence indicates abnormal neurotransmission systems in the dopaminergic, serotonergic, and
glutamatergic system
Interplay between the systems; one system may affect the other
Real reason is still unknown
Theoretical
o More on dopamine= increased levels which leads to psychosis
o Imbalance in amount of neurotransmitters, most notably dopamine
o Overacitivity accounts for the positive symptoms of schizophrenia and underactivity in another part
of the brain may account for the negative
Glutamatergic dysfunction may be genetically linked as well as causative in the cognitive impairments
and negative s/sx of this disorder
Antipsychotic drugs
Categorized as “typical”, “first generation”, or “conventional”= phenothiazine
“Atypical” or “second generation”= newer nonphenothiazine
Discovered in the 1950s- first generations
Second generations- modern drugs
Mechanism of action
20| P h a r m a c o l o g y

Most bind to D2 dopamine receptors and block the action of dopamine but positive effects only occur
overtime
Theory: blockage of dopamine receptors leads to changes in receptors with effects on cell metabolism
and function
With chronic drug administration, it is postulated that drugs re-regulate the abnormal
neurotransmission systems
First generations are only effective to positive symptoms
Indications of Phenothiazine
Schizophrenia
o Psychotic symptoms associated with brain impairment (injuries)
o Useful in manic phase of bipolar affective disorder until Lithium (drug of choice) becomes effective
Antipsychotics may also be given to intractable hiccups
Uses
Schizophrenia
o Nausea and vomiting- affect chemoreceptor trigger zone in medulla
o Intractable hiccups- mechanism of action is unclear
Use with caution in:
o BPH
o Seizure disorders
o Glaucoma
Contraindications of Phenothiazine
Liver damage
CAD
Cerebrovascular disease
Parkinsonism
Bone marrow depression
Severe hypotension and hypertension
Before administering check the vital signs
Pharmacokinetics
PO or IM
Prototype: Thorazine(chlorpromazine)
Metabolized by the cytochrome p450 system
No psychological dependency but physical dependency can occur
Withdrawal s/sx may occur
18 | P h a r m a c o l o g y
Side effects include:
o CNS depression
o Anticholinergic effects
o Antiemetic effects
o Lowering of body temperature
o Hypersensitivity reactions
o EPS
o Weight gain
o Orthostatic hypotension
Examples of phenothiazines
o Thorazine(chlorpromazine)
o Prolixin(fluphenazine)
o Compazine(prochlorperazine)
21| P h a r m a c o l o g y

o Stelazine(trifluoperazine)
o Mellaril(thioridazine)- used less commonly due to cardiac side effects
Thorazine (chlorpromazine)
1st generation phenothiazine- has higher risk of having the side effects
For psychosis or post-op patients with hiccups
EPS- extrapyramidal symptoms= parkinsonism
o Robot-like
o Extrapyramidal Symptoms (EPS)
Extrapyramidal Symptoms (EPS)
Affects EPS and basal ganglia
o System includes descending fibers that reach the medulla other than by the corticospinal tracts
o Important in maintenance of equilibrium and muscle tone
o Symptoms: dystonia, akathisia, tardive dyskinesia, and parkinsonism
Dystonia- prolonged muscle contractions causing twisting and repetitive movements or abnormal
posture. May have rhythmic jerks
Akathisia- restless, unable to sit still
o Most common symptom
Chloreiform movements- involuntary muscular twitching
Tardive dyskinesia- hyperkinetic movements of the face
o Sucking and smacking lips, facial grimaces, and tongue protrusion
Non-phenothiazines
1st generation antipsychotics
Haldol(haloperidol)- potent, long-acting
Causes high incidence of EPS
Useful in mental retardation with hyperkinesia, Tourette’s and Huntington’s disease
Comes in oral form and even once in a month injection form
Loxitane(loxapine)
Moban(molindone)
Orap(pimozide)- for Tourette’s when Haldol is not effective
o Can cause tardive dyskinesia, motor seizures, and even sudden death
2nd generation of antipsychotics
Drugs of choice
Effective in treating the positive s/sx of psychosis and have greater effectiveness in relieving the
negative s/sx
Less likely to cause EPS
Effective for both positive and negative symptoms
Clozaril (clozapine)
Prototype of atypicals
Effective but considered a second line drug because of association with agranulocytosis
Check weekly WBCs
Neutrophils will go down to <50
Haldol (Haloperidol)
19 | P h a r m a c o l o g y
A mild sedative
IM injection
Zyprexa(olanzapine)
Can cause EPS but not agranulocytosis
Cause less sedation, less orthostasis, and anticholinergic effects
22| P h a r m a c o l o g y

Seroquel(quetiapine)
Blocks dopamine and serotonin
Relieves positive and negative symptoms
Many drug interactions
o Metabolized by the cytochrome p450 system
Risperdal(risperidone)
Blocks dopamine and serotonin
Affects both positive and negative symptoms
Safest and is the first choice of treatment
Also metabolized by the cytochrome p450 system
Can cause parkinsonism
2nd generations have better results and less probability to EPS
Duration of drug intake is for years or forever, once stopped psychosis will go back
Drug selection
Duration of therapy- generally for many years as relapses can occur
Drug withdrawal- can occur if medications are stopped abruptly
o Can result in cholinergic effects such as diarrhea, drooling, and insomnia.
o Should be tapered over several weeks
Atypicals are drugs of choice as they:
o May be more effective
o Produce milder adverse effects
o Patients display greater compliance in taking them
o Drawback include: glucose intolerance, weight gains, and also are costs
Treatment of EPS
Treat with anticholinergic antiparkinson medication such as:
o Benadryl(diphenhydramine)
o Symmetrel(amantadine)
o Eldepryl(selegiline)
More likely to occur with older antipsychotic drugs
Treatment is usually for three months then gradual discontinuation
S/sx generally do not recur
Neuroleptic malignant syndrome (NMS)
Rare but potentially fatal reaction
o May occur hours to months after initial psychotic drug use
Present with high grade fever, muscle rigidity, agitation, confusion, delirium, tachycardia, respiratory
failure, and acute renal failure
Tx- stop drug, supportive care
o Dantrolene and amandatine- anti Parkinson drugs; also used to counteract NMS
No prophylactic for psychosis
Haloperidol- has highest EPS probability; 2nd is thorazine
Benadryl- anti-histamine to counter act side effects: EPS
Antidepressants and mood stabilizers
Mood disorders
Include: depression, dysthymia
o bipolar disorder (manic/panic= span of 2 weeks)
o cyclothimia- sad then happy but does not reach manic-panic situation
20 | P h a r m a c o l o g y
o Seasonal affective disorders- moods varies on the season
23| P h a r m a c o l o g y

If there was one depressive episode, higher risk for having another
Monoamine neurotransmitter dysfunction
Complex ethiology affecting neurotransmitter and receptors
Felt to be partially a result of deficiency in norepinephrine and serotonin
Interplay bet neurotransmitters: norepinephrine, serotonin, dopamine, and acetylcholine
Neuroendocrine factors
Increased secretion of corticotrophin-releasing hormone by hypothalamus, adrenocorticotrophic
hormone by pituitary, and cortisol by the adrenal gland
o Cortisol- steroid; acts on the fight or flight
o Increased cortisol may decrease numbers of sensitivity of cortisol receptors and lead to depression
Other factors
Interplay with thyroid and growth hormones
Immunity
May be linked in genetics
Environmental factors resulting in structural changes in brain such as child abuse
Types of mood disorders
Depression- imbalance of a neurotransmitter
o Lack of self-interest, anhedonia
Dysthmia- chronically depressed mood
o For at least 2 years
Bipolar- depression alternating with mania
Cyclothymia- mild type of bipolarity
Must be present for 2 years to be considered a form of depression
General characteristics of antidepressants
Vary in adverse effects
Must achieve serum level before improvement seen (2-4 weeks)
Taken orally, undergo significant first pass metabolism
Multiple drug interactions as are metabolized by cytochrome 450 enzymes
Medications will not work immediately
Mechanism of action
Normalize neurotransmission systems by altering the transmitters and receptors
Also modify interactions between neurotransmission system and endocrine functions
o ACTH (adrenocorticotrophic hormone) and cortisol levels
Neurotransmitters that are not bound are inactivated by reuptake or are metabolized by monoamine
oxidase
o Monoamine oxidase- enzyme that destroys norepinephrine and epinephrine- which is high during
stress
Contraindications
Used cautiously in schizophrenia, mixed mania, and depression
o Schizophrenia- can be used in positive symptoms
Suicidal tendencies
In severe renal, hepatic, or cardiovascular disease
Narrow-angle glaucoma- fluid buildup in the iris; increase intraocular pressure
Seizure disorders
Antidepressants
Tricyclics- oldest type of antidepressant
o More serious adverse effects especially anticholinergic and cardiac effects, weight gain, and
sedation
24| P h a r m a c o l o g y

o Can cause photosensitivity


o Elavil (Amitriptyline)- most common antidepressant
o Tofranil (imipramine)
o Sinequan (doxepin)
21 | P h a r m a c o l o g y
o Norpramin (desipramine)
Selective serotonin reuptake inhibitors (SSRI)
o Adverse effects include: nausea, sexual dysfunction, headache, increased risk of gastrointestinal
bleeding
o Safest antidepressant; most prescribed
o Never coadminister with monoamine oxidase inhibitor (MAOI)
o Prozac (fluoxetine)- long duration
o Zoloft (sertraline)
o Celexa (citalopram)
o Paxil (paroxetine)- long duration
Monoamine oxidase inhibitor (MAOI)
o Considered dangerous due to a lot of interactions with food and drugs
o Preserve norepinephrine
o Should not be combined with any other antidepressant
o Foods that contain tyramine, a monoamine precursor of norepinephrine, when taken with MAOIs
can lead to severe hypertension, stroke, or heart attack
o Diets should be checked
Tyramine can lead to severe hypertension (severe adverse effect of MAOI)
Avoid high in MSG
Avoid aged cheese, meats, concentrated yeast extracts, sauerkraut, and fava beans
o Marplan (isocarboxazid)
o Nardil (phenelzine)
o Parnate (tranylcypromine)
Lithium
o Mood stabilizing agent
o For bipolarity- mania phase
o Must be closely monitored
o Excreted by kidneys, so must have adequate renal functioning
o If hyponatremic= lithium toxicity
o This is toxic and can damage the kidneys
o Before administering check creatinine levels and clearance
o Diet should be on the normal salt level
2L water/2g salt
o Lithium toxicity
Therapeutic levels- should be checked weekly 0.6-1.2 MEQ(milliequivalent)/L
Severe toxicity- <2
Liver cannot detoxify because it is a mineral
o Mild toxicity- diarrhea, vomiting, fatigue
o Severe toxicity- If not excreted by the body adverse effects will occur
Seizure, coma, hypotension, hyperthermia, kidney failure, and death
Drugs acting on the CNS and PNS
Barbiturates
o Antisizure
25| P h a r m a c o l o g y

o Sedative
o Muscle relaxant
o Anti-anxiety
o Secobarbital (seconal)- treatment for insomnia
o Pentobarbital (Nembutal)- used in intracranial pressure
most common barbital
o Thiopental sodium (pentonal)- treatment for insomnia
o These are antiseizure medicines
Seizure- abnormal firing of the neurons; most common in pediatric patients
o Not used on ongoing seizures; due to long onset of action
Action of barbiturates
o Activates GABA receptors
o Delayed closing of chloride receptors
o Hyperpolarization of cells- becomes more negative polarity
Depression of CNS
Benzodiazepines
o Also considered as antiseizures
o Should only be used for 10 days
o Valium should be ready on bed side for active seizures (5mg; syringe); due to fast onset of drug
22 | P h a r m a c o l o g y
o Valium (diazepam); used to alleviate anxiety, and Klonopin (clonazepam)
o Valium administered IV during active seizures
o Dalmane (Flurazepam)
o Xanax (alprazolam)
o Restoril (temazepam)
o Ativan (lorazepam)- alleviate anxiety
Action of benzodiazepines
o Binds to GABA receptors
o Increases number of opening in the chloride channel
o Hyperpolarization of cells
CNS depression
o Epilepsy- To confirm epilepsy EEG is done; maintenance drug is phenytoin
Chronic, usually lifelong disorder
Classification of seizures
o Grand-mal(tonic-clonic); more on motor neurons
o Petit mal(absence)- has memory loss after
Action of anticonvulsants
o Antiseizures
o Suppresses sodium influx through the drug binding to the sodium channel= prevention of neuron
firing
o Suppress calcium influx
Valproic acid and ethosuximide are examples of drugs that suppress calcium influx
o Increase action of GABA= inhibit neurotransmitter throughout the brain
Identification of seizures
o Gran mal (tonic-clonic) and petit mal (absence)
o With the use of anticonvulsants 70% of seizures are controlled
o Take throughout the person’s lifetime
o In some cases, it is discontinued if no seizures have occurred in the past 3 to 5 years
26| P h a r m a c o l o g y

Hydantoins (classification); Phenytoin


o Prophylactic for seizures
o Not given with active seizures; just maintenance to avoid seizures
o Should not be given to pregnant patients; fetus may be aborted is taken in the 1st trimester
Teratogenic=congenital defects
o Maintenance: capsule form; wards: IV form and is only compatible with NS
If D5 water is used it will lead to precipitation=crystallization of the drug=embolus if given IV
Flush line with NS before and after administering to reduce venous irritation
Cannot be given as IM injection; irritates tissues and may cause damage
It is acidic, therefore, it may irritate patient if IV is out
o Effective for gran mal and psychomotor seizures; not effective for petit mal seizures
o Side effects and adverse reactions
Always check for gingival hyperplasia, because it is common for phenytoin
Overgrowth of gum tissues or reddened gums that bleeds easily
Thrombocytopenia-low platelet count
Leukopenia- low white blood cell count
Elevated blood sugar (hyperglycemia) results from the drug which inhibits the release of insulin
Baribiturates (Phenobarbital)
o Long-acting; effective for grand mal, partial seizures, and acute episodes of status epilepticus
Status epilepticus- rapid succession of epileptic seizures
o Can depress vital signs
Succinimides
o Zarontin (ethosuximide)- drug of choice for the absence or petit mal seizures
o Succinimide of choice
o Calcium influx inhibitor
Valproate (valproic acid)
o Hepatotoxicity is one if the possible adverse reactions
o Given as a maintenance drug; can be given every day for patients with epilepsy
Not be given to very young children and to patients with liver disorder
Can cause reye’s syndrome to children
Nursing considerations/process
o Phenytoin
o Teach patient to shake suspension form thoroughly before use to adequately mix medication and
assure accurate dosage
23 | P h a r m a c o l o g y
o Do not drive or perform hazardous activities when initiating anticonvulsant therapy
Drowsiness may occur
o If pregnant, consult with healthcare provider, phenytoin and valproic acid may have teratogenic
effects
o Avoid alcohol and other CNS depressants= cause added depressive effects on the body
o Do not stop abruptly, if medications are stopped abruptly effects will be reversed (seizure rebound)
o Pinkish or reddish brown urine= normal and harmless
Neuromuscular blocking agents
o Paralytic agents
o Given IV
o Atracurium
o Cisatracurium
o Rocuronium
27| P h a r m a c o l o g y

o Succinylcholine
Drug function of neuromuscular agents
o By blocking acetylcholine in the neuromuscular junctions (NMJ)
o Stop binding in the acetylcholine sites
Indications of neuromuscular blocking agents
o Skeletal muscle relaxant
o Used in endotracheal intubation
o Aid in muscle spasm in tetanus
o Reduce use of anesthesia
o Mechanical ventilation
o There should be an ambubag at bedside before administering paralytic agents
Nursing process
o Usually given by an anesthetist
o Assess for renal, hepatic, or pulmonary disease
o Assess for neurologic disorders such as
myasthenia gravis
Spinal cord injury
Multiple sclerosis- decrease in the sensations; problem in the myelin sheaths; slow transmission of
sensations
Anticonvulsants leads to CNS depression
Drugs affecting the autonomic nervous system
Autonomic nervous system
o Self-governing and automatic
o Composed of the sympathetic and parasympathetic
o In pharmacology
Adrenergic agents- sympathomimetic
Cholinergic agents- parasympathomimetic
Considered as inotropes- increases blood pressure; produces sympathetic response
Sympathetic- thoracic
Parasympathetic-vagus nerve
Drug class: Adrenergic agents
o Dopamine, dobutamine, norepinephrine, and epinephrine
o Used for shock; if <90/60 it is considered to be in hypovolemic shock
o If in 6 hours and not corrected it will lead to renal damage; if not corrected can cause multiple organ
damage
o Types of receptors
Alpha- vasoconstriction
Beta- heart rate
Dopaminergic- motivation, pleasure, cognition, memory, learning, and fine motor control, as well as
modulation of neuroendocrine signaling.
o Most common adrenergic drugs
Alpha 1- for vasoconstriction
Norepinephrine and dopamine
Beta 1- for heart rate (because of epinephrine)
Dopamine, dobutamine, norepinephrine, and epinephrine
o Indications
24 | P h a r m a c o l o g y
28| P h a r m a c o l o g y

Epi only has an effect for 5 minutes; does not have an overdose; used for cardiac arrest others are for
blood pressure
Cardiac arrest
Anaphylaxis
Superficial bleeding
Norepinephrine
used for hypotension
Treatment of cardiovascular problems
shock
Dopamine
Shock
Hypotension
Bradycardia
Dobutamine
Cardiogenic shock
Heart failure
Antihypertensive, diuretics, anticoagulants, and dyslipidemics
Hypertension
o Common in the US, up to 60 million afflicted
o Leads to MI, heart failure, stroke, and renal disease
MI- cessation of blood supply to the heart, necrosis
Heart failure- complication of MI; contractility of the heart
o Strong correlation with metabolic syndrome
Related to stress, lack of sleep
More on weight gain
Types of hypertension
o Essential or primary
Etiology/ cause: unknown
Contributors: include- salt sensitivity, insulin resistance, genetics (most common factor), sleep apnea,
environmental factors, etc.
o Secondary- renal, adrenal, coarctation of the aorta (dilation of the aorta), steroids, and pregnancy
Etiology: related to another disease condition/ underlying disease condition
BP review
o Any condition that affects the heart rate, stroke volume, or peripheral vascular resistance affects
arterial blood pressure
o Compensatory mechanisms to maintain balance between hypotension and hypertension
Normally, when the arterial blood pressure is elevated:
o Kidneys will excrete more fluid
Alpha I- BP
Alpha II
Receptors related to sympathetic= SA node and peripheral vascular resistance
-70 millivolts resting phase of SA node
Sodium goes to cell= increase conductivity to 70 MV
o Fluid loss will result in decreases extracellular fluid volume and blood volume
Blood volume- the more the blood volume, the higher the BP
o Decreased blood flow to the heart will reduce cardiac output
Renin-angiotensin-aldosterone system
o Decreased CO reduces arterial blood pressure
29| P h a r m a c o l o g y

Repolarization of cells
Potassium will go out; calcium takes place here- has effect contractility- prolongs; affects also
muscles (contractions; smooth muscles= vasoconstricition)- prevented by blocking calcium
o Beta receptors
Beta I- heart
Beta II- heart and lungs
o Vascular endothelium produces vasodilating substances (nitric oxide and prostatacyclin) which
reduces blood pressure
Nonpharmacologic management of hypertension
25 | P h a r m a c o l o g y
o Weight reduction
o Exercise
o Salt restriction in diet
o Stress reduction
o Moderation in alcohol intake
o If systolic BP cannot be maintained <140 systolic, time to take treatment
Antihypertensive drugs
Classes:
o Angiotensin converting enzyme (ACEI)
o Angiotensin II receptor blockers
o Antiadrenergic
o Calcium channel blockers
o Diuretics
o Direct vasodilators
o Renin inhibitors
Angiotensin converting enzyme I inhibitor
Blocks the enzyme that converts angiotensin I to angiotensin II (potent vasoconstrictor)
o Cannot totally block but only decrease
Have action of vasodilation and decrease in aldosterone production
Inhibit breakdown of bradykinins (vasodilator) prolonging effect
Reversed remodeling of heart muscle and blood vessels
Reno-protective
o Affects renin; also has a protection in the kidneys
Excellent for heart failure and hypertension
Improves post-myocardial infarction survival
Used alone or in combination
Useful in heart failure as decrease peripheral vascular resistance, cardiac workload and ventricular
remodeling
Captopril—prototype; most common
Low incidence of side effects
Can cause cough or hypotension when first started- normal and most common
Can cause hyperkalemia
Should never be used during pregnancy
May not be effective in African—americans- may add diuretic in this population to increase efficacy
Lotensin (benazepril)
Capoten (captopril)
Vasotec (enalapril)
Zestril (Lisinopril)
30| P h a r m a c o l o g y

Altace (Ramipril)
Aceon (perindopril)
Classes of hypertension
Classes of hypertension
Medications
Class I (140-149)
ACE I inhibitor and diuretics
Class II (150-159)
Vasodilators and diuretics
Class III (160-169)
Clonidine, beta blockers, and calcium channel blockers
Class IV (>170)
Clonidine, beta blockers, and calcium channel blockers
Angiotensin II receptor blockers (ARBs)
More potent than ACE I
Blocks effects of angiotensin II, compete with angiotensin II for tissue binding sites
Blocks the receptors in brain, kidneys, heart, vessels, and adrenal tissue
26 | P h a r m a c o l o g y
Has less side effects
Most common side effect: orthostatic hypotension
o Teach to slowly change position
SARTAN- end name of ACE II blockers
Similar end results as seen with ACE I
Less likely to cause hyperkalemia
Persistence of cough is rare
Prototype is Cozaar (losartan)
Atacand (candesartan)
Cozaar (losartan)
Diovan (valsartan)
o Recalled because it may cause cancer
o Because of rocket fuel contents- preservatives
Micardia (telmisartan)
Benicar (olmesartan)
Antiadrenergic
inhibit the activity of the sympathetic nervous system
blocking alpha receptors- epinephrine and norepinephrine
effective in decreasing heart rate, force of myocardial contraction, cardiac output, and blood pressure
short acting has a lot of side effects
Alpha 1 adrenergics receptor blocking agents dilate vessels and decrease peripheral vascular
resistance
Can experience first dose phenomenon with orthostatic hypotension, dizziness, syncope (can cause
head trauma, temporary loss of consciousness caused by a fall in blood pressure), possible sodium
and fluid retention
Antiadrenergics – Alpha 1
ZOSIN- end name of alpha-1
Blocks alpha I and causes vasodilation
Cardura- (doxazosin)
Minipress (prazosin)
31| P h a r m a c o l o g y

Hytrin (terazosin)
Alpha 2 antagonists
Centrally acting sympatholytics stimulate presynaptic alpha 2 receptors in the brain
Less norepinephrine is released and sympathetic outflow is reduced
Results in decrease cardiac output, heart rate, peripheral vascular resistance, and blood pressure
Vasoconstriction and has little effect on the heart rate
Centrally acting agents also can result in fluid and sodium retention
Catapres (clonidine)- orally or by patch
o Decrease BP; used in the community as emergency medication for sudden findings of hypertension
o Short acting
o > 180- hypertensive emergency
Tenex (guanfacine)
Aldomet (methyldopa)
Beta adrenergic blockers
Decrease heart rate, force of myocardial contraction, cardiac ouput and renin release from the
kidneys
27 | P h a r m a c o l o g y
Drug of choice for patients with tachycardia, angina, MI, left ventricular hypertrophy, and high renin
hypertension
Technically, it blocks beta 1
Beta 1- heart rate
o Can lead to exhaustion and myocardial death
Beta 2- heart and lungs
Angina- heavy crashing pain; sign of early possibility of MI
Pregnancy- teratogenic C and D (most is the phenytoin A)
Beta blockers I
End name OLOL
Inderal (propranolol)- hyperthyroid
o Increase HR due to T3 and T4
Inderal (propranolol)
Corgard (nadolol)
Lopressor (metoprotol)
Tenormin (atenolol)
Kerlone (betaxolol)- for glaucoma (ophthalmic)
o Hypertension- orally
IV-short acting
Oral- maintenance
If not selective- bronchoconstriction or asthma may occur
o If so, take ACE II to prevent side effects
Calcium channel blocking agents
Calcium effects is more on electrolyes= has effects on the heart rate itself
CALCIUM CHANNEL BLOCKER PICTURE
End name: Dipine- vasodilation
Verap and diltiazem (SVT supraventricular tachy- >150)
o For emergency cases
Norvasc (amlodipine)- most common
Cardizem (diltiazem)
Plendil (felodipine)
32| P h a r m a c o l o g y

Procardia (nifedepine)
Calan (verapamil)- may cause gingival hyperplasia
These are also pregnancy category C teratogenic
Useful in hypertension as it dilates peripheral arteries and decrease peripheral vascular resistance by
relaxing vascular smooth muscle
Monotherapy or in combination
Tolerated well even with renal failure
28 | P h a r m a c o l o g y
Vasodilators
Limited effect when used alone
Cannot be used alone or as monotherapy
o If vasodilator only, there is rebound effect
Vasodilating action that lowers BP also stimulates SNS. This in turn, triggers reflexive compensatory
mechanisms that raise BP
Corlopam (felodapam)- IV infusion, in hypertensive emergencies avoid in patients with allergies to
sulfites
Apresoline (hydralazine)- IV, IM, or PO
o Can cause orthostatic hypotension
o Most common vasodilator
In 2 minutes decrease in BP; 30 minutes effects will subside
Rogaine (minoxidil)- PO or topical
Hypertensive emergencies
> 180 systolic
> 120 diastolic
Is defined as having end organ damage or diastolic BP of 120 Hg or higher
With oral medications, use captopril 25-50 mg PO every 1-2 hours or clonidine 0.2 initially then 0.1 mg
every
hour until diastolic blood pressure falls below 100 Hg or if 0.7 mg has been given
Can cause stroke, MI, kidney damage
Nitroglycerine- antianginal medication
o Taken as a vasodilator
o Chest pains
o Sublingual- will alleviate pain
o In some consequences may decrease BP
o Emergency medication for angina
o Only lasts for 30 days- expiration
MAP- main arteria pressure
o Systolic (diastolic x 2) / 3
o Perfusion- < 60 low perfusion
Multiorgan damage
o > 120- hypertensive emergency
Herbals that affect the BP Ephedra (ma huang)- medicinal preparation from the plant Ephedra sinica
Ginseng- root of plants in the genus Panax Yohimbe (for erectile dysfunction)- dietary supplement
made from the bark of an African evergreen tree
Caffeine- most common
Other information
Viagra- vasodilation of pulmonary arteries
o Do not take beta blockers= lead to hypotension
33| P h a r m a c o l o g y

Check VS before giving


o < 60 bpm do not give beta blockers
Cut off of BP- 100 systolic; antihypertensives should not be given
Diuretics
Indicated for the treatment of edematous and non edematous conditions
o pulmonary edema
Only medication that decreases blood volume
May be useful in preventing renal failure by sustaining urine flow
A minimum of daily urine output approximately 400 ml is required to remove normal amount of
metabolic end products
o Polyuria- > 4L
o Oliguria- < 400-500 mL
o Anuria- < 100 mL; end stage renal disease
29 | P h a r m a c o l o g y
Acts on the kidneys to decrease absorption of sodium, chloride, water, and other substances such as
calcium
o Actions is in the nephrons
Major subclasses- thiazides, osmotic diuretic, loop, and potassium-sparing diuretics
Each act at different sites of the nephron
o Thiazides- affect is in distal tubule
o Loop- affects the ascending loop of Henle
o Potassium- sparing; quick diuretic (collecting; already urine form and distal tubule)
Used to manage
o Edema and ascites
o Management of heart failure- decrease fluids and water
o hypertension
Thiazide diuretics
End name- Thiazide
Chemically related to sulfonamides so caution to patients with sulfa allergies
Used in long term management of heart failure and hypertension
Affect distal convoluted tubule
Effectiveness decreases as the GFR decreases
o GFR output should be 30 mL/ hour
o If less than 30 mL/hour it becomes ineffective
o As rising creatinine is noted, alternatives should be used such as loop diuretic
Contraicdicated to px with renal damage or failure
Can cause hypokalemia= leads to heart attack
o Potassium most sensitive electrolyte for the heart
Diuril (cholorothiazide)
Hygroton (chlorthalidone)
Hydrodiuril (hydrochlorthiazide)
Loop diuretics
Inhibit sodium and chloride reabsorption in the ascending limb of the loop of Henle
Potent diuretic
Most common diuretic
Need to restrict dietary sodium when taking these medications
Lasix (Furosemide)- prototype and most common; if not hypokalemic
o But if hypokalemic- potassium-sparing diuretic
34| P h a r m a c o l o g y

o Given with px with pulmonary edema


o Potent diuretic
o Only work with px without kidney problems
o Excessive fluids in the lungs (crackles etc.)
Bumex (bumetamide)- more potent than Lasix
Oral- more on maintenance; treatment for hypertension; IV form- diuresis
Ototoxic
Max dose 120 mg/day
Given slow, because If rapid= deafness
Must monitor potassium levels, intake and output, and also weight
o 80% of potassium is excreted in the urine= lead to hypokalemia
Potassium-sparing diuretics
Act at distal tubule to decrease reabsorption of sodium and potassium excretion
Spironolactone- prototype, blocks sodium retaining effects of aldosterone
o Conserve potassium; excrete sodium
o Where sodium goes water follows
Weak diuretic when used alone, often used in combination
Contraindicated in renal failure
Osmotic diuretics
Produce rapid diuresis by increasing the solute load of the glomerular filtrate
Water is pulled into the intravascular space and excreted via kidneys
Useful in managing oliguria and anuria
Can prevent acute renal failure during prolonged surgery, trauma, or during chemotherapy
Helps reduce increased ICP, reduction of intraocular pressure before certain ophthalmic surgery and
for urinary excretion of toxic substances
30 | P h a r m a c o l o g y
o Stroke= increased intracranial pressure
Effect pull fluids IV space
Crystal form
Not used for oliguria and anuria
Osmitrol (mannitol)- most common osmotic diuretic
o For increased ICP
o Also for blood transfusion reactions
o Should be given fast drip- because it is a solute load; 50, 75, 100 mL Q4 large gauges of IV
cannula; because it precipitates
o Put inwarm water to dissolve
Ismotic (isosorbide), and osmoglyn (glycerin)
PHARMACOLOGY REVIEWER FOR FINALS
Anticoagulants
Drugs that affect coagulation
Thrombosis may occur in both arteries and veins
Arterial thrombi cause disease by obstructing blood flow which can result in tissue ischemia or death
o Coronary, cerebral, carotid, and the aorta
o Smoking can cause peripheral arterial disease
Venous thrombosis is associated with venous stasis. A venous thrombus is less cohesive than an
arterial embolus so no venous emboli is more prevalent
o Common to people with sedentary lifestyle, no exercise, people with varicose veins, and common
with long flights
35| P h a r m a c o l o g y

Hemostasis
Hemostasis is the prevention or stoppage of blood loss from an injured blood vessel. Process
Involves vasoconstriction, formation of a platelet plug (platelet aggregation), activation of clotting
factors and growth of fibrous tissue into the blood clot (fibrin) making it more stable
o Localized vasoconstriction is usually in the capillaries
o When blood is exposed to air it will start to clot
Clot lysis
When clot is being formed, plasminogen is bound to fibrin and becomes a component of a clot
After tear in blood vessel is repaired, plasminogen is activated by plasminogen activator to produce
plasmin. Plasmin (enzyme) breaks down the fibrin mesh and dissolves the clot
o Fibrin mesh is in the capillary
o Fibrin is broken down then is reabsorbed in the body
Blood coagulation factors
I- fibrinogen
II- prothrombin
III- thromboplastin
IV- calcium
V- labile factor
VI-
VII- proconvertin or stable factor
VIII- antihemophilic factor
IX- Christmas factor
X- stuart factor
XI- plasma thromboplastin antecedent
XII- Hageman factor
XIII- fibrin-stabilizing factor
Clotting factors can be found in the plasma
For patients with bleeding- give fresh frozen plasma
Anticoagulants
Given to prevent formation of new clots and extension of clots already present; does not dissolve clots
that are already present
Prototype: Heparin
Heparin may be used during pregnancy
Used during hemodialysis
Heparin is used with IU
31 | P h a r m a c o l o g y
Weight is based dosing per international nomogram
Differences
Heparin
Warfarin
Route
IV or SQ
Oral
Indications
Acute/ immediate cases
Long-term cases (2 weeks
); maintenance
Antidote
36| P h a r m a c o l o g y

Protamine sulfate
Vitamin K
Monitoring
aPTT
PT
Heparin
Usually given a loading dose then calculated dosage per 24h; e.g. 15-25 units/kg/hr for IV dosing
Can be given IV, SQ, or continuous infusion; cannot be given oral or IM
Contraindication of heparin: if patient has presentation of bleeding e.g. GI bleeding
To know therapeutic effect of heparin- monitor activated partial thromboplastin time (aPTT)
o Should be 1.5-2.5 control value—normal value is 35 seconds
HIT- Heparin induced thrombocytopenia
o Can cause low platelet count
o If patient has low platelet count check if it can be given
o Normal platelet count: 150,000-400,000
o < 100,000 is considered as low platelet count
Antidote: Protamine sulfate
Combines with antithrombin III which will inactivate clotting factors IX, X, XI, and XII, inhibits
conversion of prothrombin to thrombin and prevents thrombus formation. Further affects coagulation
by preventing conversion of fibrinogen to fibrin, inhibiting factors V, VIII, XIII, and platelet aggregation
INR checked Q6
Low molecular weight heparins
Given subcutaneously in abdomen and do not require close blood monitoring of blood coagulation
tests
Still should follow platelet counts
Fragmin (dalteparin)
Lovenox (enoxaparin); clexane- name used in the Philippines
o Lesser chance of heparin-induced thrombocytopenia
Coumadin (Warfarin)
Most commonly used as oral anticoagulant
Drug acts in the liver to prevent synthesis of vitamin K- dependent clotting factors (II, VI, IX, X). Acts
as competitive antagonist to hepatic use of vitamin K
Anticoagulant choice for long-term maintenance therapy
Can be combined with heparin
International normalized ratio (INR) is based on prothrombin time
o Normal baseline or control of PT is 12 seconds; therapeutic value is 1.5 times the control or 18
seconds
Therapeutic effect will occur in two weeks
o Check using prothrombin time (PT)
INR should be checked weekly
o INR- normal value: 1
o INR has eliminated disparities in different labs
Dosage reduction in patients with biliary tract disorders, liver disease, malabsorption syndrome, and
hyperthyroidism.
o Conditions increase anticoagulant drug effects by reducing absorption of vitamin K or decreasing
hepatic synthesis of clotting factors
Has multiple drug interactions
Counteract with vitamin K
37| P h a r m a c o l o g y

When to combine heparin and warfarin


Give heparin until stable or up until patient is still admitted in the hospital; then at discharge give
maintenance of warfarin= once it already has effect discontinue heparin
32 | P h a r m a c o l o g y
Other anticoagulants
Orgaran (danaparoid)- low molecular weight, heparin-like drug. Given subcutaneously.
o Used in management of hip surgery, ischemic stroke or in those who cannot take heparin.
o Does not contain heparin
Refludan (lepirudin)- used as heparin substitutes
Arixtra (fondaparinux)- binds to clot bound factor Xa, inhibits thrombin productions
o given subcutaneously
Argatroban
Angiomax (bivalirudin)
Antiplatelet aggregate drugs
Arterial thrombi are composed of primary platelets
Antiplatelet drugs act by inhibiting platelet activation, adhesion, aggregation, or procoagulant activity
Include drugs that block platelet receptors for thromboxane, adenosine diphosphate, glycoprotein
IIIa/IIb, and phosphodiesterase inhibitors
Prevents clumping of platelets
Blood thinners
Weaker compared to warfarin and heparin
Used as maintenance
Mostly oral tablets
Thromboxane A2 inhibitors
Work by inhibiting synthesis of prostaglandins. TA inhibitors work by acetylating cyclooxygenase, the
enzyme in platelets that synthesizes thromboxane A2 (which causes platelet aggregation)
ASA- acetyl salicylic acid; aspirin, is an example. It affects the platelets during its life
o Total life of a platelet- 9-10 days
NSAIDs are not so useful as action wears off as drug wears off
Adenosine diphosphate receptor antagonist
Ticid (ticlodipine)
Inhibit platelet aggregation by preventing ADP-induced binding between platelets and fibrinogens.
This reaction inhibits platelet aggregation irreversibly and persist for the lifespan of the platelet
Indicated for TIA- transient ischemic attack
o Blockage in the carotid artery
o A sign of stroke for the next 5 years
Adverse effects: neutropenia, diarrhea, and skin rashes
o Neutropenia- 65-80% low neutrophils
Even used for post angioplasty
Taken for 1 year only- due to price
Plavix (clopidogrel)
o Has fewer side effects than ASA or ticlid
o Indicated for patients with atherosclerosis for reduction of myocardial infarction, stroke, and vascular
death
o Does not need reduction in those with renal problems
Glycoprotein IIb/IIIa receptor antagonist
38| P h a r m a c o l o g y

Reopro (abciximab)- is a monoclonal antibody that prevents binding of fibrinogen, von willerbrand
factor, and other molecules to GP IIb/IIIa receptors on activated platelets. This action inhibits platelet
aggregation
o Given during angioplasty
Used with percutaneous transluminal coronary angioplasty (PTCA) or removal of atherosclerotic
plaque to prevent rethrombosis
Used with aspirin and heparin and is contraindicated in clients who have recently received oral
anticoagulants or IV dextran
Other contraindications include active bleeding, thrombocytopenia, history of stroke, surgery or
trauma within the past 6 weeks, uncontrolled hypertension, or hypersensitivity
Other glycoprotein IIb/IIIa receptor antagonist
Integrilin (Eptifibatide)- only given for 24 hours after angioplasty
o Potent antiplatelet aggregate
Aggrasat (tirofiban)
Similar mechanism to the action of reopro
33 | P h a r m a c o l o g y
Indicated for acute coronary syndrome who are managed medically or by angioplasty or atherectomy
Contraindications include
o Recent bleeding
o History of thrombocytopenia
o History of stroke within 30 days
o Major surgery or severe trauma within the past month
o Severe hypertension
o History of intracranial hemorrhage
o Neoplasm
o AV malformation
o Aneurysm
o Platelet count < 100,000
o Creatinine greater than 2 mg/dl
Phosphodiesterase inhibitor
Pletal (cilostazol)- increases cAMP which then inhibits platelet aggregation and produces vasodilation.
Drug reversibly inhibits platelet aggregation
Indicated for intermittent claudication
Indicated for peripheral vascular disease
o Buerger’s disease
o Reynaud’s syndrome
Contraindicated for patients with heart failure
Most common side effect is diarrhea and headache
Miscellanous
Persantine (dipyridamole)- inhibits platelets aggregation but mechanism is unclear
Used for prevention of thromboembolism after cardiac valve replacement and is given with Coumadin
Thrombolytics
Given to dissolve thrombi
Stimulate conversion of plasminogen to plasmin, an enzyme that breaks down fibrin (the framework of
a thrombus)
Used in severe thromboembolic diseases such as myocardial infarction, PE and ileofemoral
thrombosis
Goal is to re-establish blood flow and prevent tissue damage
39| P h a r m a c o l o g y

Also used to dissolve clots in arterial or venous cannulas or catheters


Must obtain baseline INR, aPTT, platelet count, and fibrinogen
Will monitor tabs 2-3 hours after thrombolytic therapy is instituted to determine efficacy
Activase (alteplase), Retavase (reteplase), and TNKase (tenecteplase) are tissue plasminogen
activators
o Mainly used in acute MI to dissolve clots
Eminase (anistreplase), Streptase (streptokinase), and Abbokinase (urokinase) are enzymes that
break down fibrin
o Used to lyse coronary clots in acute MI
o Streptokinase- problem allergy; byproduct of streptococcus
o Can cause anarrhytmia
Xigris (drotrecogin alfa) is a recombinant version of human activate protein C
o Approved for use in sepsis
o Sepsis causes inappropriate blood clot formation and may lead to DIC
o DIC- last part of sepsis
o Blood clots due to consumption of platelets
o experimental medication
Strongest type of anticoagulants
Used to dissolve clots
Last option
Heart attack- 6 hours timeframe for thrombolytics
Stroke- 4 hours
Drugs used to control bleeding
Amicar (aminocaproic acid) and Cyklokapron (tranaxemic acid) are used to stop bleeding caused by
thrombolytic agents
o Tranexamic acid- used for trauma
Trasylol (aprotinin) indicated in patients undergoing CABG
o Inhibits breakdown of blood clotting factors
34 | P h a r m a c o l o g y
Drugs affecting the cardiovascular system
ANTILIPIDS
Important Reminders
Risk factors for Thromboembolism
1. Obesity
2. Myocardial infarction
3. Atrial Fibrillation
4. Prosthetic heart valves
5. Arteriosclerotic heart disease
6. OCP or HRT
7. History of deep vein thrombosis or pulmonary embolism
8. Cigarette smoking
9. Immobility
Overview of Cholesterol Panel
Total Cholesterol
Desirable – Less than 200
Borderline high – 200 - 239
High – 240 or greater
Low-density lipoprotein
40| P h a r m a c o l o g y

Desired – <100
Above optimal – 100 – 139
Borderline high – 130 – 159
High – 160 – 189
Very high – 190
High-density lipoprotein
High - >60
Low - <40
Triglycerides
Normal – Less than 150 – 199
High – 200 to 499
Very high – 500 or above
DYSLIPIDEMIA
Associated with atherosclerosis and numerous pathophysiologic effects
Elevated total cholesterol, high HDL and low HDL are all risk factors for CAD
Triglycerides indicated excessive caloric intake, excessive proteins and carbohydrates are converted
to TG and obesity
Atherosclerosis is narrowing of arteries due to plaques
CAD most risky at 40 due to CHO / plaque starts from infancy – Accumulation, checked through
angiogram
Hypothyroidism – decrease in T3 and T4. Slow speak and slow thinking, edema
Contributors to Dyslipidemia
Hypothyroidism
Diabetes mellitus
Alcoholism
Obesity, beta blockers, oral estrogen, glucocorticoids, sertraline, thiazide, diuretics, protease inhibitors
High dietary intake also increases the conversion of VLDL to LDL cholesterol, and high dietary intake
of TG and saturated fat decreases the activity of LDL receptors and increases synthesis of
cholesterol.
Types of Lipoproteins
35 | P h a r m a c o l o g y
LDL – Unfavorable type. Transports 75% of serum cholesterol to peripheral tissues and the liver. High
levels are atherogenic
VLDL – Contains 75% TG and 25% cholesterol. Transports endogenous TG to fat and muscle cells
HDL – Favorable type. This LP transports cholesterol back to the liver to catabolism and excretion
Initial Management
Treat conditions that contribute to elevated lipids (DM, Hypothyroidism)
Start low fat diet
Increase intake of fiber – lowers LDL
Cholestetol lowering margarines
Weight Reduction
Exercise – Increase HDL
Smoking cessation
HRT (Hormonal Replacement Therapy)
Drug Therapy
Based on the type of dyslipidemia and its severity
Classes of agents include: HMB-CoA reductase inhibitors or “Statins”, fibrates, bile acid sequestrants
and niacin in different forms
41| P h a r m a c o l o g y

Lovaza
Zetia
HMG-CoA reductase inhibitors or “Statins”
Inhibits an enzyme (Hydroxymethylglutaryl-coenzyme A reductase) required for hepatic synthesis of
cholesterol
Decrease serum cholesterol, LDL, VLDL and TG
Reach maximal effects within about 6 weeks
Drugs also reduce C reactive protein associated with inflammation and development of coronary
artery disease
Undergo extensive first pass metabolism
Metabolism occurs in liver
Adverse effects include: HA, diarrhea, rashes, headaches, constipation, hepatotoxicity and myopathy
o Reason to stop statins is because of myopathy= rhabdomyolysis (serious syndrome due to a direct
or indirect muscle injury. It results from the death of muscle fibers and release of their contents into
the bloodstream.)= due to secretion of creatinine kinase to the bloodstream= stocked in kidneys=
renal failure
Should obtain baseline LFTs and then at 6 and 12 weeks after starting then every 6 months
If serum aminotransferases increase to more than 3x normal, should be reduced or DCed
Do not take with grapefruit juice
Pregnancy category X
Examples: Lipitor (Atorvastatin), Pravachol (pravastatin), Zocor (Simvastatin), Lescol (Fluvastatin)
Rosuvastatin- Another common statin
Bile Acid Sequestrants
Bind bile acids in the intestinal lumen. This causes the bile acids to be excreted in the feces and
prevents their being recirculates to the liver. Thus, the liver will use cholesterol to produce bile acids
thus decreasing serum levels
Especially lower LDL
Examples: Questran (Cholestyramine)
and Welchol (Colesevelam)
Often used with patients already on a statin
Long term use can affect absorption of folate and vitamins A, D, E, K
If feces turns white= pancreatitis or liver problem
FIBRATES
Tricor (Fenofibrate)
Lopid (Gemfibrozil)
These drugs increase oxidation of fatty acids in liver and muscle tissue thus decreasing hepatic
production of TG, VLDL, and increase HDL
Most effective drugs for reducing TG
can cause hepatotoxicity
Main side effects include: diarrhea, GI discomfort, cause gallstones, interact with Coumadin
NIACIN (Nicotinic Acid)
Decreases both cholesterol and triglycerides
36 | P h a r m a c o l o g y
Bottom line – decreases hepatic synthesis of TG and secretion of VLDL (which leads to decreased
production of LDL)
Need high doses for efficacy
Side effects: Flushing, pruritus, gastric irritation. May cause hyperglycemia, hyperuricemia, elevated
hepatic aminotransferase enzymes and hepatitis
42| P h a r m a c o l o g y

Can reduce flushing by starting with low doses, taking dose with meals, and taking ASA 325mg thirty
minutes before taking dose
More effective in preventing heart diseases when used in combination with another dyslipidemic such
as a bile acid sequestrant or fibrate.
Drugs affecting the respiratory system
Key terms
Ventilation
Perfusion
Diffusion
Pulmonary circulation
Surfactant
Pneumocytes
drugs for asthma and other bronchoconstrictive disorders
Asthma- inflammation, hyperreactivity, and bronchoconstriction
Gastroesphageal reflux disease (GERD) may cause microaspiration/resultant nighttime cough
___- medications can also exacerbate GERD
Asthma
May be triggered by __, __, and allergies
Can develop at any age
Seen more often in children who are exposed to allergens during infancy
Bronchoconstriction
Inflammation
Mucosal edema
Excessive mucous
Pathophysiology of asthma
Mast cells
Chemical mediators such as histamine, prostaglandins, acetylcholine, cGMP, interleukins, and
leukotrienes are released when triggered. Mobilization of eosinophils. All cause movement of fluid and
proteins into tissues.
Bronchoconstrictive substances antagonized by cAMP
Chronic obstructive pulmonary disease (COPD)
Combination of chronic bronchitis and emphysema
Bronchoconstriction and inflammation are more contant, less reversibility
Anatomic and physiologic changes occur over the years
Leads to increasing dyspnea and activity intolerance
Drug therapy
Bronchodilators
Anti-inflammatories
Bronchodilators
Adrenergics- stimulate beta II receptors in smooth muscles of bronchi and bronchioles
Receptors stimulates cAMP= bronchodilation
Cardiac stimulation is an adverse effect of these medications
Bronchodilators- adrenergics
Cautious use in hypertension and cardiac disease
Selective beta II agonists by inhalation are drugs of choice
Epinephrine subcutaneous in acute bronchoconstriction
Short acting bronchodilators
Treatment of first choice to relive acute asthma
43| P h a r m a c o l o g y

37 | P h a r m a c o l o g y
Aerosol or nebulization
May be given by MDI
Overuse will diminish their bronchodilating effects= tolerance
Proventil (albuterol)
Xopenex (levalbuterol)
Other bronchodilators
Foradil (formoterol) and serevent (salmeterol) are long acting beta II adrenergic agonists used only for
prophylaxis
Serevent (salemeterol)- used in deteriorating asthma can be life-threatening
Alupent (metaprotenerol)- intermediate acting. Useful in exercise induced asthma, treatment for acute
bronchospasm
Brethine (terbutaline)- selective beta II adrenergic agonist that is a long-acting bronchodilator
When give subcutaneous, loses selectivity
Also used to decrease premature uterine contractions during pregnancy
Xanthines
Theophylline
Mechanism of action is unclear
Bronchodilate, inhibit pulmonary edema, increase action of cilia, strengthen diaphragmatic
contractions, over-all anti-inflammatory action
Increases CO, causes peripheral vasodilation, mild diuresis, and stimulates SNS
Contraindicated in acute gastritis and PUD
Second line
Narrow therapeutic window- therapeutic range is 5-15 mcg/mLh
Multiple drug interactions
Anticholinergics
Block the action of acetylcholine in bronchial smooth muscles when given by inhalation
Action reduces intracellular guanosine monophosphate (GMP) which is a bronchoconstrictive
substance
Atrovent (ipratropium)- caution in BPH, narrow-angle glaucoma
Spiriva (tiotropium)
Anti-inflammatory agents
Suppress inflammation by inhibiting movement of fluid and protein into tissues; migration and function
of neutrophils and eosinophils, synthesis of histamine in mast cells, and production if proinflammatory
substances
Benefits: decrease mucous secretions, decreases edema, and reduced reactivity
STEROIDS
Corticosteroids
Second action is to increase the number and sensitivity of beta 2 adrenergic receptors
Can be given PO or IV
Pulmonary function usually improves within 6-8 hours
Continue drugs for 7-10 days
Steroids
Fewer long term side effects if inhaled
End –stage COPD may become steroid dependent
In asthma, systemic steroids generally are used only temporarily
Taper high dose oral steroids to avoid hypothalamic-pituitary axis suppression
- For inhalation:
44| P h a r m a c o l o g y

o Beclovent (beclomethasone)
o Pulmicor (budesonide)
o Aerobid (flunisolide)
o Flovent (fluticasone)
o Azmacort (triamcinolone)
Most inhaled steroids are being reformulated with HFA
HFA – hydrofluoroalkane
Systemic use: prednisone, methylprednisolone, and hydrocortisone
In acute, severe asthma – a systemic corticosteroid may be indicated when inhaled beta 2 agonists
are ineffective
38 | P h a r m a c o l o g y
MODIFIERS
Leukotriene Modifiers
Leukotrienes are strong chemical mediators of bronchoconstriction and inflammation
Increase mucous secretion and mucosal edema
Formed by the lipooxygenase pathway of arachidonic acid metabolism in response to cellular injury
Are released more slowly than histamine
Leukotriene Modifier Drugs
Developed to counteract the effects of leukotriene
Indicated for long term treatment of asthma in adults and children
Prevent attack induced by some allergens, exercise, cold air, hyperventilation, irritants and
ASA/NSAIDs
Not useful in acute attacks
Leukotrience Modifiers
Injured cell
Arachidonic acid
XXXXXXXXX
Lipooxygenase
Leukotrienes
XXXXXXX
Bronchi, WBCs
Bronchoconstriction
Leukotriene Modifier Drugs
Singulair (montelukast) and Accolate (zafirlukast) are leukotriene receptor antagonists
Can be used in combination with bronchodilators and corticosteroids
Less effective than low doses of inhaled steroids
Should not be used during lactation
Can cause HA, nausea, diarrhea
Mast Cell Stabilizers
- Intal (cromolyn)
- Tilade (nedocromil)
Prevent release of bronchoconstrictive and inflammatory substances when mast cells are confronted
with allergens and other stimuli
Prophylaxis only
Inhalation, nebulizer or MDI, nasal spray as well
MDI – Metered-dose inhaler
ANTI-HISTAMINE AND ALLERGIES
Antihistamines and Allergic Disorders
45| P h a r m a c o l o g y

Histamine is the first chemical mediator released in immune and inflammatory responses
Concentrated in skin, mucosal, surfaces of eyes, nose, lungs, CNS and GI tract
Located in mast cells and basophils
Interacts with histamine receptors on target organs called H1 and H2
Antihistamines
H1 receptors are located mainly on smooth muscle cells in blood vessels and the respiratory and GI
tracts
H1 binding causes: pruritus, flushing, increased mucous production, increased permeability of veins –
edema, contraction of smooth muscle in bronchi>>bronchoconstriction and cough
With H2 receptor stimulation, main effects are increased secretion of gastric acid and pepsin,
decreased immunologic and proinflammatory reactions, increased rate and force of myocardial
contraction
Allergic reactions
These are the exaggerated response by the immune system that produce tissue injury and possible
serious disease
Allergic reactions may result from specific antibodies, sensitized T lymphocytes, or both, formed
during exposure to an antigen
TYPES OF RESPONSES TO CELL-MEDIATED INVASION
Type 1: Immediate hypersensitivity (IgE induced) response triggered by the interaction of antigen with
antigen-specific IgE bound on mast cells
Anaphylaxis is an example
39 | P h a r m a c o l o g y
Does not occur on first exposure to an antigen
Can develop profound vasodilation resulting in hypotension, laryngeal edema, bronchoconstriction
Type II: (IgM or IgM) immunoglobulin mediated which generate direct damage to cell surfaces.
Example: BT reactions, hemolytic disease of newborns, hypersensitivity reactions to drugs such as
heparin or penicillin.
Type III: IgM or IgM medicated reaction characterized by formation of antigen-antibody complexes
that induce inflammatory reaction in tissues.
Prototype: Serum sickness
Immune response can occur following antitoxin administration or sulfa drugs
Type IV hypersensitivity
Delayed hypersensitivity
Cell mediated response where sensitized T lymphocytes react with an antigen to cause an
inflammation, release of lymphokines, direct cytotoxicity or both
Classic examples: tuberculin test, contact dermatitis and some graft rejections
Allergic Rhinitis
IgE mediated
Inflammation of nasal mucosa caused by a hypersensitivity reaction to inhaled allergies
Presents with itching of throat, eyes and ears
Seasonal and perennial
Can lead to chronic fatigue, difficulty sleeping, sinus infections, postnasal drip, cough and headache
Intranasal Drugs for Allergic Rhinitis
Atrovent nasal spray
Beconase (beclomethasone)
Rhinocort (budesonide)
Flonase (fluticasone)
Nasonex (mometasone)
46| P h a r m a c o l o g y

Nasalcrom (a mast cell stabilizer)


SKIN ALLERGIES
Allergic Contact Dermatitis
Type IV (hypersensitivty reaction)
Usually occurs >24 hour after reexposure
o Example: Poison Ivy
Other reactions
Allergic food reaction: result from ingestion of a protein
Most common food allergy is shellfish, others include milk, eggs, peanuts
Allergic drug reaction: unpredictable, may occur 7-10 days after initial exposure
Pseudoallergic drug reactions: resemble immune response but do not produce antibodies, i.e.
anaphylactoid
ANTIHISTAMINES
Inhibit smooth muscle constriction in blood vessels and respiratory tract and GI tract
decrease capillary permeability
decrease salivation and tear formation
act by binding with the histamine receptor
Indications for use:
allergic rhinitis
anaphylaxis
allergic conjunctivitis
drug allergies
transfusion of blood products
dermatologic conditions
nonallergic such as motion sickness, nausea, vomiting, and sleep
Precautions
40 | P h a r m a c o l o g y
Caution in pregnancy
Benign Prostatic H
bladder neck obstruction
narrow angle glaucoma
1ST GEN. H1 RECEPTORS ANTAGONISTS
Bind to central and peripheral receptors
Can cause CNS depression or stimulation
Have substantial cholinergic effects
EXAMPLES:
Chlor-Trimeton (chlorpheniramine)
Benadryl (dipenhydramine)
vistaril (hydroxyzine)
phenergan (promethazine)
2ND GEN. H1 RECEPTORS ANTAGONISTS
Selective or nonsedating
Does not cross BBB
EXAMPLES:
astelin (azelastine)
allegra (fexofenadine)
claritin (loratadine)
clarinex (desloratadine)
47| P h a r m a c o l o g y

zyrtec
Xyzal
NASAL DECONGESTANTS
Relieve nasal obstruction and discharge
Adrenergic
Rebound nasal swelling called "rhinitis medicamentosa"
EXAMPLES:
- Afrin
- Sudafed (pseudoephedrine)
- contraindicated in severe hypertension, CAD, narrow angle glaucoma, TCAs, MAOIs
ANTITUSSIVES
Suppress cough by depressing cough center in medulla or by increasing flow of saliva
For dry, hacking, nonproductive cough
not recommended for children and adolescents
EXAMPLES:
- codeine
- hydrocodone
- dextromethorphan
EXPECTORANTS
Liquefy respiratory secretions
- Guiafenesin
MUCOLYTICS
By inhalation to liquefy mucous
EXAMPLE:
- mucomyst (acetylcysteine)
- mucosolvan (ambroxol)
may be used in treating acetaminophen overdose
COLD REMEDIES
41 | P h a r m a c o l o g y
Contain antihistamine, decongestant, and an analgesic
Examples:
- chlorpheniramine
- Pseudoephedrine
- acetaminophen
- dextromethorphan
- guiafenesin
decongestants can cause stasis of secretions
Tamiflu can be used to limit spread of virus in respiratory tract
DRUGS AFFECTING THE RESPIRATORY SYTEM
KEY TERMS
Ventilation
Perfusion
Diffusion
Pulmonary Circulation
Surfactant
Pneumocytes
Drugs for Asthma and other Bronchoconstrictive Disorders
Asthma – Inflammation, hyperreactivity, and bronchoconstriction
48| P h a r m a c o l o g y

GERD – May cause microaspiration / resultant night time cough


Antiasthma – Medications can also excacerbate GERD
ASTHMA
May be triggered by Viruses, irritants, allergens.
Can develop at any age
Seen more often in children who are exposed to airway irritants during infancy
Bronchoconstriction
Inflammation
Mucosal edema
Excessive mucuos
Pathophysiology of Asthma
Mast cells
Chemical mediators such as Histamine, prostaglandins, acetylcholine, cGMP, interleukins,
leukotrienes are released when triggered. Mobilization of eosinophils. All cause movement of fluid and
proteins into tissues
Bronchoconstrictive substances antagonized by cAMP
Chronic Obstructive Pulmonary Disease (COPD)
Combination of chronic bronchitis and emphysema
Bronchoconstriction and inflammation are more contant, less reversibility
Anatomic and physiologic changes occur over the years
Leads to increasing dyspnea and activity intolerance
Drug therapy
Bronchodilators
Anti-inflammatories
Bronchodilators
Adrenergics- stimulate beta II receptors in smooth muscles of bronchi and bronchioles
Receptors stimulates cAMP= bronchodilation
Cardiac stimulation is an adverse effect of these medications
Bronchodilators- Adrenergics
Cautious use in hypertension and cardiac disease
Selective beta II agonists by inhalation are drugs of choice
Epinephrine subcutaneous in acute bronchoconstriction
Short Acting Bronchodilators
42 | P h a r m a c o l o g y
Treatment of first choice to relive acute asthma
Aerosol or nebulization
May be given by MDI
Overuse will diminish their bronchodilating effects= tolerance
Proventil (albuterol)
Xopenex (levalbuterol)
Other bronchodilators
Foradil (formoterol) and serevent (salmeterol) are long acting beta II adrenergic agonists used only for
prophylaxis
Serevent (salemeterol)- used in deteriorating asthma can be life-threatening
Alupent (metaprotenerol)- intermediate acting. Useful in exercise induced asthma, treatment for acute
bronchospasm
Brethine (terbutaline)- selective beta II adrenergic agonist that is a long-acting bronchodilator
When give subcutaneous, loses selectivity
49| P h a r m a c o l o g y

Also used to decrease premature uterine contractions during pregnancy


Xanthines
Theophylline
Mechanism of action is unclear
Bronchodilate, inhibit pulmonary edema, increase action of cilia, strengthen diaphragmatic
contractions, over-all anti-inflammatory action
Increases CO, causes peripheral vasodilation, mild diuresis, and stimulates SNS
Contraindicated in acute gastritis and PUD
Second line
Narrow therapeutic window- therapeutic range is 5-15 mcg/mLh
Multiple drug interactions
Anticholinergics
Block the action of acetylcholine in bronchial smooth muscles when given by inhalation
Action reduces intracellular guanosine monophosphate (GMP) which is a bronchoconstrictive
substance
Atrovent (ipratropium)- caution in BPH, narrow-angle glaucoma
Spiriva (tiotropium)
Anti-inflammatory agents
Suppress inflammation by inhibiting movement of fluid and protein into tissues; migration and function
of neutrophils and eosinophils, synthesis of histamine in mast cells, and production if proinflammatory
substances
Benefits: decrease mucous secretions, decreases edema, and reduced reactivity
DRUGS ACTING ON THE ENDOCRINE
Control of Hormone Release
- Blood levels of hormones
- Are controlled by negative feedback systems
- Vary only within a narrow desirable range
- Hormones are synthesized and released in response to:
1. Humoral stimuli - Blood
2. Neural Stimuli
3. Hormonal Stimuli
PITUITARY AGENTS
Desmopressin (DDAVP)
- Classification: Antidiuretic, hemostatic
- Indications: Diabetes Insipidus
- Available forms:
- Intranasal Spray
- IV
- Tablet
- Drug of choice for Diabetes Insipidus
43 | P h a r m a c o l o g y
Diabetes Insipidus
- The problem is there is a decrease in Antidiuretic Hormone (ADH) causing Polyuria
- Urine output is increased to 7L-8L/Hour
- Patients die due to Dehydration
- To confirm diabetes insipidus an urinalysis test is needed
- Gravity <1.05 is not good
Action of Desmopressin
50| P h a r m a c o l o g y

- Increase the flow of Adenosine Monophosphate and water through the kidneys, promoting
reabsorption of water and producing concentrated urine.
Nursing Process in Desmopressin
Assessment: Monitor effectiveness of therapy, be alert for Hypertension.
Nursing Diagnosis: Fluid Volume Excess
Planning and implementation:
1. Ensure nasal mucosa is intact
2. In cases of Rhinorrhea, report it to the doctor
3. Adjust the dose in coordination with the doctor In relation to the urine output of the patient
Patient Teachings:
- Instruct patient to clear nasal passages
- Tell patient to report nasal congestion, allergic rhinitis, and respiratory tract infection
- Rotate injection when using SQ form
- Warn patient to drink just enough water to satisfy the thirst
Vasopressin (Pitressin)
- Classification: Antidiuretic Hormone
- Indication:
- Diabetes Insipidus
- Hypotension
- Pulseless arrest, Asystole
- Available forms: IV
- Last option for Inotropes
- Can be a substitute for epinephrine but only once
- More on hypotension and Cardiac arrest
- Adverse Reaction:
- Arrhythmia
- Myocardial ischemia
- Bronchoconstriction, Anaphylaxis
Nursing Process in Vasopressin
Assessment: Monitor the urine specific gravity, Watch out for hypertension, Closely monitor for signs
of water intoxication.
Nursing Diagnosis: Risk for Fluid Volume Deficit
Planning and Implementation:
1. Use minimum effective dose
2. In cardiac emergency, follow the guidelines in ACLs
3. Don’t confuse Vasopressin with Desmopressin
ADRENOCORTICAL AGENTS
Hydrocortisone (Solu Cortef)
44 | P h a r m a c o l o g y
Classification: Glucorticoids (steroids), Anti-inflammatory, immunosuppressant
Indication: Severe Inflammation, Septic shock
Available forms: Tablet, IV
Contraindication: GI Ulcers, Renal disease, Osteoperosis, DM
Adverse Effects: Seizures, Arrythmia, Pancreatitis, Peptic Ulceration
- Therapy must not exceed 21 days
- Hypokalemia may occur
Action of Hydrocortisone
- Not clearly defined
51| P h a r m a c o l o g y

- May stabilize lysosomal membranes, suppress immune response, and stimulate bone marrow
Nursing Process in Hydrocortisone
Assessment: Monitor weight, Blood pressure, and Electrolytes level
Nursing Diagnosis: Risk for Infection
Planning and Implementation
1. Give oral dose with food
2. Give potassium supplements
3. Therapy must not exceed the 21 days
Patient Teachings
- Tell the patient not to abruptly stop taking hydrocortisone without the doctors content
- Warn patient about easy bruising
- Warn patient about Cushingcoid symptoms
Methylprednisolone (Medrol)
Classification: Glucocorticoids, Anti- Inflammatory, Immunosuppresant
Indication: Severe Inflammation, shock, multiple sclerosis, severe lupus nephritis
Available Forms: Tablet, IV
- The rest of the literatures are the same as hydrocortisone
ANTIDIABETIC AGENTS
Pancreas
- Triangular gland behind the stomach
- Has both exocrine and endocrine cells
- Acinar cells (Exocrine) produce an enzyme-rich juice for digestion
- Pancreatic Islets (Islets of Langerhans) contain endocrine cells
- Alpha cells produce glucagon (a hyperglycemic hormone)
- Beta cells produce insulin (a hypoglycemic hormone)
Two Types of Diabetes
Type 1 Diabetes
- Disease case by lack of insulin
- Destroys beta cells therefore becoming autoimmune which causes a decrease in insulin and an
increase of blood sugar
- Oral medications cannot be used on Type 1 diabetes patients
Type 2 Diabetes
- Is a disease of insulin resistance by cells
- The most common type of diabetes
- Obesity
- Decrease in receptors of cells because fats covers the receptors
45 | P h a r m a c o l o g y
- Once the use of insulin starts it is used for lifetime
- Insulin is the last option
Insulin
Classification: Antidiabetic Agent
Indication: Moderate to severe diabetec Ketoacidosis, Control of Hyperglycemia, Hyperkalemia
Available Forms: IV, SQ
Common Types of Insulin
Rapid Acting
- Regular Insulin (Humulin R)
- 1 hour onset of action
- Appears to be clear
52| P h a r m a c o l o g y

- Is used in acute cases (IV)


- Insulin Lispro
- Insulin Aspart
Intermediate Acting
- Isophane Insulin (Humulin N)
- Appears to be cloudy
- Onset of action is 4 hours
- Insuline Zinc (Lente)
Long Acting
- Insulin Glargine (Lantus)
- Is used as maintenance
- Insulin Determir (Levemir)
- You can mix Humulin R and Humulin N for better Longevity and Effectivity
- Humulin R is aspirated first before Humulin N to not transfer cloudiness to clear
- Give air to Humulin N first
Contraindication: HYPOGLYCEMIA
Adverse Reactions:
- Hypoglycemia
- Urticaria
- Itching
- Anaphylaxis
- Lipodystrophy
- A problem in SQ fats where it appears to be depressed due to the repetition of using the same site
over and over or injecting cold insulin
- To prevent, rotate the injection site or change places. Insulin must be room temperature
Action of Insulin
- Increases glucose transport into the muscle and fat cell membranes to reduce glucose level
Nursing Process (Insulin)
Assessment:
46 | P h a r m a c o l o g y
- Assess patients glucose level before starting the therapy
- Monitor patient glycosylated haemoglobin regulary
- Monitor urine ketones when glucose level is elevated
Nursing Diagnosis: Risk for injury related to drug induced hypoglycaemia
Planning and Implementation:
- Dose is always expressed in units
- Don’t shake the insulin to prevent bubbles
- Rapid acting insulin should be given 15 minutes before meals
- Regular insulin can be mixed with NPH insulin in any proportions
- SQ route – 45 to 90 degree injection
- Don’t rub the site on injection
- Treat Hypoglycemia with rapid acting glucose or with Glucagon IV, IV Glucose
- If patient is awake you can give juice
Insulin Overdose – Antidote is Glucagon
Patient Teachings
- Tell patient that insulin relieves symptom but does not cure the disease
- Tell patient that glucose monitoring and urine ketone test are essential to dosage and success of
therapy
53| P h a r m a c o l o g y

- Hypoglycemia is hazardous – Brain damage


- Smoking decreases absorption of insulin
- Advise the patient to carry or wear medical identification at all times
ORAL HYPOGLYCEMIC AGENTS
Oral Hypoglycemic Drugs
1. Sulfonylurea drugs
2. Meglinitides
3. Biguanides
4. Alpha-Glucosidase Inhibitors
5. Thiazolidinediones
6. Dipeptidyl peptidase-4 (DPP-4) Inhibitors
Insulin Secretagogues
- Sulfonylurea drugs
- Meglinitides
Insulin Sensitizers
- Biguanides
- Thiazolidinediones
Others
- Alpha-Glucosidase Inhibotors
- Gastrointestinal hormones
INSULIN SECRETAGOGUES
- Are drugs which increase the amount of insulin secreted by the pancreas.
- Include: Sulfonylureas and Meglinitides
Mechanism of Action of SULFONYLUREAS
- Stimulate insulin release from functioning B cells of ATP-Sensitive K Channels which causes
depolarization and opening of voltage-dependent calcium channels, which causes an increase in
intracellular calcium in the beta cells, which stimulates insulin release.
47 | P h a r m a c o l o g y
Pharmacokinetics of Sulfonylureas
- Orally, well absorbed
- Reach peak concentration after 2-4 hours
- All are highly bound to plasma proteins
- Duration of action is variable
- Second generation has longer duration than first generation, has lesser side effects and more
effective
- Steven – Johnsons Syndrome a type III is related to sulfonylureas
- Metabolized in liver
- Excreted in urine (elderly and renal disease)
- Cross placents, stimulate fetal B-cells to release insulin – Fetal hypoglycaemia at birth
First Generation Sulfonylureas
Tolbutamide:
- Safe for diabetic patients or pts with renal impairment
Second Generation Sulfonylureas
Glipzide – Glyburide (Glibenclamide)
- More potent than first generation
- Have longer duration of action
- Less frequency of administration
- Have fewer adverse effects
54| P h a r m a c o l o g y

- Have fewer drug interaction


48 | P h a r m a c o l o g y
Unwanted Effects:
1. Hyperinsulinemia and Hypoglycemia
- Less in tolbutamide
- More in old age, hepatic and renal diseases
2. Weight gain due to increase in appetite
3. GIT upset
Contraindications
- Hepatic impairment or renal insufficiency
- Pregnancy and Lactation
- Type 1 Diabetes
MEGLINITIDES
- Repaglinide
- Are rapidly acting insulin secretagogues
- Stimulate insulin release from functioning B-cells via blocking ATP-sensitive K-channels resulting in
calcium influx and insulin exocytosis
- Orally, well absorbed
- Very fast onset of action, peak 1 hour
- Short duration of action (4 hours)
- Metabolized in liver and excreted in bile
- Taken just before each meal (3 times/day)
Uses of Meglinitides
- Type II Diabetes:
- Monotherapy or combined with metformin (Better than monotherapy)
- Specific use in patients allergic to sulfur or sulfonylureas
Adverse Effect of Meglinitides
- Hypoglycemia
- Weight gain
INSULIN SENSITIZERS
- Are drugs which increase the sensitivity of target organs to insulin
- Include: Biguanides and Thiazolidinediones (Glitazones)
Gastrointestinal drugs
Acid related pathophysiology
The stomach secretes Hydrochloric acid (HCl)- activates pepsinogen into the enzyme pepsin, which
then helps digestion by breaking the bonds linking amino acids, a process known as proteolysis.
Bicarbonate- acts to regulate pH in the small intestine. It is released from the pancreas in response to
the hormone secretin to neutralize the acidic chyme entering the duodenum from the stomach.
49 | P h a r m a c o l o g y
Pepsinogen- powerful and abundant protein digestive enzyme secreted by the gastric chief cells as a
proenzyme and then converted by gastric acid in the gastric lumen to the active enzyme pepsin.
Intrinsic factor- glycoprotein secreted by parietal (humans) or chief (rodents) cells of the gastric
mucosa. In humans, it has an important role in the absorption of vitamin B12 (cobalamin)
Mucus
Prostaglandins
Acid-controlling agents
Glands of the stomach
Cardiac
55| P h a r m a c o l o g y

Pyloric
Gastric
o the cells of the gastric glands are the largest in number and of primary importance when discussing
acid control
Cells of the gastric gland
Hydrochloric acid
Secreted by the parietal cells when stimulated by food
Maintains stomach at pH of 1 to 4
Secretion is also stimulated by:
o Large fatty meals
o Excessive amounts of alcohol
Parietal cells
Produce and secrete HCl
Primary site of action for many acid-controller drugs
Chief cells
Secrete pepsinogen, a proenzyme
Pepsinogen becomes pepsin when activated by exposure to acid
Pepsin breaks down proteins (proteolytic)
Mucoid cells
Mucus-secreting cells (surface epithelial cells)
Provide a protective mucus coat
Protects against self-digestion by HCl
Acid related diseases
Caused by imbalance of the three cells of the gastric gland and their secretions
Most common: hyperacidity
Clients reports symptoms of overproduction of HCl by the parietal cells as indigestion, sour stomach,
heartburn, and acid stomach
PUD: peptic ulcer disease
GERD: gastroespphageal reflux disease
Helicobacter pylori (H. pylori)
o Bacterium found in the GI tract of 90% of patients with duodenal ulcers and 70% of those with
gastric ulcers
o Combination therapy is used most often to eradicate H. pylori
Treatment for H. pylori
Eight regimens approved by the FDA
H. pylori is not associated with acute perforating ulcers
It is suggested that factors other than the presence of H. pylori lead to ulceration
Types of acid-controlling agents
50 | P h a r m a c o l o g y
Antacids
𝐻2 antagonists
Proton pump inhibitors
Antacids: mechanism of action
Promotes gastric mucosal defense mechanisms
Secretion of:
o Mucus: protective barrier against HCl
o Bicarbonate: helps buffer acidic properties of HCl
o Prostaglandins: prevent activation of proton pump which results to lower HCl production
56| P h a r m a c o l o g y

Antacids do not prevent over production of acids


Antacids do neutralize the acid once it’s in the stomach
Antacids: drug effects
Reduction of pain associated with acid-related disorders
o Raising gastric pH from 1.3 to 1.6 neutralizes 50% of the gastric acid
o Raising gastric pH 1 point (1.3 to 2.3) neutralizes 90% of the gastric acid
o Reducing acidity reduces pain
Antacids: aluminum salts
Forms: carbonate, hydroxide
Have constipating effects
Often used with magnesium to counteract constipation
Examples
o Aluminum carbonate: basaljel
o Hydroxide salt: alternaGEL
o Combination products (aluminum and magnesium)
Gaviscon, Maalox, Mylanta, di-gel
Antacids: Magnesium salts
Forms: carbonate, hydroxide, oxide, trisilicate
Commonly cause diarrhea; usually with other agents to counteract this effect
Dangerous when used with renal failure—the failing kidney cannot excrete extra magnesium, resulting
in hypermagnesemia
Examples:
o Hydroxide salt: magnesium hydroxide (MOM)
o Carbonate salt: gaviscon (also a combination product)
o Combination products such as Maalox, Mylanta (aluminum and magnesium)
Antacids: calcium salts
ALOH
HCl
H20
ALCL
MgOH MgOH
HCl
H20
MgCl MgCl
51 | P h a r m a c o l o g y
Forms: many, but carbonate is the most common
May cause constipation
Their use may result in kidney stones
Long duration of acid action may cause increased gastric acid secretion (hyperacidity rebound)
Often advertised as an extra source of dietary calcium
o Example: Tums (calcium carbonate)
Antacids: sodium bicarbonate
Highly soluble
Buffers the acidic properties of HCl
Quick onset, but short duration
May cause metabolic alkalosis
Sodium content may cause problems in patients with heart failure, hypertension, or renal insufficiency
(fluid retention)
57| P h a r m a c o l o g y

Antacids and antiflatulents


Antiflatulents: used to relieve the painful symptoms associated with gas
Several agents are used to bind or alter intestinal gas and are often added to antacid combination
products
Over-the-counter antiflatulents
o Activated charcoal
o Simethicone
Alters elasticity of mucus-coated bubbles, causing them to break
Used often, but there are limited data to support effectiveness
Antacids: side effects
Minimal and depend on the compound used
o Aluminum and calcium
Constipation
o Magnesium
Diarrhea
o Calcium carbonate
Produces gas and belching; often combined with simethicone
Antacids: drug interactions
Absorption of other drugs to antacids
o Reduces the ability of the other drug to be absorbed into the body
Chelation
CaCOCaCO CaCO
HCl
HCO3 HCO3
CaClCaCl CaCl
NaHCO3NaHCO3 NaHCO3
HCl
H2CO3 H2CO3H2CO3
CaClCaCl CaCl
H2O
CO2 CO2
52 | P h a r m a c o l o g y
o Chemical binding or inactivation of another drug
o Produces insoluble complexes
o Result: reduced drug absorption
Antacids: nursing implications
Asses for allergies and preexisting conditions that may restrict the use of antacids such as:
o Fluid imbalances
o Renal disease
o Heart failure
o Pregnancy
o GI obstruction
Patients with heart failure or hypertension should use low-sodium antacids such as riopan, Maalox, or
Mylanta II
Monitor for side effects
o Nausea, vomiting, abdominal pain, diarrhea
o With calcium-containing products: constipation and rebound
Monitor for therapeutic response
58| P h a r m a c o l o g y

o Notify healthcare provider if syptoms are not relieved


Histamine type 2 (H2) receptor antagonist
Reduce acid secretion
All available OTC in lower dosage forms
Most popular drug for treatment of acid-related disorders
o Cimetidine (Tagamet)
o Famotidine (Pepcid)
o Ranitidine (Zantac)
H2 antagonists: mechanism of action
Blocks histamine 2 (H2) at the receptors of acid-producing parietal cells
Production of hydrogen ions is reduced, resulting in decreased production of HCl
H2 antagonists: indications
GERD
PUD
Erosive espphagitis
Adjunct therapy of upper GI bleeding
Pathologic gastric hypersecretory condition (Zollinger-Ellison syndrome)
H2 antagonists: side effects
Overall, less than 3% incidence of side effects
Cimetidine may induce impotence and gynecomastia
May see
o Lethargy, headaches, confusion, diarrhea, urticaria, sweating, flushing, and other effects
H2 antagonists: nursing implications
Assess for allergies and impaired renal or liver function
Use with caution with patients who are confused, disoriented, or elderly (higher incidence of CNS side
effects)
Take 1 hour before or after antacids
For intravenous doses, follow administration guidelines
H2 antagonists: drug interactions
Smoking has been shown to decrease the effectives of H2 blockers
o Increases gastric acid production
Proton pump inhibitors
The parietal cells release positive hydrogen ions (protons) during HCl production
This process is called the proton pump
H2 blockers and antihistamines do not stop the action of this pump
Proton pump inhibitors: mechanism of action
Irreversibly bind to H*/K* ATPase enzyme
Result: achlrohydria—all gastric acid secretion is blocked
Proton pump inhibitors: indications
GERD maintenance therapy
Erosive esophagitis
Short-term treatment of active duodenal and benign gastric ulcers
Zollinger-Ellison syndrome
Treatment of H. pylori- induced ulcers
Proton pump inhibitors: drug effects
53 | P h a r m a c o l o g y
Total inhibition of gastric acid secretion
o Lansoprazole (prevacid)
59| P h a r m a c o l o g y

o Omeprazole (prilosec)- the first in this new class of drugs


o Rabeprazole (acipHex)
o Pantoprazole (protonix)
o Esomeprazole (Nexium)
Proton pump inhibitors: nursing implications
Assess for allergies and history of liver diseases
Pantoprazole (protonix) is the only proton pump inhibitor available for for parenteral administration,
and can be used for patients who are unable to take oral medications
May increase serum levels of diazepam, phenytoin, and cause increased chances for bleeding with
warfarin
Instruct the patient taking omeprazole (prisolec):
o It should be taken before meals
o The capsule should be swallowed whole, not crushed, opened, or chewed
o It may be given with antacids
o Emphasize that the treatment will be short-term
Proton pump inhibitors: side effects
Safe for short-term therapy
Incidence is low and uncommon
Other drugs
Sucralfate (Carafate)
Misoprostol (Cytotec)
Sucralfate (Carafate)
Cytoprotective agent
Used for ulcers, erosions, PUD
Attracted to and binds to the base of ulcers and erosions, forming a protective barrier over these
areas.
Protects these areas from pepsin, which normally breaks down into proteins (making ulcers worse)
Little absorption from the gut
May cause constipation, nausea, and dry mouth
May impair absorption of other drugs, especially tetracycline
Binds with phosphate; may be used in chronic renal failure to reduce phosphate levels
Do not administer with other medications
Misoprostol (cytotec)
Used for prevention of NSAID- induced gastric ulcers
Doses that are therapeutic enough to treat duodenal ulcers often produce abdominal cramps, diarrhea
Synthetic prostaglandin analog
Prostaglandins have cytoprotective activity
o Protect gastric mucosa from injury by enhancing local production of mucus or bicarbonate
o Promote local cell regeneration
o Help to maintain mucosal blood flow
Vomiting center and chemoreceptor trigger zone
VC and CTZ
Both located in the brain
o Once stimulated, cause the vomiting reflex
o Found in the medulla oblongata
Definitions
Nausea- unpleasant feeling that often precedes vomiting
Emesis (vomiting)- forcible emptying of gastric and, occasionally, intestinal contents
60| P h a r m a c o l o g y

Antiemetic agents- used to relieve nausea and vomiting


Indications
Specific indications vary per class of antiemetics
General use: prevention and reduction of nausea and vomiting
Mechanism of actions and indications
Many different mechanisms of action
Most work by blocking one of the vomiting pathways, thus blocking the stimulus that induces vomiting
Antihistamine agents (H1 receptor blockers)
o Inhibit ACh by binding to H1 receptors
o Prevent cholinergic stimulation in vestibular and reticular areas, thus preventing nausea and
vomiting
o Diphenhydramine (Benadryl), meclizine (antivert, promethazine (Phenergan)
o Also used for nonproductive cough, allergy symptoms, and sedation
Anticholinergic agents (ACh blockers)
o Binds to and blocks acetylcholine (ACh) receptors in the inner ear labyrinth
54 | P h a r m a c o l o g y
o Block transmission of nauseating stimuli to the CTZ
o Also block transmission of nauseating stimuli from the reticular formation to the VC
o Scopolamine
o Also used for motion sickness
Prokinetic agents
o Block dopamine in the CTZ
o Cause CTZ to be desensitized to impulses it receives from the GI tract
o Also stimulates peristalsis in the GI tract, enhancing emptying of the stomach contents
o Metoclopramide (reglan)
o Also used for GERD, delayed gastric empting
Neuroleptic agents
o Block dopamine receptors in the CTZ
o Chlorpromazine (thorazine)
o Prochlorperazine (compazine)
o Also used for psychotic disorders, intractable hiccups
Tetrahydrocannabinoids (THC)
o Major psychoactive substance in marijuana
o Inhibitory effects on reticular formation, thalamus, and cerebral cortex
o Alter mood and body’s perception of its surroundings
o Dronabinol (marinol)
o Used for nausea and vomiting associated with chemotherapy and anorexia associated with weight
loss in AIDS patients
Serotonin blockers
o Blocks serotonin receptors in the GI tract, CTZ, and VC
o Dolaseltron (Anzemet), granisetron (kytril), ondansentron (Zofran)
o Used for nausea and vomiting for patients receiving chemotherapy and postoperative nausea and
vomiting
Nursing implications
Assess complete nausea and vomiting history, including precipitating factors
Assess current medications
Assess for contraindications and potential drug interactions
61| P h a r m a c o l o g y

Many of these agents cause severe drowsiness; warn patients about driving or performing hazardous
tasks
Taking antiemetics with alcohol may cause severe CNS depression
Teach patients to change positions slowly to avoid hypotensive effects
For chemotherapy, antiemetics often give ½ to 3 hours before a chemotherapy agent
Monitor for therapeutic effects
Monitor for adverse effects
Side effects
Vary according to agent used
Stem from their nonselective blockade of various receptors
harmacology
PHARMACOLOGY
Introduction to Pharmacology
What is a generic name?
- Chemical name of a drug
- Refers to the chemical make-up rather than the advertised brand
- A term referring to any drug marketed under its chemical name without advertising
What is drug classification?
- Similar chemical structures
- Same mechanism of action
Drugs that have similar chemical structures
Same mechanism of action
A related mode of action
Used to treat the same disease
SAMPLES OF CLASSIFICATIONS OF DRUGS
5-Alpha-reductase inhibitor
Angiotensin II receptor antagonist
ACE inhibitor
Alpha-adrenergic agonist
Beta blocker
Dopamine agonist
Dopamine antagonist
Incretin mimetic
Nonsteroidal anti-inflammatory drug – cyclooxygenase drug
Proton-pump inhibitor
Renin inhibitor
Pharmacokinetics
- Process of absorption, distribution, metabolism and elimination of drugs.
1. Absorption- movement of drug particles from GI tract to the body fluids.
2. Distribution- is the process by which drugs become available to the body fluids and tissues.
3. Protein Binding- portion of the drug that is bound inactive.
4. Metabolism- process by which body inactivates or biotransforms drug.
Hepatic firstpass- drugs pass first in the liver before going to the blood.
5. Excretion/Elimination- main route is through kidneys.
6. Half Life- the time it takes for the one half of drug to be concentrated to be eliminated.
Pharmacodynamics
- Study how drug affects the body.
62| P h a r m a c o l o g y

1. Onset of Action- time it takes to reach the minimum effective concentration after the drug is
administered.
2. Peak of Action- occurs when the drug reaches its highest blood or plasma concentration.
3. Duration of Action- Length of time the drug has a pharmacologic effect.
4. Agonist- produce a response
5. Antagonist- Blocks a response
6. Side Effects- Physiologic effect not related to the desired effect
7. Adverse Effect- more severe than side effect. It is dangerous and detrimental.
8. Toxic effects or toxicity- occurs when there is unintentional over dosage of a drug.
9. Tolerance- a decreased responsiveness over the course of the therapy
10. Placebo effect- is a psychological benefit from a compound that may not have the chemical
structure of a drug effect.
RA 9165-Comprehensive Dangerous Drug Act 2002
- Creation of PDEA
- Nurses must administer controlled drugs with a witness
2|Pharmacology
- Sharing of controlled drugs is prohibited
Dosage Calculation Conversion
- 1 grain= 60 mg
1 teaspoon
=
5 ml
1tablespoon
=
15 ml
1 oz
=
30 ml
1oz
=
2 tbsp
1 lbs
=
0.45 kg
1 kg
=
2.2 lbs
1 cup
=
180 ml
1 pint
=
500 ml
1 quart
=
1000ml
Calculating Oral Dosages
Formula:
63| P h a r m a c o l o g y

D/S X Q= Amount to give


Example:
1. Give 150 mg of ibuprofen syrup every 6 hours for 7 days. The available stock dose is 150 mg/ml.
How many ml of syrup should be given to the patient?
Answer: 28 ml per week
Computation:
1. 150mg/150mg x 1 ml= 1ml
2. 1ml x 4 (6 hours in a day is x4)= 4ml/day
3. 4ml x 7 (7 days)= 28 ml per week
Take note: Dosage above 5000mg causes toxicity
Calculation of Flow Rates
1. Milliliters per hours
Formula: number of ml/number of hours= ml/hr
3|Pharmacology
Example: Infuse 1000ml lactated ringer’s solution over 10 hours.
Answer: 100 ml/hr
Computation:
1000ml/10 hours= 100 ml/hr
IV SETS
Microdrip- 60 gtts/min
Macrodrip- 15 gtts/ min
Formula:
Ml/hr x gtts/min all over 60= gtts/min
Pediatric Calculation
1. Fried’s Rule (under 2 y.o)
Formula:
age in mos/ 150 lbs x adult dose= infant dose
2. Young’s Rule ( 2-12 y.o)
Formula:
age in yrs./ age in yrs. + 12
3. Clark’s Rule
Formula:
Child’s weight in lbs./ 150 lbs x adult dose
ANTIBACTERIALS
Bacteriostatic drugs – inhibit the growth of bacteria (inhibit growth to prevent multiplication)
Bactericidal drugs – kill bacteria
Culture and Sensitivity test (C&S)-can detect the infective microorganism
present in a sample and what drug can kill it.
Additive – equal to the sum of the effects of the two antibiotics
Potentiative – occurs when one antibiotic potentiates the effects of the second antibiotic, increasing
the effectiveness
Anatagonistic – is a combination of a drug that is bactericidal and a drug that is bacteriostatic. When
these two drugs are used together, the desired effect may be greatly reduced.
Narrow-spectrum – primarily effective against one type of organism
Broad-spectrum – can be effective against both gram-positive and gram-negative organisms.
Mechanisms of Antibacterial Action
1. Inhibition of bacterial cell-wall synthesis( prevent bacterial cell wall to build up)
64| P h a r m a c o l o g y

2. Alteration of membrane permeability (when taking antibiotics, the cell wall will allow fluids to enter
and it will swell, it will rupture and the cell will die)
3. inhibition of protein synthesis (when taking some antibiotics it will prevent the cell of the bacteria to
produce protein and if no protein, the cell will die) Ribosomes = protein (Rough ER)
4. inhibition of the synthesis of bacterial ribonucleic acid (RNA) and deoxyribonucleic acid (DNA) (A
cell cannot live without DNA, it will not multiply. No DNA = dead cell)
ANTIBIOTIC RESISTANCE
1. Self-Medicating (DO NOT SELF MEDICATE!!)
2. Don’t stop taking medications even to symptoms subsided
4|Pharmacology
3. Repeated exposures
4. Do not argue with the doctor’s order (do not demand a drug you don’t need)
5. Follow orders, take prescribed drugs religiously
6. Do not take antibiotics if it is a viral infection (Antibiotics are ineffective against viruses)
7. Do not give left over/expired antibiotics
8. Give it round the clock
General Adverse Reactions to Antibacterials
1. Allergic (hypersensitivity) reactions, most common side effect = skin test!!
2. Superinfection (kills normal flora or the body)
– *E. coli – large colon, helps produces Vitamin K
– *Bile – large intestine that produces colon bacili that produces brown color in stool
– *Lactobacillus – vagina, helps keep acidic environment
– *Improve normal flora: Drink yakult to increase colon bacili, yogurt, probiotics, kimchi
3. Organ toxicity
PENICILLINS
Penicillin
- Penicillin’s beta lactam structure (beta lactam ring)interferes with bacterial-cell wall synthesis by
inhibitingthe bacterial enzyme that is necessary for cell divisionand cellular synthesis.
- Can be both bacteriostatic and bactericidal
- Mainly referred to as beta-lactam antibiotics
Beta lactamase - enzyme produced by microorganisms, the penicillin cannot penetrate/activate, so
that penicillin cannot kill the bacteria
Pharmacokinetics – Penicillin
After oral administration, penicillin are absorbed mainly in the duodenum and upper jejunum of the
small intestine.
Pharmacodynamics – Penicillin
- Bactericidal in action
Penicillin: Mechanism of Action
- Penicillins enter the bacteria via the cell wall.
- Inside the cell, they bind to penicillin-binding protein.
- Once bound, normal cell wall synthesis is disrupted.
- Result: bacteria cells die from cell lysis.
- Penicillins do not kill other cells in the body.
Natural penicillins
- penicillin G (Pentids, Pfizerpen, Celinex)
- penicillin V (V-Cillin, Betapen-VK)
Aminopenicillins
- Amoxicillin (Amoxin)
65| P h a r m a c o l o g y

- Ampicillin
Penicillinase-resistant penicillins
- Cloxacillin (Ciclox)
- Dicloxacillin (Dynapen)
- Nafcillin (Nafcil)
Extended-spectrum penicillins
- piperacillin, ticarcillin, carbenicillin, mezlocillin
Penicillin-beta-lactamase inhibitor combination drugs:
– ampicillin + sulbactam = Unasyn
5|Pharmacology
– amoxicillin + clavulanic acid = Augmentin
– ticarcillin + clavulanic acid = Timentin
– piperacillin + tazobactam = Zosyn
Beta-Lactamase Inhibitors
- inhibits the bacterial beta-lactamases, making the antibiotic effective and extending its antimicrobial
effect
- Oral use: Clavulanic acid (Augmentin, TImentin)
- Parenteral use: Sulbactam (Unasyn) and Tazobactam (Zosyn)
Side effects and Adverse Reactions
- Nausea, vomiting and diarrhea are common GI disturbances
- Severe allergic reaction leads toanaphylactic shock. Clinical manifestations of a severe allergic
reaction include laryngeal edema, severe bronchoconstriction with stridor, and hypotension.
CEPHALOSPHORINS
- administered when patient is sensitive to penicillin
- have a beta lactam structure and act byi nhibiting the bacterial enzyme necessary for cell wall
synthesis
Pharmacokinetics
- Adminitered orally
- Food decreases absorption
- Administered parenterally.
- Not Absorbed in the GI tract.
Pharmacodynamics
- Inhibit cell wall synthesis
Indications:
- Pharyngitis
- Tonsillitis
- Lower resp tract infections
- Septicemia
- Meningitis
- Perioperative prophylaxis
CEPHALOSPHORIN GENERATIONS
1. First Generation of Cephalosporins – effective against gram postivie bacteria and most gram
negative bacteria)
2. Second Generation of Cephalosporins – same
effectiveness as first generation. Possess a broader spectrum against other gram negative bacteria
*First and second generation of Cephalosporins are
usually localized
66| P h a r m a c o l o g y

3. Third Generation of Cephalosporins – same effectiveness of second and first generations. Also
effective against gram negative bacteria, less effective against gram positive bacteria
4. Fourth Generation of Cephalosporins – is similar to third generation. It is resistant to most beta-
lactamase bacteria. Has broader gram positive coverage than third generation.
* Third and fourth generation of Cephalosporins can cross
the blood-brain barrier
6|Pharmacology
Cephalosporins: First Generation
- Cefadroxil
- Cephalexin (Keflex, Cefalin, Ceporex)
- Cephradine
- Cefazolin Na (Ancef and Kefzol)
- Good gram-positive coverage
- Poor gram-negative coverage used for surgical prophylaxis, URIs, otitis media
Cephalosporins: Second Generation
- Cefaclor (Ceclor)
- Cefoxitine (Mefoxin)
- Cefuroxime (Kefurox and Ceftin)
- Good gram-positive coverage
- Better gram-negative coverage than first generation
- Used prophylactically for abdominal or colorectal surgeries
Cephalosporins: Third Generation
- Ceftizoxime (Tergecin, Ceftizox)
- Ceftazidime (Fortaz, Fortum)
- Cefotaxime (Claforan)
- Cefixime (Suprax)
- Ceftriaxone (Rocephin) Most potent group against gram-negative Less active against gram-positive
Used for difficult-to-treat organisms such as Pseudomonas
Cephalosporin: Fourth Generation
- cefepime (Maxipime) Newest cephalosporin agents. Broader spectrum of antibacterial activity than
third generation, especially against gram-positive
bacteria.
MACROLIDES
- Azithromycin (Zithromax) – long half-life, up to 3 days
- Clarithromycin (Biaxin) – Klaricid
- Erythromycin (E-Mycin)
- administered orally or intravenously (IV)
Pharmacodynamics:
1. Macrolides suppress bacterial protein synthesis (no protein synthesis – attack ribosomes of the cell
to prevent protein synthesis)
2. Azithromycin (Zithromax) has up to a 40- to 68- hour half-life and is prescribed to be taken only
once a day for 3-5 days.
SIDE EFFECTS AND ADVERSE REACTIONS
- GI disturbances such as nausea, vomiting, diarrhea and abdominal cramping
NURSING PROCESS FOR MACROLIDES
1. Monitor patient for liver damage resulting from prolonged use and high dosage of macrolides such
as azithromycin. Signs of liver dysfunction include elevated liver enzyme levels (Alt and Ast) and
jaundice.
67| P h a r m a c o l o g y

2. Administer oral azithromycin 1 hour before or 2 hours after meals. Give with a full glass of water,
not fruit juice. Give the drug with food if GI upset occurs.
7|Pharmacology
3. Administer antacids either 2 hours before or 2 hours after azithromycin (decrease absorption of
drug)
NEVER GIVE ANTACID WITH OTHER DRUGS!!
Lacrosamides – very strong and expensive
Clindamycin (Cleocin) and Lincomycin (Lincocin)
- active against most gram positive organisms
Glycopeptides
1. Vancomycin (Vancocin) – when penicillin is resistant (because of self medication = superinfection),
will take a patient a longer time to heal.
Pharmacokinetics:
Vancomycin is given IV for severe infection.
Pharmacodynamics:
- Vancomycin inhibits bacterial cell wall synthesis (for it not to enter the cell wall)
Side Effects and Adverse Reactions
- Vancomycin may cause nephrotoxicity and ototoxicity.
*Ototoxicity results in damage of cranial nerve VIII, can result in a permanent hearing loss (auditory
branch) or temporary or permanent loss of balance (vestibular branch)
TETRACYCLINE
- inhibiting bacterial protein synthesis
- have bacteriostatic effect
- Tetracycline in combination with metronidazole and bismuth subsalicylate is useful in treating
Helicobacter pylori, a bacterium in the stomach that causes peptic ulcer.
- Tetracycline should not be taken with magnesium and aluminum antacidpreparations, milk products
containing calciumor iron containing drugs, because thesesubstances bind with tetracycline and
prevent absorption of the drug.
- Should be taken with an empty stomach 1 hour or 2 hours after mealtime.
Side effects and Adverse Reaction
- Gi disturbances such as nausea, vomiting, and diarrhea
- Photosensitivity (sunburn reaction)
NURSING PROCESS FOR TETRACYCLINE
1. Advise patient to use sunblock and protective clothing (sunglasses, umbrella) during sun exposure.
Photosensitivity is associated with tetracycline.
2. Educate patient to avoid milk products, iron and antacids. Tetracycline should be taken 1 hour
before or 2 hours after meals with a full glass of water. If GI upset occurs, the drug can be taken with
nondairy foods.
AMINOGLYCOSIDES
- inhibiting bacterial protein synthesis
- are for serious infections
Pharmacodynamics:
- usually administered IV
Streptomycin
- first aminoglycoside available
- used to treat tuberculosis
68| P h a r m a c o l o g y

- Because of ototoxicity and the bacterial resistance that can develop, it is infrequently used today.
Despite its toxicity, streptomycin is the drug of choice to treat tularemia and bubonic pneumonic forms
of plague.
8|Pharmacology
• Parenteral anti-tb
Neomycin sulfate
- for liver diseases
- decreases bacteria in bowel
- preoperative bowel antiseptic abdominal surgery (for killing of colon basili to prevent E. coli from
spreading)
Side effects and Adverse Reactions
- ototoxicity and nephrotoxicity
- the nurse must assess changes in patient’s hearing, balance, and urinary output.
FLUOROQUINOLONES (QUINOLONES)
- used to interfere with the enzyme DNA gyrase
- Norfloxacin is indicated for urinary tract infections.
- Ciprofloxacin is approved for use for urinary tract infections
- Levofloxacin (Levaquin) is used primarily to treat respiratory problems
NURSING PROCESS FOR FLUOROQUINOLONES
1. Administer levoflaxin 2 hours before or after antacids and iron products for absorption. Give with a
full glass of water. If GI distress occurs, drug may be taken with food.
2. Inform patient that photosensitivity is a side effect of most fluoroquinolones. Patient should use
sunglasses, sun block, and protective clothing when in the sun.
ANTITUBERCULARS
- Antitubercular Drugs –take religiously, if not repeat!
Isoniazid (INH)
- When a person is diagnosed with TB, family members are usually given prophylactic doses of
isoniazid for 6 months to 1 year.
- Single drug therapy with isoniazid proved ineffective in treating TB, because resistance to the drug
developed in a short time.
- When a combination of antitubercular drugs was used, bacterial resistance did not occur, and the
duration of treatment was reduced from 2 years to 6 to 9 months.
Side effects and Adverse Reactions
- peripheral neuropathy (destroys nerves, tingling sensation/numbness)
- can be prevented by taking pyridoxine (VitaminB6)
- Hepatotoxicity is an adverse reaction to isoniazid, streptomycin, pyrazinamide and rifampicin.
Side effects and Adverse Reactions
1. Isoniazid = hyperglycemia, hyperkalemia,hypophosphatemia, and hypocalcemia
2. Rifampin = body fluids orange, soft contact lenses permanently discolored
3. Pyrazinamide = gout, sensitivity to light, liver toxicity
4. Ethambutol = dizziness, confusion, hallucinations and join pain
5. Streptomycin = ototoxicity, optic nerve toxicity, encephalopathy, angioedema, CNS and respiratory
depression, nephrotoxicity, and hepatoxicity
NURSING PROCESS ANTITUBERCULAR DRUGS
1. Isoniazid (INH) 1 hour before or 2 hours after meals. Food decreases absorption rate.
2. give pyridoxine (B6) as prescribed with isoniazid to prevent peripheral neuropathy
3. encourage eye examinations (every 3 months) for
69| P h a r m a c o l o g y

patients taking isoniazid and ethambutol, because these antitubercular drugs may cause visual
disturbances.
4. teach patient not to take antacid while taking antitubercular drugs, because they decrease drug
absorption. patient should also avoid alcohol, because it may increase risk of hepatotoxicity
5. inform patient taking rifampicin that urine, feces, saliva, sputum, sweat, and tears may turn a
harmlessr edorange color. soft contact lenses may be permanently stained
Metronidazole (Flagyl)
- intestinal antibiotic
9|Pharmacology
- used to treat intestinal amebiasis
- classified as an antibacterial/antiprotozoal
- commonly used with other agents to treat Helicobacter pylori, which is associated with frequent
recurrent peptic ulcers
Side effects and adverse reaction
- dark or reddish brown urine
- metallic taste
- nausea
- head aches
- Allergies
- seizures
PHARMACOLOGY REVIEWER FOR MIDTERMS
Pain Medications
Inflammation
Part of the first line defense
Response to tissue and injury infection
A vascular reaction takes place in which fluids, elements of blood, leukocytes(WBCs), and chemical
mediators accumulate
Supposed to be normal and counteract infection
Protective mechanism in which the body attempts to neutralize and destroy harmful agents at the site
of injury and to establish conditions for tissue repair
Infection
Caused by microorganisms and results in inflammation
Chemical Mediators
Histamine
o First mediator in inflammatory process
o Cause dilation of arterioles
o Increase capillary permeability
Kinins(Bradykinin)
o Increase capillary permeability
o Increase pain
Prostaglandin
o Increase capillary permeability
o Increase vasodilation
o Increase fever and pain
Pathophysiology
Cardinal signs
o Redness
o Swelling
70| P h a r m a c o l o g y

o Heat
o Pain
o Loss of function
Important Terms
We don’t reverse histamine and bradykinin
Prostaglandins (chemical mediator)
Effects: vasodilation, relaxation of smooth muscle, increased capillary permeability, and sensitization
of nerve cells to pain cause of inflammation and pain
Cyclooxygenase (COX)- Enzyme responsible for converting arachidonic acid to prostaglandins
COX 1- traditional; converts arachidonic acids to prostaglandins
o The stomach has prostaglandin coating= protection and regulate blood platelets
o COX 1 converted to COX 2= prostaglandins
COX 2- triggers inflammation and pain
Stopping the pathway= decreases inflammation
10 | P h a r m a c o l o g y
Earliest type of NSAIDs
Aspirin (salicylates)- Inhibit biosynthesis of prostaglandins (prostaglandin inhibitor)
o Prostaglandin inhibitor- Affect the inflammatory process (anti-inflammatory agents)
o Anti-inflammatory agents- relieve pain (analgesic), reduce elevated body temperature (anti-pyretic)
also an anticoagulant (platelet aggregation) to prevent stroke and MI; it is over the counter; not safe
for children; not any more used for inflammation and pain due to many side effects and adverse
Most NSAIDs are used to decrease inflammation and pain for patients who have some type of arthritic
condition
o For chronic pain or arthritis- NSAIDS is used
Anti-Inflammatory Drugs
Anti-inflammatory drug groups
o Non-steroidal and anti-inflammatory drugs
o Corticosteroids
o Disease modifying antirheumatic drugs
o Anti-gout drugs
Either steroidal or non-steroidal
Non-Steroidal Anti-Inflammatory Drugs
More appropriate for reducing swelling, pain, and stiffness of joints
Common but has a lot of side and adverse reactions
Common for arthritic pain, menstrual cramps, migraine, and toothache
COX 2- more potent in inflammation
1st- blocks cox 1; 2nd- blocks cox 2
Prostaglandin- potent inflammatory agent
First generation of NSAIDs
Blocks COX 1 and COX 2, but blocks COX 1 first which prevents the conversion to COX 2
Ketorolac- given as IV; post-op patients
2nd gen
Cox 2 inhibitors- safer compared to the first generation
Aspirin
Not used as an antipyretic but anti-coagulant
Gastric distress- prostaglandins lines the stomach w/o cox 1= Upper GI bleeding
Pharmacokinetics- if taken as anticoagulant should be given with meals; after lunch or 12 afternoon;
PC (post meals)
71| P h a r m a c o l o g y

Hypersensitivity to aspirin
o Effects: tinnitus (ringing of the ears), vertigo (dizziness), anaphylaxis, and bronchospasm
(contraction of the bronchial muscles=difficulty in breathing)
Nursing process
Observe patients for signs of bleeding
o Melena- black tarry stools- sign of upper GI bleeding
o Bleeding gums, petechiae, ecchymosis, and purpura when taking high dosages of aspirin
300 mg for pain; 81 mg for anticoagulant
Pharmacokinetics
Aspirin is well absorbed in the GI tract which can cause GI upset
o Should be taken with food, milk, or water
o Should not be taken on the last trimester of pregnancy= causes premature closure of the ductus
arteriosus
Nursing Interventions
Eliminate risk for injuries
Advise patient to not take aspirin with alcohol (increases gastric juices)
Educate patient to inform dentist if aspirin is taken
Instruct to discontinue aspirin 5 to 7 days before surgery to reduce bleeding
W/ dengue= cannot prescribe aspirin will cause platelet drop due to anticoagulant effect
11 | P h a r m a c o l o g y
Keep aspirin out of reach of children
Reye’s syndrome- problem for aspillet or aspirin
o Viral problem then given aspirin; <7 yrs old= causes liver damage= brain encephalopathy.
New formula for aspillet- neo-aspillet
Phenylacetic acid dervatives
Ketorolac (Toradol)
First injectable NSAID
Recommended for short term management of pain
o IV formula; usually given to post-op patients
Possible of GI bleeding and irritation
Because it is not safe it is used as creams
Skin test first
Propionic acid derivatives
Ibuprofen
Reduce fever
Treat pain or inflammation caused by many conditions such as headache, toothache, back pain,
arthritis, menstrual cramps, or minor injuries
Nephrotoxicity and GI distress= GI ulcers= most common
Side and adverse effects of NSAIDs
Ototoxic- tinnitus
GI distress
Dizziness, confusion, and edema
Blood dyscrasia and dysrhythmias
Nephrotoxic- worst adverse reaction= acute kidney injury
Upper GI bleeding
CVD- in long term use
Allergy
Constipation
72| P h a r m a c o l o g y

Selective COX-2 Inhibitors (Second generation NSAIDs)


Coxibs- end names
Celecoxibs (Celebrex) and Nabumetone (Relafen)
COX 2 inhibitors- decrease risk of peptic ulcers
o Clinical trials revealed effects= cardiovascular diseases and stroke
Designed for arthritis
NSAIDs are not really safe; prescriptions and advices are needed.
Anti-gout Medications
Anti—inflammatory gout medication
For elderly- gouts can cause bone and joint deformity
Colchicine- first drug to be given for gouts
Stops migration of WBCs to the injured site
o Reduce swelling= reduce pain
Gastric inflammation- a common problem
o Should be taken with food
o With high doses- nausea, vomiting, diarrhea, or abdominal pain occurs in approximately 75% of
patients taking the drug
Has drug interaction with antibiotics
Uric acid inhibitor
Prophylaxis/To prevent gouty arthritis- allopurinol; for maintenance only
Allopurinol (Zyloprim)- never given for acute pain
Inhibits final steps or uric acid biosynthesis
o Lowers serum uric acid levels, preventing the precipitation of an attack
Diet modification+allopurinol
Taken during chemotheraphy
12 | P h a r m a c o l o g y
Pharmacodynamics: xanthine oxidase inhibitor
Inhibits the production of uric acid by inhibiting the enzyme xanthine oxidase
o Hypoxanthine=xanthine=uric acid
o Needed in the synthesis of uric acid
If not converted to uric acid it would stay as a nonpotent serum
Interferes with conversion of ATP/ADP
Side effects and adverse reactions- GI distress and dysrhythmias
Uricosurics
Increase excretion uric acid through urine
Probenecid-Affects the nephrons- In the distal tubule; prevent recycling of uric acid
Colchicine (prevent migration of WBC)+probenecid(excretion)= prevent acute attack
Use of aspirin should be avoided, because it causes uric acid retention
Corticosteroids- from the adrenal cortex
Prednisone, prednisolone, and dexamethasone
Control inflammation by suppressing or preventing the many components of the inflammatory process
Not the drug of choice for arthritis
o Due to numerous side effects
Frequently used to control arthritic flare ups
Last option for inflammation
Exact mechanism is unknown thought to decrease lysosomal activities
Decreases immune system
Can cause cushing’s syndrome as effect
73| P h a r m a c o l o g y

o Buffalo hump
o Moon face
o Central obesity
Can also cause GI ulcers
Taper off when discontinuing- if patient is using a steroid tablet; it should not be immediately stopped
because it will cause hypertension.
o Gradually decrease dosages
Non opioid and opioid analgesic
NSAIDs- can be classified in non-opioid analgesic
Non opioid
Known as non-narcotic analgesic
Acetaminophen aka paracetamol- inhibit prostaglandin synthesis
Antipyretic and analgesic
Non-opioid but is not an NSAID
Pharmacokinetics
Short half-life can be administered every 4 hours as needed
o Max dose- 4g/day
o Frequent takes should limit the dose to 2 g/day to avoid possibility of hepatic or renal dysfunction
o Large doses or overdoses can be toxic to hepatic cells
o Adult- 10 days max
o Children- 5 days max
Or 1,000 mg Q6/day- abroad to prevent seizures
Antidote for paracetamol overdose- acetylcysteine- mucolytic (respiratory drug; lyses mucus
secretions) and also a liver protectant
Pharmacodynamics
Eliminates mild to moderate pain
Side effects and adverse reactions
o Overdoses- extremely toxic to the liver cells= hepatotoxicity= s/sx (NAVDA)
Opiod: Narcotic Analgesic
Known as narcotic analgesic
13 | P h a r m a c o l o g y
Addiction- psychological and physical dependence upon a substance beyond normal volume control,
usually after prolonged use of a substance
Effects: analgesia, respiratory depression, euphoria, sedation, orthostatic hypotension (decrease in
blood pressure when rising from a sitting-lying position)
Pain=peripheral nerves=chemical mediators (histamine, bradykinin, and prostaglandins)
=spinothalamic tract= thalamus (pain is already determined) =parietal lobe(processing; pain)= frontal
lobe(reaction)
Non opioid- blocks only the chemical mediators
o Mild to moderate pain
Narcotic (opioids)- moderate to severe pain
o Acts on the brain; decreases function of brain so it cannot interpret pain; depresses CNS function
Patients can die due to apnea; respiratory arrest
Tramadol
Opioid medication used to treat moderate to severe pain
Often combined with paracetamol to relieve pain
1/10 the potency of morphine
Equally potent compared to pethidine and codeine
74| P h a r m a c o l o g y

Can be in IV and tablet form


Vomiting and hypotension- side effect
Side effects and adverse reactions: it is a downer- not recommended for suicidal patients
o Constipation, itchiness, nausea, and seizures
Morphine
For chronic pain
Check PR, if below 101 bpm do not give morphine
Only used for patients with MI; last option
Patients with cancer and post-op patients
Antidote for morphine- Naloxone or Narcan
o When there is respiratory depression
Opioids does not only suppress pain impulses but also respiration and coughing by acting on the
respiratory and cough centers in the medulla of the brainstem
Nursing process for Morphine
Morphine overdose: pinpoint pupils and slower reaction to light; decreased RR; decrease gag reflex
Codeine
Analgesic but not as strong as morphine
o But also relieves pain and cough
1/15 to 1/20 potency but can also relieve mild to moderate pain
Most opioids (except meperidine; Demerol) has an antitussive effect= cough suppression= cause sore
throat and chest pain
A derivative of morphine
Used as an antitussive- cough suppression
Meperedine (Demerol)
Common for OB px; pacreatitis
Large doses= neurotoxicity; nervousness, tremors, agitation, irritability, and worst case: seizures
Not for long-term usage
Dose is limited to 600 mg/24-hour period no longer than 48 to 72 hours
Patient controlled analgesia(PCA)-Syringe pump a patient can control and it will deliver the drug;
everytime there is pain the patient can press it the Demerol will enter.
Fentanyl
Can be mixed with anesthesia
Available in injections and dermal patches
Side effect: hypotension, decrease in HR
75 times stronger than morphine
Most widely used synthetic opioid
Hydromorphone (dilaudid)
A semisynthetic opioid similar to morphine
14 | P h a r m a c o l o g y
Analgesic effect is approximately 6 times stronger/more potent then morphine with fewer hypnotic
effects and GI distress
Patients with stage 4 cancer
Side effects and adverse reactions: respiratory depression, orthostatic hypotension, tachycardia,
drowsiness, and mental clouding
Withdrawal syndrome- irritability, diaphoresis, restlessness, muscle twitching, increased PR and BP,
agitation, and anxiousness
Side effects and Adverse Reactions of opioid analgesics In general
Anxiety, depression, flu-like symptoms, headache, GI upset, hallucinations, hypoventilation and apnea
75| P h a r m a c o l o g y

Naloxone (Narcan)
Opiod antagonist
Administered IM or IV
Monitor for bleeding and hypotension
Anxiety
Anxiety is more on GABA- drugs decreases this
SSRI- used for depression; selective serotonin receptor inhibitor
Sleep and Insomnia
Following stage 4 of NREM sleep, will have 5-20 minutes of REM sleep with dreaming and increased
physiologic activity
REM sleep is felt to be mentally and emotionally restorative
REM deprivation can lead to psychological problems and psychosis
Insomnia
Can result from pain, anxiety, illness, changes in environment, and from certain medications
Benzodiazepines
Used for anxiety and insomnia
Prototype: diazepam
Can result in psychologic dependency, thus, abuse
Withdrawal symptoms can result if abruptly stopped; should be gradually tapered and discontinued
Can cause excessive sedation, impairment of physical and mental activities, and respiratory
depression
Not for long-term use
Do not suppress REM sleep
Vary in plasma half-lives, metabolites, and uses
Bind with benzodiazepine receptors in nerve cells of the brain, the receptor also has binding sites for
GABA
o When benzo/GABA binding occurs, then chloride ions in the cells causing decreased response to
the excitatory neurotransmitters such as norepinephrine
Competes with the GABA sites
Action potential- positive causes it
o Na goes in= electrical impulse
o Blocking GABA= chloride (-) goes inside= less excitation= more relaxation
K
Pho
Pro
Mg
HCO3
Cl
Ca
Na
15 | P h a r m a c o l o g y
Classifications: muscle relaxants, hypnotics, and for anxiety
Pharmacokinetics
Well-absorbed orally
o Given oral or IV
Widely distributed in body tissues
Highly bound to plasma proteins
Lipid soluble; can easily enter the CNS
76| P h a r m a c o l o g y

Metabolized by the liver by cytochrome p450 enzymes and by CYP3A4 enzymes in intestines
Most benzodiazepines are metabolized into active metabolites that require further metabolism before
clearance
Depending on half-life, can result in accumulation and subsequent adverse drug effects
High doses= decrease brain function (CNS depression), decreases vital signs
Do not give together with downer drugs
Shorter-acting benzos
o Versed(midazolam)- 30-60 minutes
o Halcion(triazolam)- 4-6 hours
o Dalmane(flurazepam)- 6-8 hours
o Xanax(alprazolam)-4-6 hours
given to px with cardiac conditions; MI patients
Given at 9 pm
One of the safe types of benzodiazepines
o Serax(oxazepam)- 2-4 hours
Longer-acting benzos
indicated for seizure px
o Klonopin(clonazepam)- action may last for weeks
o Librium(chlordiazepoxide)- several days
o Tranxene(chloraxepate)- lasts for days
Drugs and its common uses
Xanax(alprazolam)- anxiety and panic disorder
Librium(chlordiazepoxide)- anxiety and alcohol withdrawal
Klonopin(clonazepam)- seizure disorders and panic disorders
Valium(diazepam)- anxiety, seizure disorders, alcohol withdrawal, muscle spasms, and for
preoperation medication
o For active seizure give valium IV (for babies)
o If no IV, IM or anal
Dalmane(flurazepam)- insomnia
Ativan(lorazepam)- anxiety and preop medication
Versed(midazolam)- preop sedation and anesthetic induction
Restoril(temazepam)- insomnia
Contraindication to use
Respiratory disorders
Severe liver or kidney disease
History of alcohol or drug abuse
o If there is alcohol abuse= higher doses are needed
Hypersensitivity reactions
o Skin tests are not needed= because allergic reactions are not common
Valium (Diazepam)
Valium is prescribed 30 minutes before operation
o Only lasts for 1 hour; short acting
o Decrease stress and anxiety
o Antidote- flumazenil
Used as hypnotics
Anti-anxiety meds may also be given to px with cardiac diseases
Miscellaneous antianxiety and sedative-hypnotic agents
Noctec (chloral hydrate)- chloroform
77| P h a r m a c o l o g y

o Oldest sleeping medication


o Does not affect REM sleep
o Tolerance increases after two weeks
16 | P h a r m a c o l o g y
Natural hypnotics
o Melatonin-hormone- produced by the pineal gland
o Endogenous melatonin is derived from tryptophan= converted to serotonin= melatonin
o Melatonin affects sleep-wake cycles; released during sleep and levels are low during waking hours
o Used for jet lag due to disruption of circadian rhythms
o Caution with patients with liver or renal problems
Benzodiazepine withdrawal
Mild s/sx- sweating, irritation
o Occurs to half of clients after taking doses for 6-12 weeks or longer
o Severe s/sx if taking large doses for 4 months or longer with abrupt discontinuation
o Effects are related to decrease in GABA neurotransmission resulting to CNS stimulation
Signs and symptoms include: anxiety, psychomotor agitation, insomnia, irritability, headache, tremors,
and palpitations
o Others- confusion, depersonalization, psychosis, and seizures
Severe s/sx most pronounced in short-acting drugs such as Xanax, Ativan, and Halcion. Reduce dose
by 10-25% every 1-2 weeks over 4-16 weeks
Benzodiazepine toxicity
Effects include: excessive sedation, respiratory depression, and coma
Has shorter duration than many benzos, so, repeated dosing may be necessary
o For overdose, give 0.2 mg/30 seconds, wait 30 seconds, then 0.3 mg/30 seconds, then 0.5 mg/60
seconds up to max of 3 mg
Toxicity is common in geriatric patients and children
Antidote: Romazicon (flumazenil)
Naloxone is for opioids, flumazenil is for benzodiazepines
Drug therapy for anxiety
Not recommended for everyday stress
For chronic pain, have not proven to be effective
Antipsychotics
Psychosis- severe mental disorder characterized by disordered thought processes, inappropriate
emotional responses, bizarre behavior, agitation, aggressiveness, hostility, social withdrawal,
deterioration in occupational and social functioning, hallucinations, and paranoid delusions
Nursing diagnosis: Altered thought processes
To be classified as psychotic: should be observed with paranoid delusions and hallucinations
Psychosis
Hallucinations- false interpretations of sensory perceptions
o Objects or people are absent
o Unable to distinguish between false perception and reality
o Schizophrenia or bipolar disorder- auditory
o Delirium- visual or tactile
o Dementia- visual
Common in alcoholics- delirium and visual or tactile
Illusions- objects are present but different interpretation
Delusions- false beliefs that persist in absence of reason or evidence
o Delusion of grandeur- perceives self with high standing
78| P h a r m a c o l o g y

o May believe others control their thoughts, feelings, or seek to harm them
Psychosis may be acute or chronic
o Acute- confusion or delirium; can be precipitated by illness, drug effects, or superimposed on
chronic dementias
Schizophrenia
Consist of variety of related disorders
Does have a genetic predisposition
Positive symptoms- high brain activity
o CNS stimulation, agitation, behavioral disturbances, delusions, hallucinations, insomnia, and
paranoia
Negative symptoms- low brain activity
17 | P h a r m a c o l o g y
o Anhedonia, lack of motivation, blunted effect, poor hygiene, poor social skills, and social withdrawal
Etiology of schizophrenia
Evidence indicates abnormal neurotransmission systems in the dopaminergic, serotonergic, and
glutamatergic system
Interplay between the systems; one system may affect the other
Real reason is still unknown
Theoretical
o More on dopamine= increased levels which leads to psychosis
o Imbalance in amount of neurotransmitters, most notably dopamine
o Overacitivity accounts for the positive symptoms of schizophrenia and underactivity in another part
of the brain may account for the negative
Glutamatergic dysfunction may be genetically linked as well as causative in the cognitive impairments
and negative s/sx of this disorder
Antipsychotic drugs
Categorized as “typical”, “first generation”, or “conventional”= phenothiazine
“Atypical” or “second generation”= newer nonphenothiazine
Discovered in the 1950s- first generations
Second generations- modern drugs
Mechanism of action
Most bind to D2 dopamine receptors and block the action of dopamine but positive effects only occur
overtime
Theory: blockage of dopamine receptors leads to changes in receptors with effects on cell metabolism
and function
With chronic drug administration, it is postulated that drugs re-regulate the abnormal
neurotransmission systems
First generations are only effective to positive symptoms
Indications of Phenothiazine
Schizophrenia
o Psychotic symptoms associated with brain impairment (injuries)
o Useful in manic phase of bipolar affective disorder until Lithium (drug of choice) becomes effective
Antipsychotics may also be given to intractable hiccups
Uses
Schizophrenia
o Nausea and vomiting- affect chemoreceptor trigger zone in medulla
o Intractable hiccups- mechanism of action is unclear
Use with caution in:
79| P h a r m a c o l o g y

o BPH
o Seizure disorders
o Glaucoma
Contraindications of Phenothiazine
Liver damage
CAD
Cerebrovascular disease
Parkinsonism
Bone marrow depression
Severe hypotension and hypertension
Before administering check the vital signs
Pharmacokinetics
PO or IM
Prototype: Thorazine(chlorpromazine)
Metabolized by the cytochrome p450 system
No psychological dependency but physical dependency can occur
Withdrawal s/sx may occur
18 | P h a r m a c o l o g y
Side effects include:
o CNS depression
o Anticholinergic effects
o Antiemetic effects
o Lowering of body temperature
o Hypersensitivity reactions
o EPS
o Weight gain
o Orthostatic hypotension
Examples of phenothiazines
o Thorazine(chlorpromazine)
o Prolixin(fluphenazine)
o Compazine(prochlorperazine)
o Stelazine(trifluoperazine)
o Mellaril(thioridazine)- used less commonly due to cardiac side effects
Thorazine (chlorpromazine)
1st generation phenothiazine- has higher risk of having the side effects
For psychosis or post-op patients with hiccups
EPS- extrapyramidal symptoms= parkinsonism
o Robot-like
o Extrapyramidal Symptoms (EPS)
Extrapyramidal Symptoms (EPS)
Affects EPS and basal ganglia
o System includes descending fibers that reach the medulla other than by the corticospinal tracts
o Important in maintenance of equilibrium and muscle tone
o Symptoms: dystonia, akathisia, tardive dyskinesia, and parkinsonism
Dystonia- prolonged muscle contractions causing twisting and repetitive movements or abnormal
posture. May have rhythmic jerks
Akathisia- restless, unable to sit still
o Most common symptom
80| P h a r m a c o l o g y

Chloreiform movements- involuntary muscular twitching


Tardive dyskinesia- hyperkinetic movements of the face
o Sucking and smacking lips, facial grimaces, and tongue protrusion
Non-phenothiazines
1st generation antipsychotics
Haldol(haloperidol)- potent, long-acting
Causes high incidence of EPS
Useful in mental retardation with hyperkinesia, Tourette’s and Huntington’s disease
Comes in oral form and even once in a month injection form
Loxitane(loxapine)
Moban(molindone)
Orap(pimozide)- for Tourette’s when Haldol is not effective
o Can cause tardive dyskinesia, motor seizures, and even sudden death
2nd generation of antipsychotics
Drugs of choice
Effective in treating the positive s/sx of psychosis and have greater effectiveness in relieving the
negative s/sx
Less likely to cause EPS
Effective for both positive and negative symptoms
Clozaril (clozapine)
Prototype of atypicals
Effective but considered a second line drug because of association with agranulocytosis
Check weekly WBCs
Neutrophils will go down to <50
Haldol (Haloperidol)
19 | P h a r m a c o l o g y
A mild sedative
IM injection
Zyprexa(olanzapine)
Can cause EPS but not agranulocytosis
Cause less sedation, less orthostasis, and anticholinergic effects
Seroquel(quetiapine)
Blocks dopamine and serotonin
Relieves positive and negative symptoms
Many drug interactions
o Metabolized by the cytochrome p450 system
Risperdal(risperidone)
Blocks dopamine and serotonin
Affects both positive and negative symptoms
Safest and is the first choice of treatment
Also metabolized by the cytochrome p450 system
Can cause parkinsonism
2nd generations have better results and less probability to EPS
Duration of drug intake is for years or forever, once stopped psychosis will go back
Drug selection
Duration of therapy- generally for many years as relapses can occur
Drug withdrawal- can occur if medications are stopped abruptly
o Can result in cholinergic effects such as diarrhea, drooling, and insomnia.
81| P h a r m a c o l o g y

o Should be tapered over several weeks


Atypicals are drugs of choice as they:
o May be more effective
o Produce milder adverse effects
o Patients display greater compliance in taking them
o Drawback include: glucose intolerance, weight gains, and also are costs
Treatment of EPS
Treat with anticholinergic antiparkinson medication such as:
o Benadryl(diphenhydramine)
o Symmetrel(amantadine)
o Eldepryl(selegiline)
More likely to occur with older antipsychotic drugs
Treatment is usually for three months then gradual discontinuation
S/sx generally do not recur
Neuroleptic malignant syndrome (NMS)
Rare but potentially fatal reaction
o May occur hours to months after initial psychotic drug use
Present with high grade fever, muscle rigidity, agitation, confusion, delirium, tachycardia, respiratory
failure, and acute renal failure
Tx- stop drug, supportive care
o Dantrolene and amandatine- anti Parkinson drugs; also used to counteract NMS
No prophylactic for psychosis
Haloperidol- has highest EPS probability; 2nd is thorazine
Benadryl- anti-histamine to counter act side effects: EPS
Antidepressants and mood stabilizers
Mood disorders
Include: depression, dysthymia
o bipolar disorder (manic/panic= span of 2 weeks)
o cyclothimia- sad then happy but does not reach manic-panic situation
20 | P h a r m a c o l o g y
o Seasonal affective disorders- moods varies on the season
If there was one depressive episode, higher risk for having another
Monoamine neurotransmitter dysfunction
Complex ethiology affecting neurotransmitter and receptors
Felt to be partially a result of deficiency in norepinephrine and serotonin
Interplay bet neurotransmitters: norepinephrine, serotonin, dopamine, and acetylcholine
Neuroendocrine factors
Increased secretion of corticotrophin-releasing hormone by hypothalamus, adrenocorticotrophic
hormone by pituitary, and cortisol by the adrenal gland
o Cortisol- steroid; acts on the fight or flight
o Increased cortisol may decrease numbers of sensitivity of cortisol receptors and lead to depression
Other factors
Interplay with thyroid and growth hormones
Immunity
May be linked in genetics
Environmental factors resulting in structural changes in brain such as child abuse
Types of mood disorders
Depression- imbalance of a neurotransmitter
82| P h a r m a c o l o g y

o Lack of self-interest, anhedonia


Dysthmia- chronically depressed mood
o For at least 2 years
Bipolar- depression alternating with mania
Cyclothymia- mild type of bipolarity
Must be present for 2 years to be considered a form of depression
General characteristics of antidepressants
Vary in adverse effects
Must achieve serum level before improvement seen (2-4 weeks)
Taken orally, undergo significant first pass metabolism
Multiple drug interactions as are metabolized by cytochrome 450 enzymes
Medications will not work immediately
Mechanism of action
Normalize neurotransmission systems by altering the transmitters and receptors
Also modify interactions between neurotransmission system and endocrine functions
o ACTH (adrenocorticotrophic hormone) and cortisol levels
Neurotransmitters that are not bound are inactivated by reuptake or are metabolized by monoamine
oxidase
o Monoamine oxidase- enzyme that destroys norepinephrine and epinephrine- which is high during
stress
Contraindications
Used cautiously in schizophrenia, mixed mania, and depression
o Schizophrenia- can be used in positive symptoms
Suicidal tendencies
In severe renal, hepatic, or cardiovascular disease
Narrow-angle glaucoma- fluid buildup in the iris; increase intraocular pressure
Seizure disorders
Antidepressants
Tricyclics- oldest type of antidepressant
o More serious adverse effects especially anticholinergic and cardiac effects, weight gain, and
sedation
o Can cause photosensitivity
o Elavil (Amitriptyline)- most common antidepressant
o Tofranil (imipramine)
o Sinequan (doxepin)
21 | P h a r m a c o l o g y
o Norpramin (desipramine)
Selective serotonin reuptake inhibitors (SSRI)
o Adverse effects include: nausea, sexual dysfunction, headache, increased risk of gastrointestinal
bleeding
o Safest antidepressant; most prescribed
o Never coadminister with monoamine oxidase inhibitor (MAOI)
o Prozac (fluoxetine)- long duration
o Zoloft (sertraline)
o Celexa (citalopram)
o Paxil (paroxetine)- long duration
Monoamine oxidase inhibitor (MAOI)
o Considered dangerous due to a lot of interactions with food and drugs
83| P h a r m a c o l o g y

o Preserve norepinephrine
o Should not be combined with any other antidepressant
o Foods that contain tyramine, a monoamine precursor of norepinephrine, when taken with MAOIs
can lead to severe hypertension, stroke, or heart attack
o Diets should be checked
Tyramine can lead to severe hypertension (severe adverse effect of MAOI)
Avoid high in MSG
Avoid aged cheese, meats, concentrated yeast extracts, sauerkraut, and fava beans
o Marplan (isocarboxazid)
o Nardil (phenelzine)
o Parnate (tranylcypromine)
Lithium
o Mood stabilizing agent
o For bipolarity- mania phase
o Must be closely monitored
o Excreted by kidneys, so must have adequate renal functioning
o If hyponatremic= lithium toxicity
o This is toxic and can damage the kidneys
o Before administering check creatinine levels and clearance
o Diet should be on the normal salt level
2L water/2g salt
o Lithium toxicity
Therapeutic levels- should be checked weekly 0.6-1.2 MEQ(milliequivalent)/L
Severe toxicity- <2
Liver cannot detoxify because it is a mineral
o Mild toxicity- diarrhea, vomiting, fatigue
o Severe toxicity- If not excreted by the body adverse effects will occur
Seizure, coma, hypotension, hyperthermia, kidney failure, and death
Drugs acting on the CNS and PNS
Barbiturates
o Antisizure
o Sedative
o Muscle relaxant
o Anti-anxiety
o Secobarbital (seconal)- treatment for insomnia
o Pentobarbital (Nembutal)- used in intracranial pressure
most common barbital
o Thiopental sodium (pentonal)- treatment for insomnia
o These are antiseizure medicines
Seizure- abnormal firing of the neurons; most common in pediatric patients
o Not used on ongoing seizures; due to long onset of action
Action of barbiturates
o Activates GABA receptors
o Delayed closing of chloride receptors
o Hyperpolarization of cells- becomes more negative polarity
Depression of CNS
Benzodiazepines
o Also considered as antiseizures
84| P h a r m a c o l o g y

o Should only be used for 10 days


o Valium should be ready on bed side for active seizures (5mg; syringe); due to fast onset of drug
22 | P h a r m a c o l o g y
o Valium (diazepam); used to alleviate anxiety, and Klonopin (clonazepam)
o Valium administered IV during active seizures
o Dalmane (Flurazepam)
o Xanax (alprazolam)
o Restoril (temazepam)
o Ativan (lorazepam)- alleviate anxiety
Action of benzodiazepines
o Binds to GABA receptors
o Increases number of opening in the chloride channel
o Hyperpolarization of cells
CNS depression
o Epilepsy- To confirm epilepsy EEG is done; maintenance drug is phenytoin
Chronic, usually lifelong disorder
Classification of seizures
o Grand-mal(tonic-clonic); more on motor neurons
o Petit mal(absence)- has memory loss after
Action of anticonvulsants
o Antiseizures
o Suppresses sodium influx through the drug binding to the sodium channel= prevention of neuron
firing
o Suppress calcium influx
Valproic acid and ethosuximide are examples of drugs that suppress calcium influx
o Increase action of GABA= inhibit neurotransmitter throughout the brain
Identification of seizures
o Gran mal (tonic-clonic) and petit mal (absence)
o With the use of anticonvulsants 70% of seizures are controlled
o Take throughout the person’s lifetime
o In some cases, it is discontinued if no seizures have occurred in the past 3 to 5 years
Hydantoins (classification); Phenytoin
o Prophylactic for seizures
o Not given with active seizures; just maintenance to avoid seizures
o Should not be given to pregnant patients; fetus may be aborted is taken in the 1st trimester
Teratogenic=congenital defects
o Maintenance: capsule form; wards: IV form and is only compatible with NS
If D5 water is used it will lead to precipitation=crystallization of the drug=embolus if given IV
Flush line with NS before and after administering to reduce venous irritation
Cannot be given as IM injection; irritates tissues and may cause damage
It is acidic, therefore, it may irritate patient if IV is out
o Effective for gran mal and psychomotor seizures; not effective for petit mal seizures
o Side effects and adverse reactions
Always check for gingival hyperplasia, because it is common for phenytoin
Overgrowth of gum tissues or reddened gums that bleeds easily
Thrombocytopenia-low platelet count
Leukopenia- low white blood cell count
Elevated blood sugar (hyperglycemia) results from the drug which inhibits the release of insulin
85| P h a r m a c o l o g y

Baribiturates (Phenobarbital)
o Long-acting; effective for grand mal, partial seizures, and acute episodes of status epilepticus
Status epilepticus- rapid succession of epileptic seizures
o Can depress vital signs
Succinimides
o Zarontin (ethosuximide)- drug of choice for the absence or petit mal seizures
o Succinimide of choice
o Calcium influx inhibitor
Valproate (valproic acid)
o Hepatotoxicity is one if the possible adverse reactions
o Given as a maintenance drug; can be given every day for patients with epilepsy
Not be given to very young children and to patients with liver disorder
Can cause reye’s syndrome to children
Nursing considerations/process
o Phenytoin
o Teach patient to shake suspension form thoroughly before use to adequately mix medication and
assure accurate dosage
23 | P h a r m a c o l o g y
o Do not drive or perform hazardous activities when initiating anticonvulsant therapy
Drowsiness may occur
o If pregnant, consult with healthcare provider, phenytoin and valproic acid may have teratogenic
effects
o Avoid alcohol and other CNS depressants= cause added depressive effects on the body
o Do not stop abruptly, if medications are stopped abruptly effects will be reversed (seizure rebound)
o Pinkish or reddish brown urine= normal and harmless
Neuromuscular blocking agents
o Paralytic agents
o Given IV
o Atracurium
o Cisatracurium
o Rocuronium
o Succinylcholine
Drug function of neuromuscular agents
o By blocking acetylcholine in the neuromuscular junctions (NMJ)
o Stop binding in the acetylcholine sites
Indications of neuromuscular blocking agents
o Skeletal muscle relaxant
o Used in endotracheal intubation
o Aid in muscle spasm in tetanus
o Reduce use of anesthesia
o Mechanical ventilation
o There should be an ambubag at bedside before administering paralytic agents
Nursing process
o Usually given by an anesthetist
o Assess for renal, hepatic, or pulmonary disease
o Assess for neurologic disorders such as
myasthenia gravis
Spinal cord injury
86| P h a r m a c o l o g y

Multiple sclerosis- decrease in the sensations; problem in the myelin sheaths; slow transmission of
sensations
Anticonvulsants leads to CNS depression
Drugs affecting the autonomic nervous system
Autonomic nervous system
o Self-governing and automatic
o Composed of the sympathetic and parasympathetic
o In pharmacology
Adrenergic agents- sympathomimetic
Cholinergic agents- parasympathomimetic
Considered as inotropes- increases blood pressure; produces sympathetic response
Sympathetic- thoracic
Parasympathetic-vagus nerve
Drug class: Adrenergic agents
o Dopamine, dobutamine, norepinephrine, and epinephrine
o Used for shock; if <90/60 it is considered to be in hypovolemic shock
o If in 6 hours and not corrected it will lead to renal damage; if not corrected can cause multiple organ
damage
o Types of receptors
Alpha- vasoconstriction
Beta- heart rate
Dopaminergic- motivation, pleasure, cognition, memory, learning, and fine motor control, as well as
modulation of neuroendocrine signaling.
o Most common adrenergic drugs
Alpha 1- for vasoconstriction
Norepinephrine and dopamine
Beta 1- for heart rate (because of epinephrine)
Dopamine, dobutamine, norepinephrine, and epinephrine
o Indications
24 | P h a r m a c o l o g y
Epi only has an effect for 5 minutes; does not have an overdose; used for cardiac arrest others are for
blood pressure
Cardiac arrest
Anaphylaxis
Superficial bleeding
Norepinephrine
used for hypotension
Treatment of cardiovascular problems
shock
Dopamine
Shock
Hypotension
Bradycardia
Dobutamine
Cardiogenic shock
Heart failure
Antihypertensive, diuretics, anticoagulants, and dyslipidemics
Hypertension
87| P h a r m a c o l o g y

o Common in the US, up to 60 million afflicted


o Leads to MI, heart failure, stroke, and renal disease
MI- cessation of blood supply to the heart, necrosis
Heart failure- complication of MI; contractility of the heart
o Strong correlation with metabolic syndrome
Related to stress, lack of sleep
More on weight gain
Types of hypertension
o Essential or primary
Etiology/ cause: unknown
Contributors: include- salt sensitivity, insulin resistance, genetics (most common factor), sleep apnea,
environmental factors, etc.
o Secondary- renal, adrenal, coarctation of the aorta (dilation of the aorta), steroids, and pregnancy
Etiology: related to another disease condition/ underlying disease condition
BP review
o Any condition that affects the heart rate, stroke volume, or peripheral vascular resistance affects
arterial blood pressure
o Compensatory mechanisms to maintain balance between hypotension and hypertension
Normally, when the arterial blood pressure is elevated:
o Kidneys will excrete more fluid
Alpha I- BP
Alpha II
Receptors related to sympathetic= SA node and peripheral vascular resistance
-70 millivolts resting phase of SA node
Sodium goes to cell= increase conductivity to 70 MV
o Fluid loss will result in decreases extracellular fluid volume and blood volume
Blood volume- the more the blood volume, the higher the BP
o Decreased blood flow to the heart will reduce cardiac output
Renin-angiotensin-aldosterone system
o Decreased CO reduces arterial blood pressure
Repolarization of cells
Potassium will go out; calcium takes place here- has effect contractility- prolongs; affects also
muscles (contractions; smooth muscles= vasoconstricition)- prevented by blocking calcium
o Beta receptors
Beta I- heart
Beta II- heart and lungs
o Vascular endothelium produces vasodilating substances (nitric oxide and prostatacyclin) which
reduces blood pressure
Nonpharmacologic management of hypertension
25 | P h a r m a c o l o g y
o Weight reduction
o Exercise
o Salt restriction in diet
o Stress reduction
o Moderation in alcohol intake
o If systolic BP cannot be maintained <140 systolic, time to take treatment
Antihypertensive drugs
Classes:
88| P h a r m a c o l o g y

o Angiotensin converting enzyme (ACEI)


o Angiotensin II receptor blockers
o Antiadrenergic
o Calcium channel blockers
o Diuretics
o Direct vasodilators
o Renin inhibitors
Angiotensin converting enzyme I inhibitor
Blocks the enzyme that converts angiotensin I to angiotensin II (potent vasoconstrictor)
o Cannot totally block but only decrease
Have action of vasodilation and decrease in aldosterone production
Inhibit breakdown of bradykinins (vasodilator) prolonging effect
Reversed remodeling of heart muscle and blood vessels
Reno-protective
o Affects renin; also has a protection in the kidneys
Excellent for heart failure and hypertension
Improves post-myocardial infarction survival
Used alone or in combination
Useful in heart failure as decrease peripheral vascular resistance, cardiac workload and ventricular
remodeling
Captopril—prototype; most common
Low incidence of side effects
Can cause cough or hypotension when first started- normal and most common
Can cause hyperkalemia
Should never be used during pregnancy
May not be effective in African—americans- may add diuretic in this population to increase efficacy
Lotensin (benazepril)
Capoten (captopril)
Vasotec (enalapril)
Zestril (Lisinopril)
Altace (Ramipril)
Aceon (perindopril)
Classes of hypertension
Classes of hypertension
Medications
Class I (140-149)
ACE I inhibitor and diuretics
Class II (150-159)
Vasodilators and diuretics
Class III (160-169)
Clonidine, beta blockers, and calcium channel blockers
Class IV (>170)
Clonidine, beta blockers, and calcium channel blockers
Angiotensin II receptor blockers (ARBs)
More potent than ACE I
Blocks effects of angiotensin II, compete with angiotensin II for tissue binding sites
Blocks the receptors in brain, kidneys, heart, vessels, and adrenal tissue
26 | P h a r m a c o l o g y
89| P h a r m a c o l o g y

Has less side effects


Most common side effect: orthostatic hypotension
o Teach to slowly change position
SARTAN- end name of ACE II blockers
Similar end results as seen with ACE I
Less likely to cause hyperkalemia
Persistence of cough is rare
Prototype is Cozaar (losartan)
Atacand (candesartan)
Cozaar (losartan)
Diovan (valsartan)
o Recalled because it may cause cancer
o Because of rocket fuel contents- preservatives
Micardia (telmisartan)
Benicar (olmesartan)
Antiadrenergic
inhibit the activity of the sympathetic nervous system
blocking alpha receptors- epinephrine and norepinephrine
effective in decreasing heart rate, force of myocardial contraction, cardiac output, and blood pressure
short acting has a lot of side effects
Alpha 1 adrenergics receptor blocking agents dilate vessels and decrease peripheral vascular
resistance
Can experience first dose phenomenon with orthostatic hypotension, dizziness, syncope (can cause
head trauma, temporary loss of consciousness caused by a fall in blood pressure), possible sodium
and fluid retention
Antiadrenergics – Alpha 1
ZOSIN- end name of alpha-1
Blocks alpha I and causes vasodilation
Cardura- (doxazosin)
Minipress (prazosin)
Hytrin (terazosin)
Alpha 2 antagonists
Centrally acting sympatholytics stimulate presynaptic alpha 2 receptors in the brain
Less norepinephrine is released and sympathetic outflow is reduced
Results in decrease cardiac output, heart rate, peripheral vascular resistance, and blood pressure
Vasoconstriction and has little effect on the heart rate
Centrally acting agents also can result in fluid and sodium retention
Catapres (clonidine)- orally or by patch
o Decrease BP; used in the community as emergency medication for sudden findings of hypertension
o Short acting
o > 180- hypertensive emergency
Tenex (guanfacine)
Aldomet (methyldopa)
Beta adrenergic blockers
Decrease heart rate, force of myocardial contraction, cardiac ouput and renin release from the
kidneys
27 | P h a r m a c o l o g y
90| P h a r m a c o l o g y

Drug of choice for patients with tachycardia, angina, MI, left ventricular hypertrophy, and high renin
hypertension
Technically, it blocks beta 1
Beta 1- heart rate
o Can lead to exhaustion and myocardial death
Beta 2- heart and lungs
Angina- heavy crashing pain; sign of early possibility of MI
Pregnancy- teratogenic C and D (most is the phenytoin A)
Beta blockers I
End name OLOL
Inderal (propranolol)- hyperthyroid
o Increase HR due to T3 and T4
Inderal (propranolol)
Corgard (nadolol)
Lopressor (metoprotol)
Tenormin (atenolol)
Kerlone (betaxolol)- for glaucoma (ophthalmic)
o Hypertension- orally
IV-short acting
Oral- maintenance
If not selective- bronchoconstriction or asthma may occur
o If so, take ACE II to prevent side effects
Calcium channel blocking agents
Calcium effects is more on electrolyes= has effects on the heart rate itself
CALCIUM CHANNEL BLOCKER PICTURE
End name: Dipine- vasodilation
Verap and diltiazem (SVT supraventricular tachy- >150)
o For emergency cases
Norvasc (amlodipine)- most common
Cardizem (diltiazem)
Plendil (felodipine)
Procardia (nifedepine)
Calan (verapamil)- may cause gingival hyperplasia
These are also pregnancy category C teratogenic
Useful in hypertension as it dilates peripheral arteries and decrease peripheral vascular resistance by
relaxing vascular smooth muscle
Monotherapy or in combination
Tolerated well even with renal failure
28 | P h a r m a c o l o g y
Vasodilators
Limited effect when used alone
Cannot be used alone or as monotherapy
o If vasodilator only, there is rebound effect
Vasodilating action that lowers BP also stimulates SNS. This in turn, triggers reflexive compensatory
mechanisms that raise BP
Corlopam (felodapam)- IV infusion, in hypertensive emergencies avoid in patients with allergies to
sulfites
Apresoline (hydralazine)- IV, IM, or PO
91| P h a r m a c o l o g y

o Can cause orthostatic hypotension


o Most common vasodilator
In 2 minutes decrease in BP; 30 minutes effects will subside
Rogaine (minoxidil)- PO or topical
Hypertensive emergencies
> 180 systolic
> 120 diastolic
Is defined as having end organ damage or diastolic BP of 120 Hg or higher
With oral medications, use captopril 25-50 mg PO every 1-2 hours or clonidine 0.2 initially then 0.1 mg
every
hour until diastolic blood pressure falls below 100 Hg or if 0.7 mg has been given
Can cause stroke, MI, kidney damage
Nitroglycerine- antianginal medication
o Taken as a vasodilator
o Chest pains
o Sublingual- will alleviate pain
o In some consequences may decrease BP
o Emergency medication for angina
o Only lasts for 30 days- expiration
MAP- main arteria pressure
o Systolic (diastolic x 2) / 3
o Perfusion- < 60 low perfusion
Multiorgan damage
o > 120- hypertensive emergency
Herbals that affect the BP Ephedra (ma huang)- medicinal preparation from the plant Ephedra sinica
Ginseng- root of plants in the genus Panax Yohimbe (for erectile dysfunction)- dietary supplement
made from the bark of an African evergreen tree
Caffeine- most common
Other information
Viagra- vasodilation of pulmonary arteries
o Do not take beta blockers= lead to hypotension
Check VS before giving
o < 60 bpm do not give beta blockers
Cut off of BP- 100 systolic; antihypertensives should not be given
Diuretics
Indicated for the treatment of edematous and non edematous conditions
o pulmonary edema
Only medication that decreases blood volume
May be useful in preventing renal failure by sustaining urine flow
A minimum of daily urine output approximately 400 ml is required to remove normal amount of
metabolic end products
o Polyuria- > 4L
o Oliguria- < 400-500 mL
o Anuria- < 100 mL; end stage renal disease
29 | P h a r m a c o l o g y
Acts on the kidneys to decrease absorption of sodium, chloride, water, and other substances such as
calcium
o Actions is in the nephrons
92| P h a r m a c o l o g y

Major subclasses- thiazides, osmotic diuretic, loop, and potassium-sparing diuretics


Each act at different sites of the nephron
o Thiazides- affect is in distal tubule
o Loop- affects the ascending loop of Henle
o Potassium- sparing; quick diuretic (collecting; already urine form and distal tubule)
Used to manage
o Edema and ascites
o Management of heart failure- decrease fluids and water
o hypertension
Thiazide diuretics
End name- Thiazide
Chemically related to sulfonamides so caution to patients with sulfa allergies
Used in long term management of heart failure and hypertension
Affect distal convoluted tubule
Effectiveness decreases as the GFR decreases
o GFR output should be 30 mL/ hour
o If less than 30 mL/hour it becomes ineffective
o As rising creatinine is noted, alternatives should be used such as loop diuretic
Contraicdicated to px with renal damage or failure
Can cause hypokalemia= leads to heart attack
o Potassium most sensitive electrolyte for the heart
Diuril (cholorothiazide)
Hygroton (chlorthalidone)
Hydrodiuril (hydrochlorthiazide)
Loop diuretics
Inhibit sodium and chloride reabsorption in the ascending limb of the loop of Henle
Potent diuretic
Most common diuretic
Need to restrict dietary sodium when taking these medications
Lasix (Furosemide)- prototype and most common; if not hypokalemic
o But if hypokalemic- potassium-sparing diuretic
o Given with px with pulmonary edema
o Potent diuretic
o Only work with px without kidney problems
o Excessive fluids in the lungs (crackles etc.)
Bumex (bumetamide)- more potent than Lasix
Oral- more on maintenance; treatment for hypertension; IV form- diuresis
Ototoxic
Max dose 120 mg/day
Given slow, because If rapid= deafness
Must monitor potassium levels, intake and output, and also weight
o 80% of potassium is excreted in the urine= lead to hypokalemia
Potassium-sparing diuretics
Act at distal tubule to decrease reabsorption of sodium and potassium excretion
Spironolactone- prototype, blocks sodium retaining effects of aldosterone
o Conserve potassium; excrete sodium
o Where sodium goes water follows
Weak diuretic when used alone, often used in combination
93| P h a r m a c o l o g y

Contraindicated in renal failure


Osmotic diuretics
Produce rapid diuresis by increasing the solute load of the glomerular filtrate
Water is pulled into the intravascular space and excreted via kidneys
Useful in managing oliguria and anuria
Can prevent acute renal failure during prolonged surgery, trauma, or during chemotherapy
Helps reduce increased ICP, reduction of intraocular pressure before certain ophthalmic surgery and
for urinary excretion of toxic substances
30 | P h a r m a c o l o g y
o Stroke= increased intracranial pressure
Effect pull fluids IV space
Crystal form
Not used for oliguria and anuria
Osmitrol (mannitol)- most common osmotic diuretic
o For increased ICP
o Also for blood transfusion reactions
o Should be given fast drip- because it is a solute load; 50, 75, 100 mL Q4 large gauges of IV
cannula; because it precipitates
o Put inwarm water to dissolve
Ismotic (isosorbide), and osmoglyn (glycerin)
PHARMACOLOGY REVIEWER FOR FINALS
Anticoagulants
Drugs that affect coagulation
Thrombosis may occur in both arteries and veins
Arterial thrombi cause disease by obstructing blood flow which can result in tissue ischemia or death
o Coronary, cerebral, carotid, and the aorta
o Smoking can cause peripheral arterial disease
Venous thrombosis is associated with venous stasis. A venous thrombus is less cohesive than an
arterial embolus so no venous emboli is more prevalent
o Common to people with sedentary lifestyle, no exercise, people with varicose veins, and common
with long flights
Hemostasis
Hemostasis is the prevention or stoppage of blood loss from an injured blood vessel. Process
Involves vasoconstriction, formation of a platelet plug (platelet aggregation), activation of clotting
factors and growth of fibrous tissue into the blood clot (fibrin) making it more stable
o Localized vasoconstriction is usually in the capillaries
o When blood is exposed to air it will start to clot
Clot lysis
When clot is being formed, plasminogen is bound to fibrin and becomes a component of a clot
After tear in blood vessel is repaired, plasminogen is activated by plasminogen activator to produce
plasmin. Plasmin (enzyme) breaks down the fibrin mesh and dissolves the clot
o Fibrin mesh is in the capillary
o Fibrin is broken down then is reabsorbed in the body
Blood coagulation factors
I- fibrinogen
II- prothrombin
III- thromboplastin
IV- calcium
94| P h a r m a c o l o g y

V- labile factor
VI-
VII- proconvertin or stable factor
VIII- antihemophilic factor
IX- Christmas factor
X- stuart factor
XI- plasma thromboplastin antecedent
XII- Hageman factor
XIII- fibrin-stabilizing factor
Clotting factors can be found in the plasma
For patients with bleeding- give fresh frozen plasma
Anticoagulants
Given to prevent formation of new clots and extension of clots already present; does not dissolve clots
that are already present
Prototype: Heparin
Heparin may be used during pregnancy
Used during hemodialysis
Heparin is used with IU
31 | P h a r m a c o l o g y
Weight is based dosing per international nomogram
Differences
Heparin
Warfarin
Route
IV or SQ
Oral
Indications
Acute/ immediate cases
Long-term cases (2 weeks
); maintenance
Antidote
Protamine sulfate
Vitamin K
Monitoring
aPTT
PT
Heparin
Usually given a loading dose then calculated dosage per 24h; e.g. 15-25 units/kg/hr for IV dosing
Can be given IV, SQ, or continuous infusion; cannot be given oral or IM
Contraindication of heparin: if patient has presentation of bleeding e.g. GI bleeding
To know therapeutic effect of heparin- monitor activated partial thromboplastin time (aPTT)
o Should be 1.5-2.5 control value—normal value is 35 seconds
HIT- Heparin induced thrombocytopenia
o Can cause low platelet count
o If patient has low platelet count check if it can be given
o Normal platelet count: 150,000-400,000
o < 100,000 is considered as low platelet count
Antidote: Protamine sulfate
95| P h a r m a c o l o g y

Combines with antithrombin III which will inactivate clotting factors IX, X, XI, and XII, inhibits
conversion of prothrombin to thrombin and prevents thrombus formation. Further affects coagulation
by preventing conversion of fibrinogen to fibrin, inhibiting factors V, VIII, XIII, and platelet aggregation
INR checked Q6
Low molecular weight heparins
Given subcutaneously in abdomen and do not require close blood monitoring of blood coagulation
tests
Still should follow platelet counts
Fragmin (dalteparin)
Lovenox (enoxaparin); clexane- name used in the Philippines
o Lesser chance of heparin-induced thrombocytopenia
Coumadin (Warfarin)
Most commonly used as oral anticoagulant
Drug acts in the liver to prevent synthesis of vitamin K- dependent clotting factors (II, VI, IX, X). Acts
as competitive antagonist to hepatic use of vitamin K
Anticoagulant choice for long-term maintenance therapy
Can be combined with heparin
International normalized ratio (INR) is based on prothrombin time
o Normal baseline or control of PT is 12 seconds; therapeutic value is 1.5 times the control or 18
seconds
Therapeutic effect will occur in two weeks
o Check using prothrombin time (PT)
INR should be checked weekly
o INR- normal value: 1
o INR has eliminated disparities in different labs
Dosage reduction in patients with biliary tract disorders, liver disease, malabsorption syndrome, and
hyperthyroidism.
o Conditions increase anticoagulant drug effects by reducing absorption of vitamin K or decreasing
hepatic synthesis of clotting factors
Has multiple drug interactions
Counteract with vitamin K
When to combine heparin and warfarin
Give heparin until stable or up until patient is still admitted in the hospital; then at discharge give
maintenance of warfarin= once it already has effect discontinue heparin
32 | P h a r m a c o l o g y
Other anticoagulants
Orgaran (danaparoid)- low molecular weight, heparin-like drug. Given subcutaneously.
o Used in management of hip surgery, ischemic stroke or in those who cannot take heparin.
o Does not contain heparin
Refludan (lepirudin)- used as heparin substitutes
Arixtra (fondaparinux)- binds to clot bound factor Xa, inhibits thrombin productions
o given subcutaneously
Argatroban
Angiomax (bivalirudin)
Antiplatelet aggregate drugs
Arterial thrombi are composed of primary platelets
Antiplatelet drugs act by inhibiting platelet activation, adhesion, aggregation, or procoagulant activity
96| P h a r m a c o l o g y

Include drugs that block platelet receptors for thromboxane, adenosine diphosphate, glycoprotein
IIIa/IIb, and phosphodiesterase inhibitors
Prevents clumping of platelets
Blood thinners
Weaker compared to warfarin and heparin
Used as maintenance
Mostly oral tablets
Thromboxane A2 inhibitors
Work by inhibiting synthesis of prostaglandins. TA inhibitors work by acetylating cyclooxygenase, the
enzyme in platelets that synthesizes thromboxane A2 (which causes platelet aggregation)
ASA- acetyl salicylic acid; aspirin, is an example. It affects the platelets during its life
o Total life of a platelet- 9-10 days
NSAIDs are not so useful as action wears off as drug wears off
Adenosine diphosphate receptor antagonist
Ticid (ticlodipine)
Inhibit platelet aggregation by preventing ADP-induced binding between platelets and fibrinogens.
This reaction inhibits platelet aggregation irreversibly and persist for the lifespan of the platelet
Indicated for TIA- transient ischemic attack
o Blockage in the carotid artery
o A sign of stroke for the next 5 years
Adverse effects: neutropenia, diarrhea, and skin rashes
o Neutropenia- 65-80% low neutrophils
Even used for post angioplasty
Taken for 1 year only- due to price
Plavix (clopidogrel)
o Has fewer side effects than ASA or ticlid
o Indicated for patients with atherosclerosis for reduction of myocardial infarction, stroke, and vascular
death
o Does not need reduction in those with renal problems
Glycoprotein IIb/IIIa receptor antagonist
Reopro (abciximab)- is a monoclonal antibody that prevents binding of fibrinogen, von willerbrand
factor, and other molecules to GP IIb/IIIa receptors on activated platelets. This action inhibits platelet
aggregation
o Given during angioplasty
Used with percutaneous transluminal coronary angioplasty (PTCA) or removal of atherosclerotic
plaque to prevent rethrombosis
Used with aspirin and heparin and is contraindicated in clients who have recently received oral
anticoagulants or IV dextran
Other contraindications include active bleeding, thrombocytopenia, history of stroke, surgery or
trauma within the past 6 weeks, uncontrolled hypertension, or hypersensitivity
Other glycoprotein IIb/IIIa receptor antagonist
Integrilin (Eptifibatide)- only given for 24 hours after angioplasty
o Potent antiplatelet aggregate
Aggrasat (tirofiban)
Similar mechanism to the action of reopro
33 | P h a r m a c o l o g y
Indicated for acute coronary syndrome who are managed medically or by angioplasty or atherectomy
Contraindications include
97| P h a r m a c o l o g y

o Recent bleeding
o History of thrombocytopenia
o History of stroke within 30 days
o Major surgery or severe trauma within the past month
o Severe hypertension
o History of intracranial hemorrhage
o Neoplasm
o AV malformation
o Aneurysm
o Platelet count < 100,000
o Creatinine greater than 2 mg/dl
Phosphodiesterase inhibitor
Pletal (cilostazol)- increases cAMP which then inhibits platelet aggregation and produces vasodilation.
Drug reversibly inhibits platelet aggregation
Indicated for intermittent claudication
Indicated for peripheral vascular disease
o Buerger’s disease
o Reynaud’s syndrome
Contraindicated for patients with heart failure
Most common side effect is diarrhea and headache
Miscellanous
Persantine (dipyridamole)- inhibits platelets aggregation but mechanism is unclear
Used for prevention of thromboembolism after cardiac valve replacement and is given with Coumadin
Thrombolytics
Given to dissolve thrombi
Stimulate conversion of plasminogen to plasmin, an enzyme that breaks down fibrin (the framework of
a thrombus)
Used in severe thromboembolic diseases such as myocardial infarction, PE and ileofemoral
thrombosis
Goal is to re-establish blood flow and prevent tissue damage
Also used to dissolve clots in arterial or venous cannulas or catheters
Must obtain baseline INR, aPTT, platelet count, and fibrinogen
Will monitor tabs 2-3 hours after thrombolytic therapy is instituted to determine efficacy
Activase (alteplase), Retavase (reteplase), and TNKase (tenecteplase) are tissue plasminogen
activators
o Mainly used in acute MI to dissolve clots
Eminase (anistreplase), Streptase (streptokinase), and Abbokinase (urokinase) are enzymes that
break down fibrin
o Used to lyse coronary clots in acute MI
o Streptokinase- problem allergy; byproduct of streptococcus
o Can cause anarrhytmia
Xigris (drotrecogin alfa) is a recombinant version of human activate protein C
o Approved for use in sepsis
o Sepsis causes inappropriate blood clot formation and may lead to DIC
o DIC- last part of sepsis
o Blood clots due to consumption of platelets
o experimental medication
Strongest type of anticoagulants
98| P h a r m a c o l o g y

Used to dissolve clots


Last option
Heart attack- 6 hours timeframe for thrombolytics
Stroke- 4 hours
Drugs used to control bleeding
Amicar (aminocaproic acid) and Cyklokapron (tranaxemic acid) are used to stop bleeding caused by
thrombolytic agents
o Tranexamic acid- used for trauma
Trasylol (aprotinin) indicated in patients undergoing CABG
o Inhibits breakdown of blood clotting factors
34 | P h a r m a c o l o g y
Drugs affecting the cardiovascular system
ANTILIPIDS
Important Reminders
Risk factors for Thromboembolism
1. Obesity
2. Myocardial infarction
3. Atrial Fibrillation
4. Prosthetic heart valves
5. Arteriosclerotic heart disease
6. OCP or HRT
7. History of deep vein thrombosis or pulmonary embolism
8. Cigarette smoking
9. Immobility
Overview of Cholesterol Panel
Total Cholesterol
Desirable – Less than 200
Borderline high – 200 - 239
High – 240 or greater
Low-density lipoprotein
Desired – <100
Above optimal – 100 – 139
Borderline high – 130 – 159
High – 160 – 189
Very high – 190
High-density lipoprotein
High - >60
Low - <40
Triglycerides
Normal – Less than 150 – 199
High – 200 to 499
Very high – 500 or above
DYSLIPIDEMIA
Associated with atherosclerosis and numerous pathophysiologic effects
Elevated total cholesterol, high HDL and low HDL are all risk factors for CAD
Triglycerides indicated excessive caloric intake, excessive proteins and carbohydrates are converted
to TG and obesity
Atherosclerosis is narrowing of arteries due to plaques
99| P h a r m a c o l o g y

CAD most risky at 40 due to CHO / plaque starts from infancy – Accumulation, checked through
angiogram
Hypothyroidism – decrease in T3 and T4. Slow speak and slow thinking, edema
Contributors to Dyslipidemia
Hypothyroidism
Diabetes mellitus
Alcoholism
Obesity, beta blockers, oral estrogen, glucocorticoids, sertraline, thiazide, diuretics, protease inhibitors
High dietary intake also increases the conversion of VLDL to LDL cholesterol, and high dietary intake
of TG and saturated fat decreases the activity of LDL receptors and increases synthesis of
cholesterol.
Types of Lipoproteins
35 | P h a r m a c o l o g y
LDL – Unfavorable type. Transports 75% of serum cholesterol to peripheral tissues and the liver. High
levels are atherogenic
VLDL – Contains 75% TG and 25% cholesterol. Transports endogenous TG to fat and muscle cells
HDL – Favorable type. This LP transports cholesterol back to the liver to catabolism and excretion
Initial Management
Treat conditions that contribute to elevated lipids (DM, Hypothyroidism)
Start low fat diet
Increase intake of fiber – lowers LDL
Cholestetol lowering margarines
Weight Reduction
Exercise – Increase HDL
Smoking cessation
HRT (Hormonal Replacement Therapy)
Drug Therapy
Based on the type of dyslipidemia and its severity
Classes of agents include: HMB-CoA reductase inhibitors or “Statins”, fibrates, bile acid sequestrants
and niacin in different forms
Lovaza
Zetia
HMG-CoA reductase inhibitors or “Statins”
Inhibits an enzyme (Hydroxymethylglutaryl-coenzyme A reductase) required for hepatic synthesis of
cholesterol
Decrease serum cholesterol, LDL, VLDL and TG
Reach maximal effects within about 6 weeks
Drugs also reduce C reactive protein associated with inflammation and development of coronary
artery disease
Undergo extensive first pass metabolism
Metabolism occurs in liver
Adverse effects include: HA, diarrhea, rashes, headaches, constipation, hepatotoxicity and myopathy
o Reason to stop statins is because of myopathy= rhabdomyolysis (serious syndrome due to a direct
or indirect muscle injury. It results from the death of muscle fibers and release of their contents into
the bloodstream.)= due to secretion of creatinine kinase to the bloodstream= stocked in kidneys=
renal failure
Should obtain baseline LFTs and then at 6 and 12 weeks after starting then every 6 months
If serum aminotransferases increase to more than 3x normal, should be reduced or DCed
100| P h a r m a c o l o g y

Do not take with grapefruit juice


Pregnancy category X
Examples: Lipitor (Atorvastatin), Pravachol (pravastatin), Zocor (Simvastatin), Lescol (Fluvastatin)
Rosuvastatin- Another common statin
Bile Acid Sequestrants
Bind bile acids in the intestinal lumen. This causes the bile acids to be excreted in the feces and
prevents their being recirculates to the liver. Thus, the liver will use cholesterol to produce bile acids
thus decreasing serum levels
Especially lower LDL
Examples: Questran (Cholestyramine)
and Welchol (Colesevelam)
Often used with patients already on a statin
Long term use can affect absorption of folate and vitamins A, D, E, K
If feces turns white= pancreatitis or liver problem
FIBRATES
Tricor (Fenofibrate)
Lopid (Gemfibrozil)
These drugs increase oxidation of fatty acids in liver and muscle tissue thus decreasing hepatic
production of TG, VLDL, and increase HDL
Most effective drugs for reducing TG
can cause hepatotoxicity
Main side effects include: diarrhea, GI discomfort, cause gallstones, interact with Coumadin
NIACIN (Nicotinic Acid)
Decreases both cholesterol and triglycerides
36 | P h a r m a c o l o g y
Bottom line – decreases hepatic synthesis of TG and secretion of VLDL (which leads to decreased
production of LDL)
Need high doses for efficacy
Side effects: Flushing, pruritus, gastric irritation. May cause hyperglycemia, hyperuricemia, elevated
hepatic aminotransferase enzymes and hepatitis
Can reduce flushing by starting with low doses, taking dose with meals, and taking ASA 325mg thirty
minutes before taking dose
More effective in preventing heart diseases when used in combination with another dyslipidemic such
as a bile acid sequestrant or fibrate.
Drugs affecting the respiratory system
Key terms
Ventilation
Perfusion
Diffusion
Pulmonary circulation
Surfactant
Pneumocytes
drugs for asthma and other bronchoconstrictive disorders
Asthma- inflammation, hyperreactivity, and bronchoconstriction
Gastroesphageal reflux disease (GERD) may cause microaspiration/resultant nighttime cough
___- medications can also exacerbate GERD
Asthma
May be triggered by __, __, and allergies
101| P h a r m a c o l o g y

Can develop at any age


Seen more often in children who are exposed to allergens during infancy
Bronchoconstriction
Inflammation
Mucosal edema
Excessive mucous
Pathophysiology of asthma
Mast cells
Chemical mediators such as histamine, prostaglandins, acetylcholine, cGMP, interleukins, and
leukotrienes are released when triggered. Mobilization of eosinophils. All cause movement of fluid and
proteins into tissues.
Bronchoconstrictive substances antagonized by cAMP
Chronic obstructive pulmonary disease (COPD)
Combination of chronic bronchitis and emphysema
Bronchoconstriction and inflammation are more contant, less reversibility
Anatomic and physiologic changes occur over the years
Leads to increasing dyspnea and activity intolerance
Drug therapy
Bronchodilators
Anti-inflammatories
Bronchodilators
Adrenergics- stimulate beta II receptors in smooth muscles of bronchi and bronchioles
Receptors stimulates cAMP= bronchodilation
Cardiac stimulation is an adverse effect of these medications
Bronchodilators- adrenergics
Cautious use in hypertension and cardiac disease
Selective beta II agonists by inhalation are drugs of choice
Epinephrine subcutaneous in acute bronchoconstriction
Short acting bronchodilators
Treatment of first choice to relive acute asthma
37 | P h a r m a c o l o g y
Aerosol or nebulization
May be given by MDI
Overuse will diminish their bronchodilating effects= tolerance
Proventil (albuterol)
Xopenex (levalbuterol)
Other bronchodilators
Foradil (formoterol) and serevent (salmeterol) are long acting beta II adrenergic agonists used only for
prophylaxis
Serevent (salemeterol)- used in deteriorating asthma can be life-threatening
Alupent (metaprotenerol)- intermediate acting. Useful in exercise induced asthma, treatment for acute
bronchospasm
Brethine (terbutaline)- selective beta II adrenergic agonist that is a long-acting bronchodilator
When give subcutaneous, loses selectivity
Also used to decrease premature uterine contractions during pregnancy
Xanthines
Theophylline
Mechanism of action is unclear
102| P h a r m a c o l o g y

Bronchodilate, inhibit pulmonary edema, increase action of cilia, strengthen diaphragmatic


contractions, over-all anti-inflammatory action
Increases CO, causes peripheral vasodilation, mild diuresis, and stimulates SNS
Contraindicated in acute gastritis and PUD
Second line
Narrow therapeutic window- therapeutic range is 5-15 mcg/mLh
Multiple drug interactions
Anticholinergics
Block the action of acetylcholine in bronchial smooth muscles when given by inhalation
Action reduces intracellular guanosine monophosphate (GMP) which is a bronchoconstrictive
substance
Atrovent (ipratropium)- caution in BPH, narrow-angle glaucoma
Spiriva (tiotropium)
Anti-inflammatory agents
Suppress inflammation by inhibiting movement of fluid and protein into tissues; migration and function
of neutrophils and eosinophils, synthesis of histamine in mast cells, and production if proinflammatory
substances
Benefits: decrease mucous secretions, decreases edema, and reduced reactivity
STEROIDS
Corticosteroids
Second action is to increase the number and sensitivity of beta 2 adrenergic receptors
Can be given PO or IV
Pulmonary function usually improves within 6-8 hours
Continue drugs for 7-10 days
Steroids
Fewer long term side effects if inhaled
End –stage COPD may become steroid dependent
In asthma, systemic steroids generally are used only temporarily
Taper high dose oral steroids to avoid hypothalamic-pituitary axis suppression
- For inhalation:
o Beclovent (beclomethasone)
o Pulmicor (budesonide)
o Aerobid (flunisolide)
o Flovent (fluticasone)
o Azmacort (triamcinolone)
Most inhaled steroids are being reformulated with HFA
HFA – hydrofluoroalkane
Systemic use: prednisone, methylprednisolone, and hydrocortisone
In acute, severe asthma – a systemic corticosteroid may be indicated when inhaled beta 2 agonists
are ineffective
38 | P h a r m a c o l o g y
MODIFIERS
Leukotriene Modifiers
Leukotrienes are strong chemical mediators of bronchoconstriction and inflammation
Increase mucous secretion and mucosal edema
Formed by the lipooxygenase pathway of arachidonic acid metabolism in response to cellular injury
Are released more slowly than histamine
Leukotriene Modifier Drugs
103| P h a r m a c o l o g y

Developed to counteract the effects of leukotriene


Indicated for long term treatment of asthma in adults and children
Prevent attack induced by some allergens, exercise, cold air, hyperventilation, irritants and
ASA/NSAIDs
Not useful in acute attacks
Leukotrience Modifiers
Injured cell
Arachidonic acid
XXXXXXXXX
Lipooxygenase
Leukotrienes
XXXXXXX
Bronchi, WBCs
Bronchoconstriction
Leukotriene Modifier Drugs
Singulair (montelukast) and Accolate (zafirlukast) are leukotriene receptor antagonists
Can be used in combination with bronchodilators and corticosteroids
Less effective than low doses of inhaled steroids
Should not be used during lactation
Can cause HA, nausea, diarrhea
Mast Cell Stabilizers
- Intal (cromolyn)
- Tilade (nedocromil)
Prevent release of bronchoconstrictive and inflammatory substances when mast cells are confronted
with allergens and other stimuli
Prophylaxis only
Inhalation, nebulizer or MDI, nasal spray as well
MDI – Metered-dose inhaler
ANTI-HISTAMINE AND ALLERGIES
Antihistamines and Allergic Disorders
Histamine is the first chemical mediator released in immune and inflammatory responses
Concentrated in skin, mucosal, surfaces of eyes, nose, lungs, CNS and GI tract
Located in mast cells and basophils
Interacts with histamine receptors on target organs called H1 and H2
Antihistamines
H1 receptors are located mainly on smooth muscle cells in blood vessels and the respiratory and GI
tracts
H1 binding causes: pruritus, flushing, increased mucous production, increased permeability of veins –
edema, contraction of smooth muscle in bronchi>>bronchoconstriction and cough
With H2 receptor stimulation, main effects are increased secretion of gastric acid and pepsin,
decreased immunologic and proinflammatory reactions, increased rate and force of myocardial
contraction
Allergic reactions
These are the exaggerated response by the immune system that produce tissue injury and possible
serious disease
Allergic reactions may result from specific antibodies, sensitized T lymphocytes, or both, formed
during exposure to an antigen
TYPES OF RESPONSES TO CELL-MEDIATED INVASION
104| P h a r m a c o l o g y

Type 1: Immediate hypersensitivity (IgE induced) response triggered by the interaction of antigen with
antigen-specific IgE bound on mast cells
Anaphylaxis is an example
39 | P h a r m a c o l o g y
Does not occur on first exposure to an antigen
Can develop profound vasodilation resulting in hypotension, laryngeal edema, bronchoconstriction
Type II: (IgM or IgM) immunoglobulin mediated which generate direct damage to cell surfaces.
Example: BT reactions, hemolytic disease of newborns, hypersensitivity reactions to drugs such as
heparin or penicillin.
Type III: IgM or IgM medicated reaction characterized by formation of antigen-antibody complexes
that induce inflammatory reaction in tissues.
Prototype: Serum sickness
Immune response can occur following antitoxin administration or sulfa drugs
Type IV hypersensitivity
Delayed hypersensitivity
Cell mediated response where sensitized T lymphocytes react with an antigen to cause an
inflammation, release of lymphokines, direct cytotoxicity or both
Classic examples: tuberculin test, contact dermatitis and some graft rejections
Allergic Rhinitis
IgE mediated
Inflammation of nasal mucosa caused by a hypersensitivity reaction to inhaled allergies
Presents with itching of throat, eyes and ears
Seasonal and perennial
Can lead to chronic fatigue, difficulty sleeping, sinus infections, postnasal drip, cough and headache
Intranasal Drugs for Allergic Rhinitis
Atrovent nasal spray
Beconase (beclomethasone)
Rhinocort (budesonide)
Flonase (fluticasone)
Nasonex (mometasone)
Nasalcrom (a mast cell stabilizer)
SKIN ALLERGIES
Allergic Contact Dermatitis
Type IV (hypersensitivty reaction)
Usually occurs >24 hour after reexposure
o Example: Poison Ivy
Other reactions
Allergic food reaction: result from ingestion of a protein
Most common food allergy is shellfish, others include milk, eggs, peanuts
Allergic drug reaction: unpredictable, may occur 7-10 days after initial exposure
Pseudoallergic drug reactions: resemble immune response but do not produce antibodies, i.e.
anaphylactoid
ANTIHISTAMINES
Inhibit smooth muscle constriction in blood vessels and respiratory tract and GI tract
decrease capillary permeability
decrease salivation and tear formation
act by binding with the histamine receptor
Indications for use:
105| P h a r m a c o l o g y

allergic rhinitis
anaphylaxis
allergic conjunctivitis
drug allergies
transfusion of blood products
dermatologic conditions
nonallergic such as motion sickness, nausea, vomiting, and sleep
Precautions
40 | P h a r m a c o l o g y
Caution in pregnancy
Benign Prostatic H
bladder neck obstruction
narrow angle glaucoma
1ST GEN. H1 RECEPTORS ANTAGONISTS
Bind to central and peripheral receptors
Can cause CNS depression or stimulation
Have substantial cholinergic effects
EXAMPLES:
Chlor-Trimeton (chlorpheniramine)
Benadryl (dipenhydramine)
vistaril (hydroxyzine)
phenergan (promethazine)
2ND GEN. H1 RECEPTORS ANTAGONISTS
Selective or nonsedating
Does not cross BBB
EXAMPLES:
astelin (azelastine)
allegra (fexofenadine)
claritin (loratadine)
clarinex (desloratadine)
zyrtec
Xyzal
NASAL DECONGESTANTS
Relieve nasal obstruction and discharge
Adrenergic
Rebound nasal swelling called "rhinitis medicamentosa"
EXAMPLES:
- Afrin
- Sudafed (pseudoephedrine)
- contraindicated in severe hypertension, CAD, narrow angle glaucoma, TCAs, MAOIs
ANTITUSSIVES
Suppress cough by depressing cough center in medulla or by increasing flow of saliva
For dry, hacking, nonproductive cough
not recommended for children and adolescents
EXAMPLES:
- codeine
- hydrocodone
- dextromethorphan
106| P h a r m a c o l o g y

EXPECTORANTS
Liquefy respiratory secretions
- Guiafenesin
MUCOLYTICS
By inhalation to liquefy mucous
EXAMPLE:
- mucomyst (acetylcysteine)
- mucosolvan (ambroxol)
may be used in treating acetaminophen overdose
COLD REMEDIES
41 | P h a r m a c o l o g y
Contain antihistamine, decongestant, and an analgesic
Examples:
- chlorpheniramine
- Pseudoephedrine
- acetaminophen
- dextromethorphan
- guiafenesin
decongestants can cause stasis of secretions
Tamiflu can be used to limit spread of virus in respiratory tract
DRUGS AFFECTING THE RESPIRATORY SYTEM
KEY TERMS
Ventilation
Perfusion
Diffusion
Pulmonary Circulation
Surfactant
Pneumocytes
Drugs for Asthma and other Bronchoconstrictive Disorders
Asthma – Inflammation, hyperreactivity, and bronchoconstriction
GERD – May cause microaspiration / resultant night time cough
Antiasthma – Medications can also excacerbate GERD
ASTHMA
May be triggered by Viruses, irritants, allergens.
Can develop at any age
Seen more often in children who are exposed to airway irritants during infancy
Bronchoconstriction
Inflammation
Mucosal edema
Excessive mucuos
Pathophysiology of Asthma
Mast cells
Chemical mediators such as Histamine, prostaglandins, acetylcholine, cGMP, interleukins,
leukotrienes are released when triggered. Mobilization of eosinophils. All cause movement of fluid and
proteins into tissues
Bronchoconstrictive substances antagonized by cAMP
Chronic Obstructive Pulmonary Disease (COPD)
Combination of chronic bronchitis and emphysema
107| P h a r m a c o l o g y

Bronchoconstriction and inflammation are more contant, less reversibility


Anatomic and physiologic changes occur over the years
Leads to increasing dyspnea and activity intolerance
Drug therapy
Bronchodilators
Anti-inflammatories
Bronchodilators
Adrenergics- stimulate beta II receptors in smooth muscles of bronchi and bronchioles
Receptors stimulates cAMP= bronchodilation
Cardiac stimulation is an adverse effect of these medications
Bronchodilators- Adrenergics
Cautious use in hypertension and cardiac disease
Selective beta II agonists by inhalation are drugs of choice
Epinephrine subcutaneous in acute bronchoconstriction
Short Acting Bronchodilators
42 | P h a r m a c o l o g y
Treatment of first choice to relive acute asthma
Aerosol or nebulization
May be given by MDI
Overuse will diminish their bronchodilating effects= tolerance
Proventil (albuterol)
Xopenex (levalbuterol)
Other bronchodilators
Foradil (formoterol) and serevent (salmeterol) are long acting beta II adrenergic agonists used only for
prophylaxis
Serevent (salemeterol)- used in deteriorating asthma can be life-threatening
Alupent (metaprotenerol)- intermediate acting. Useful in exercise induced asthma, treatment for acute
bronchospasm
Brethine (terbutaline)- selective beta II adrenergic agonist that is a long-acting bronchodilator
When give subcutaneous, loses selectivity
Also used to decrease premature uterine contractions during pregnancy
Xanthines
Theophylline
Mechanism of action is unclear
Bronchodilate, inhibit pulmonary edema, increase action of cilia, strengthen diaphragmatic
contractions, over-all anti-inflammatory action
Increases CO, causes peripheral vasodilation, mild diuresis, and stimulates SNS
Contraindicated in acute gastritis and PUD
Second line
Narrow therapeutic window- therapeutic range is 5-15 mcg/mLh
Multiple drug interactions
Anticholinergics
Block the action of acetylcholine in bronchial smooth muscles when given by inhalation
Action reduces intracellular guanosine monophosphate (GMP) which is a bronchoconstrictive
substance
Atrovent (ipratropium)- caution in BPH, narrow-angle glaucoma
Spiriva (tiotropium)
Anti-inflammatory agents
108| P h a r m a c o l o g y

Suppress inflammation by inhibiting movement of fluid and protein into tissues; migration and function
of neutrophils and eosinophils, synthesis of histamine in mast cells, and production if proinflammatory
substances
Benefits: decrease mucous secretions, decreases edema, and reduced reactivity
DRUGS ACTING ON THE ENDOCRINE
Control of Hormone Release
- Blood levels of hormones
- Are controlled by negative feedback systems
- Vary only within a narrow desirable range
- Hormones are synthesized and released in response to:
1. Humoral stimuli - Blood
2. Neural Stimuli
3. Hormonal Stimuli
PITUITARY AGENTS
Desmopressin (DDAVP)
- Classification: Antidiuretic, hemostatic
- Indications: Diabetes Insipidus
- Available forms:
- Intranasal Spray
- IV
- Tablet
- Drug of choice for Diabetes Insipidus
43 | P h a r m a c o l o g y
Diabetes Insipidus
- The problem is there is a decrease in Antidiuretic Hormone (ADH) causing Polyuria
- Urine output is increased to 7L-8L/Hour
- Patients die due to Dehydration
- To confirm diabetes insipidus an urinalysis test is needed
- Gravity <1.05 is not good
Action of Desmopressin
- Increase the flow of Adenosine Monophosphate and water through the kidneys, promoting
reabsorption of water and producing concentrated urine.
Nursing Process in Desmopressin
Assessment: Monitor effectiveness of therapy, be alert for Hypertension.
Nursing Diagnosis: Fluid Volume Excess
Planning and implementation:
1. Ensure nasal mucosa is intact
2. In cases of Rhinorrhea, report it to the doctor
3. Adjust the dose in coordination with the doctor In relation to the urine output of the patient
Patient Teachings:
- Instruct patient to clear nasal passages
- Tell patient to report nasal congestion, allergic rhinitis, and respiratory tract infection
- Rotate injection when using SQ form
- Warn patient to drink just enough water to satisfy the thirst
Vasopressin (Pitressin)
- Classification: Antidiuretic Hormone
- Indication:
- Diabetes Insipidus
109| P h a r m a c o l o g y

- Hypotension
- Pulseless arrest, Asystole
- Available forms: IV
- Last option for Inotropes
- Can be a substitute for epinephrine but only once
- More on hypotension and Cardiac arrest
- Adverse Reaction:
- Arrhythmia
- Myocardial ischemia
- Bronchoconstriction, Anaphylaxis
Nursing Process in Vasopressin
Assessment: Monitor the urine specific gravity, Watch out for hypertension, Closely monitor for signs
of water intoxication.
Nursing Diagnosis: Risk for Fluid Volume Deficit
Planning and Implementation:
1. Use minimum effective dose
2. In cardiac emergency, follow the guidelines in ACLs
3. Don’t confuse Vasopressin with Desmopressin
ADRENOCORTICAL AGENTS
Hydrocortisone (Solu Cortef)
44 | P h a r m a c o l o g y
Classification: Glucorticoids (steroids), Anti-inflammatory, immunosuppressant
Indication: Severe Inflammation, Septic shock
Available forms: Tablet, IV
Contraindication: GI Ulcers, Renal disease, Osteoperosis, DM
Adverse Effects: Seizures, Arrythmia, Pancreatitis, Peptic Ulceration
- Therapy must not exceed 21 days
- Hypokalemia may occur
Action of Hydrocortisone
- Not clearly defined
- May stabilize lysosomal membranes, suppress immune response, and stimulate bone marrow
Nursing Process in Hydrocortisone
Assessment: Monitor weight, Blood pressure, and Electrolytes level
Nursing Diagnosis: Risk for Infection
Planning and Implementation
1. Give oral dose with food
2. Give potassium supplements
3. Therapy must not exceed the 21 days
Patient Teachings
- Tell the patient not to abruptly stop taking hydrocortisone without the doctors content
- Warn patient about easy bruising
- Warn patient about Cushingcoid symptoms
Methylprednisolone (Medrol)
Classification: Glucocorticoids, Anti- Inflammatory, Immunosuppresant
Indication: Severe Inflammation, shock, multiple sclerosis, severe lupus nephritis
Available Forms: Tablet, IV
- The rest of the literatures are the same as hydrocortisone
ANTIDIABETIC AGENTS
110| P h a r m a c o l o g y

Pancreas
- Triangular gland behind the stomach
- Has both exocrine and endocrine cells
- Acinar cells (Exocrine) produce an enzyme-rich juice for digestion
- Pancreatic Islets (Islets of Langerhans) contain endocrine cells
- Alpha cells produce glucagon (a hyperglycemic hormone)
- Beta cells produce insulin (a hypoglycemic hormone)
Two Types of Diabetes
Type 1 Diabetes
- Disease case by lack of insulin
- Destroys beta cells therefore becoming autoimmune which causes a decrease in insulin and an
increase of blood sugar
- Oral medications cannot be used on Type 1 diabetes patients
Type 2 Diabetes
- Is a disease of insulin resistance by cells
- The most common type of diabetes
- Obesity
- Decrease in receptors of cells because fats covers the receptors
45 | P h a r m a c o l o g y
- Once the use of insulin starts it is used for lifetime
- Insulin is the last option
Insulin
Classification: Antidiabetic Agent
Indication: Moderate to severe diabetec Ketoacidosis, Control of Hyperglycemia, Hyperkalemia
Available Forms: IV, SQ
Common Types of Insulin
Rapid Acting
- Regular Insulin (Humulin R)
- 1 hour onset of action
- Appears to be clear
- Is used in acute cases (IV)
- Insulin Lispro
- Insulin Aspart
Intermediate Acting
- Isophane Insulin (Humulin N)
- Appears to be cloudy
- Onset of action is 4 hours
- Insuline Zinc (Lente)
Long Acting
- Insulin Glargine (Lantus)
- Is used as maintenance
- Insulin Determir (Levemir)
- You can mix Humulin R and Humulin N for better Longevity and Effectivity
- Humulin R is aspirated first before Humulin N to not transfer cloudiness to clear
- Give air to Humulin N first
Contraindication: HYPOGLYCEMIA
Adverse Reactions:
- Hypoglycemia
111| P h a r m a c o l o g y

- Urticaria
- Itching
- Anaphylaxis
- Lipodystrophy
- A problem in SQ fats where it appears to be depressed due to the repetition of using the same site
over and over or injecting cold insulin
- To prevent, rotate the injection site or change places. Insulin must be room temperature
Action of Insulin
- Increases glucose transport into the muscle and fat cell membranes to reduce glucose level
Nursing Process (Insulin)
Assessment:
46 | P h a r m a c o l o g y
- Assess patients glucose level before starting the therapy
- Monitor patient glycosylated haemoglobin regulary
- Monitor urine ketones when glucose level is elevated
Nursing Diagnosis: Risk for injury related to drug induced hypoglycaemia
Planning and Implementation:
- Dose is always expressed in units
- Don’t shake the insulin to prevent bubbles
- Rapid acting insulin should be given 15 minutes before meals
- Regular insulin can be mixed with NPH insulin in any proportions
- SQ route – 45 to 90 degree injection
- Don’t rub the site on injection
- Treat Hypoglycemia with rapid acting glucose or with Glucagon IV, IV Glucose
- If patient is awake you can give juice
Insulin Overdose – Antidote is Glucagon
Patient Teachings
- Tell patient that insulin relieves symptom but does not cure the disease
- Tell patient that glucose monitoring and urine ketone test are essential to dosage and success of
therapy
- Hypoglycemia is hazardous – Brain damage
- Smoking decreases absorption of insulin
- Advise the patient to carry or wear medical identification at all times
ORAL HYPOGLYCEMIC AGENTS
Oral Hypoglycemic Drugs
1. Sulfonylurea drugs
2. Meglinitides
3. Biguanides
4. Alpha-Glucosidase Inhibitors
5. Thiazolidinediones
6. Dipeptidyl peptidase-4 (DPP-4) Inhibitors
Insulin Secretagogues
- Sulfonylurea drugs
- Meglinitides
Insulin Sensitizers
- Biguanides
- Thiazolidinediones
Others
112| P h a r m a c o l o g y

- Alpha-Glucosidase Inhibotors
- Gastrointestinal hormones
INSULIN SECRETAGOGUES
- Are drugs which increase the amount of insulin secreted by the pancreas.
- Include: Sulfonylureas and Meglinitides
Mechanism of Action of SULFONYLUREAS
- Stimulate insulin release from functioning B cells of ATP-Sensitive K Channels which causes
depolarization and opening of voltage-dependent calcium channels, which causes an increase in
intracellular calcium in the beta cells, which stimulates insulin release.
47 | P h a r m a c o l o g y
Pharmacokinetics of Sulfonylureas
- Orally, well absorbed
- Reach peak concentration after 2-4 hours
- All are highly bound to plasma proteins
- Duration of action is variable
- Second generation has longer duration than first generation, has lesser side effects and more
effective
- Steven – Johnsons Syndrome a type III is related to sulfonylureas
- Metabolized in liver
- Excreted in urine (elderly and renal disease)
- Cross placents, stimulate fetal B-cells to release insulin – Fetal hypoglycaemia at birth
First Generation Sulfonylureas
Tolbutamide:
- Safe for diabetic patients or pts with renal impairment
Second Generation Sulfonylureas
Glipzide – Glyburide (Glibenclamide)
- More potent than first generation
- Have longer duration of action
- Less frequency of administration
- Have fewer adverse effects
- Have fewer drug interaction
48 | P h a r m a c o l o g y
Unwanted Effects:
1. Hyperinsulinemia and Hypoglycemia
- Less in tolbutamide
- More in old age, hepatic and renal diseases
2. Weight gain due to increase in appetite
3. GIT upset
Contraindications
- Hepatic impairment or renal insufficiency
- Pregnancy and Lactation
- Type 1 Diabetes
MEGLINITIDES
- Repaglinide
- Are rapidly acting insulin secretagogues
- Stimulate insulin release from functioning B-cells via blocking ATP-sensitive K-channels resulting in
calcium influx and insulin exocytosis
- Orally, well absorbed
113| P h a r m a c o l o g y

- Very fast onset of action, peak 1 hour


- Short duration of action (4 hours)
- Metabolized in liver and excreted in bile
- Taken just before each meal (3 times/day)
Uses of Meglinitides
- Type II Diabetes:
- Monotherapy or combined with metformin (Better than monotherapy)
- Specific use in patients allergic to sulfur or sulfonylureas
Adverse Effect of Meglinitides
- Hypoglycemia
- Weight gain
INSULIN SENSITIZERS
- Are drugs which increase the sensitivity of target organs to insulin
- Include: Biguanides and Thiazolidinediones (Glitazones)
Gastrointestinal drugs
Acid related pathophysiology
The stomach secretes Hydrochloric acid (HCl)- activates pepsinogen into the enzyme pepsin, which
then helps digestion by breaking the bonds linking amino acids, a process known as proteolysis.
Bicarbonate- acts to regulate pH in the small intestine. It is released from the pancreas in response to
the hormone secretin to neutralize the acidic chyme entering the duodenum from the stomach.
49 | P h a r m a c o l o g y
Pepsinogen- powerful and abundant protein digestive enzyme secreted by the gastric chief cells as a
proenzyme and then converted by gastric acid in the gastric lumen to the active enzyme pepsin.
Intrinsic factor- glycoprotein secreted by parietal (humans) or chief (rodents) cells of the gastric
mucosa. In humans, it has an important role in the absorption of vitamin B12 (cobalamin)
Mucus
Prostaglandins
Acid-controlling agents
Glands of the stomach
Cardiac
Pyloric
Gastric
o the cells of the gastric glands are the largest in number and of primary importance when discussing
acid control
Cells of the gastric gland
Hydrochloric acid
Secreted by the parietal cells when stimulated by food
Maintains stomach at pH of 1 to 4
Secretion is also stimulated by:
o Large fatty meals
o Excessive amounts of alcohol
Parietal cells
Produce and secrete HCl
Primary site of action for many acid-controller drugs
Chief cells
Secrete pepsinogen, a proenzyme
Pepsinogen becomes pepsin when activated by exposure to acid
Pepsin breaks down proteins (proteolytic)
114| P h a r m a c o l o g y

Mucoid cells
Mucus-secreting cells (surface epithelial cells)
Provide a protective mucus coat
Protects against self-digestion by HCl
Acid related diseases
Caused by imbalance of the three cells of the gastric gland and their secretions
Most common: hyperacidity
Clients reports symptoms of overproduction of HCl by the parietal cells as indigestion, sour stomach,
heartburn, and acid stomach
PUD: peptic ulcer disease
GERD: gastroespphageal reflux disease
Helicobacter pylori (H. pylori)
o Bacterium found in the GI tract of 90% of patients with duodenal ulcers and 70% of those with
gastric ulcers
o Combination therapy is used most often to eradicate H. pylori
Treatment for H. pylori
Eight regimens approved by the FDA
H. pylori is not associated with acute perforating ulcers
It is suggested that factors other than the presence of H. pylori lead to ulceration
Types of acid-controlling agents
50 | P h a r m a c o l o g y
Antacids
𝐻2 antagonists
Proton pump inhibitors
Antacids: mechanism of action
Promotes gastric mucosal defense mechanisms
Secretion of:
o Mucus: protective barrier against HCl
o Bicarbonate: helps buffer acidic properties of HCl
o Prostaglandins: prevent activation of proton pump which results to lower HCl production
Antacids do not prevent over production of acids
Antacids do neutralize the acid once it’s in the stomach
Antacids: drug effects
Reduction of pain associated with acid-related disorders
o Raising gastric pH from 1.3 to 1.6 neutralizes 50% of the gastric acid
o Raising gastric pH 1 point (1.3 to 2.3) neutralizes 90% of the gastric acid
o Reducing acidity reduces pain
Antacids: aluminum salts
Forms: carbonate, hydroxide
Have constipating effects
Often used with magnesium to counteract constipation
Examples
o Aluminum carbonate: basaljel
o Hydroxide salt: alternaGEL
o Combination products (aluminum and magnesium)
Gaviscon, Maalox, Mylanta, di-gel
Antacids: Magnesium salts
Forms: carbonate, hydroxide, oxide, trisilicate
115| P h a r m a c o l o g y

Commonly cause diarrhea; usually with other agents to counteract this effect
Dangerous when used with renal failure—the failing kidney cannot excrete extra magnesium, resulting
in hypermagnesemia
Examples:
o Hydroxide salt: magnesium hydroxide (MOM)
o Carbonate salt: gaviscon (also a combination product)
o Combination products such as Maalox, Mylanta (aluminum and magnesium)
Antacids: calcium salts
ALOH
HCl
H20
ALCL
MgOH MgOH
HCl
H20
MgCl MgCl
51 | P h a r m a c o l o g y
Forms: many, but carbonate is the most common
May cause constipation
Their use may result in kidney stones
Long duration of acid action may cause increased gastric acid secretion (hyperacidity rebound)
Often advertised as an extra source of dietary calcium
o Example: Tums (calcium carbonate)
Antacids: sodium bicarbonate
Highly soluble
Buffers the acidic properties of HCl
Quick onset, but short duration
May cause metabolic alkalosis
Sodium content may cause problems in patients with heart failure, hypertension, or renal insufficiency
(fluid retention)
Antacids and antiflatulents
Antiflatulents: used to relieve the painful symptoms associated with gas
Several agents are used to bind or alter intestinal gas and are often added to antacid combination
products
Over-the-counter antiflatulents
o Activated charcoal
o Simethicone
Alters elasticity of mucus-coated bubbles, causing them to break
Used often, but there are limited data to support effectiveness
Antacids: side effects
Minimal and depend on the compound used
o Aluminum and calcium
Constipation
o Magnesium
Diarrhea
o Calcium carbonate
Produces gas and belching; often combined with simethicone
Antacids: drug interactions
116| P h a r m a c o l o g y

Absorption of other drugs to antacids


o Reduces the ability of the other drug to be absorbed into the body
Chelation
CaCOCaCO CaCO
HCl
HCO3 HCO3
CaClCaCl CaCl
NaHCO3NaHCO3 NaHCO3
HCl
H2CO3 H2CO3H2CO3
CaClCaCl CaCl
H2O
CO2 CO2
52 | P h a r m a c o l o g y
o Chemical binding or inactivation of another drug
o Produces insoluble complexes
o Result: reduced drug absorption
Antacids: nursing implications
Asses for allergies and preexisting conditions that may restrict the use of antacids such as:
o Fluid imbalances
o Renal disease
o Heart failure
o Pregnancy
o GI obstruction
Patients with heart failure or hypertension should use low-sodium antacids such as riopan, Maalox, or
Mylanta II
Monitor for side effects
o Nausea, vomiting, abdominal pain, diarrhea
o With calcium-containing products: constipation and rebound
Monitor for therapeutic response
o Notify healthcare provider if syptoms are not relieved
Histamine type 2 (H2) receptor antagonist
Reduce acid secretion
All available OTC in lower dosage forms
Most popular drug for treatment of acid-related disorders
o Cimetidine (Tagamet)
o Famotidine (Pepcid)
o Ranitidine (Zantac)
H2 antagonists: mechanism of action
Blocks histamine 2 (H2) at the receptors of acid-producing parietal cells
Production of hydrogen ions is reduced, resulting in decreased production of HCl
H2 antagonists: indications
GERD
PUD
Erosive espphagitis
Adjunct therapy of upper GI bleeding
Pathologic gastric hypersecretory condition (Zollinger-Ellison syndrome)
H2 antagonists: side effects
117| P h a r m a c o l o g y

Overall, less than 3% incidence of side effects


Cimetidine may induce impotence and gynecomastia
May see
o Lethargy, headaches, confusion, diarrhea, urticaria, sweating, flushing, and other effects
H2 antagonists: nursing implications
Assess for allergies and impaired renal or liver function
Use with caution with patients who are confused, disoriented, or elderly (higher incidence of CNS side
effects)
Take 1 hour before or after antacids
For intravenous doses, follow administration guidelines
H2 antagonists: drug interactions
Smoking has been shown to decrease the effectives of H2 blockers
o Increases gastric acid production
Proton pump inhibitors
The parietal cells release positive hydrogen ions (protons) during HCl production
This process is called the proton pump
H2 blockers and antihistamines do not stop the action of this pump
Proton pump inhibitors: mechanism of action
Irreversibly bind to H*/K* ATPase enzyme
Result: achlrohydria—all gastric acid secretion is blocked
Proton pump inhibitors: indications
GERD maintenance therapy
Erosive esophagitis
Short-term treatment of active duodenal and benign gastric ulcers
Zollinger-Ellison syndrome
Treatment of H. pylori- induced ulcers
Proton pump inhibitors: drug effects
53 | P h a r m a c o l o g y
Total inhibition of gastric acid secretion
o Lansoprazole (prevacid)
o Omeprazole (prilosec)- the first in this new class of drugs
o Rabeprazole (acipHex)
o Pantoprazole (protonix)
o Esomeprazole (Nexium)
Proton pump inhibitors: nursing implications
Assess for allergies and history of liver diseases
Pantoprazole (protonix) is the only proton pump inhibitor available for for parenteral administration,
and can be used for patients who are unable to take oral medications
May increase serum levels of diazepam, phenytoin, and cause increased chances for bleeding with
warfarin
Instruct the patient taking omeprazole (prisolec):
o It should be taken before meals
o The capsule should be swallowed whole, not crushed, opened, or chewed
o It may be given with antacids
o Emphasize that the treatment will be short-term
Proton pump inhibitors: side effects
Safe for short-term therapy
Incidence is low and uncommon
118| P h a r m a c o l o g y

Other drugs
Sucralfate (Carafate)
Misoprostol (Cytotec)
Sucralfate (Carafate)
Cytoprotective agent
Used for ulcers, erosions, PUD
Attracted to and binds to the base of ulcers and erosions, forming a protective barrier over these
areas.
Protects these areas from pepsin, which normally breaks down into proteins (making ulcers worse)
Little absorption from the gut
May cause constipation, nausea, and dry mouth
May impair absorption of other drugs, especially tetracycline
Binds with phosphate; may be used in chronic renal failure to reduce phosphate levels
Do not administer with other medications
Misoprostol (cytotec)
Used for prevention of NSAID- induced gastric ulcers
Doses that are therapeutic enough to treat duodenal ulcers often produce abdominal cramps, diarrhea
Synthetic prostaglandin analog
Prostaglandins have cytoprotective activity
o Protect gastric mucosa from injury by enhancing local production of mucus or bicarbonate
o Promote local cell regeneration
o Help to maintain mucosal blood flow
Vomiting center and chemoreceptor trigger zone
VC and CTZ
Both located in the brain
o Once stimulated, cause the vomiting reflex
o Found in the medulla oblongata
Definitions
Nausea- unpleasant feeling that often precedes vomiting
Emesis (vomiting)- forcible emptying of gastric and, occasionally, intestinal contents
Antiemetic agents- used to relieve nausea and vomiting
Indications
Specific indications vary per class of antiemetics
General use: prevention and reduction of nausea and vomiting
Mechanism of actions and indications
Many different mechanisms of action
Most work by blocking one of the vomiting pathways, thus blocking the stimulus that induces vomiting
Antihistamine agents (H1 receptor blockers)
o Inhibit ACh by binding to H1 receptors
o Prevent cholinergic stimulation in vestibular and reticular areas, thus preventing nausea and
vomiting
o Diphenhydramine (Benadryl), meclizine (antivert, promethazine (Phenergan)
o Also used for nonproductive cough, allergy symptoms, and sedation
Anticholinergic agents (ACh blockers)
o Binds to and blocks acetylcholine (ACh) receptors in the inner ear labyrinth
54 | P h a r m a c o l o g y
o Block transmission of nauseating stimuli to the CTZ
o Also block transmission of nauseating stimuli from the reticular formation to the VC
119| P h a r m a c o l o g y

o Scopolamine
o Also used for motion sickness
Prokinetic agents
o Block dopamine in the CTZ
o Cause CTZ to be desensitized to impulses it receives from the GI tract
o Also stimulates peristalsis in the GI tract, enhancing emptying of the stomach contents
o Metoclopramide (reglan)
o Also used for GERD, delayed gastric empting
Neuroleptic agents
o Block dopamine receptors in the CTZ
o Chlorpromazine (thorazine)
o Prochlorperazine (compazine)
o Also used for psychotic disorders, intractable hiccups
Tetrahydrocannabinoids (THC)
o Major psychoactive substance in marijuana
o Inhibitory effects on reticular formation, thalamus, and cerebral cortex
o Alter mood and body’s perception of its surroundings
o Dronabinol (marinol)
o Used for nausea and vomiting associated with chemotherapy and anorexia associated with weight
loss in AIDS patients
Serotonin blockers
o Blocks serotonin receptors in the GI tract, CTZ, and VC
o Dolaseltron (Anzemet), granisetron (kytril), ondansentron (Zofran)
o Used for nausea and vomiting for patients receiving chemotherapy and postoperative nausea and
vomiting
Nursing implications
Assess complete nausea and vomiting history, including precipitating factors
Assess current medications
Assess for contraindications and potential drug interactions
Many of these agents cause severe drowsiness; warn patients about driving or performing hazardous
tasks
Taking antiemetics with alcohol may cause severe CNS depression
Teach patients to change positions slowly to avoid hypotensive effects
For chemotherapy, antiemetics often give ½ to 3 hours before a chemotherapy agent
Monitor for therapeutic effects
Monitor for adverse effects
Side effects
Vary according to agent used
Stem from their nonselective blockade of various receptors

You might also like